ACCP Pharmacotherapy

Pataasin ang iyong marka sa homework at exams ngayon gamit ang Quizwiz!

The following four patients have very poorly controlled asthma on high dose inhaled corticosteroids (ICS) plus a LABA. Which patient would benefit most from a trial of golimumab? A. A fifty-year-old architect with severe plaque psoriasis. B. An 18-year-old College freshman with irritable bowel disease. C. A 62-year-old retired man with osteoarthritis of the knee. D. A 24-year-old man with poorly controlled bipolar disorder.

A fifty-year-old architect with severe plaque psoriasis. (Golimumab, A human monoclonal antibody Tumor Necrosis Factor Alpha (TNF-α), Is most likely to benefit the patient with concomitant inflammatory disorders (poorly controlled asthma) and severe plaque psoriasis.)

You are interested in determining the effect of long-term use of a drug on patient height. You compare results between groups of children 2-5, 6-10, 11-14, and 15-18 years of age. Which of the following statistical tests is the most appropriate choice? A. Kruskal-Wallis. B. Chi-square. C. ANOVA. D. Paired t-test.

Answer C: ANOVA. (The ANOVA test is a generalization of the t-test for comparing means resulting from parametric data across more than two groups, and this study compares four age groups (Answer C is correct). The Kruskal-Wallis test is a nonparametric analog of the ANOVA procedure (Answer A is incorrect). Height is measured on a continuous, not categorical, scale, and chi-square tests can be used only with categorical data (Answer B is incorrect). The paired t-test is used for comparing data from just two groups when data are paired, as in a pre-post design. Neither of these conditions applies in this case (Answer D is incorrect).)

A woman who is 18 weeks' pregnant is seeking treatment for moderate-to-severe plaque psoriasis. Which one of the following approaches to therapy is best for this patient? A. Moderate potency corticosteroids applied daily. B. Oral acitretin once daily. C. Oral methotrexate once weekly. D. Subcutaneous etanercept twice weekly.

Answer A: Moderate potency corticosteroids applied daily. (The available agents to treat psoriasis in pregnant women are limited by a lack of evidence to support their safety to the mother and fetus. Current recommendations from the National Psoriasis Foundation and their review of treatment options suggest that low- to mid-potency steroids should be used as a first-line therapy for pregnant women with psoriasis (Answer A is correct). Drugs such as acitretin and methorexate should be avoided because they are known teratogens (Answer B and Answer C are incorrect). Although no negative fetal-outcome data are associated with use of etanercept, the available cohort of patients in the literature is scarce and a true safety evaluation is not available (Answer D is incorrect). For this reason, TNF-a-inhibitors are not preferred (and are third-line agents) for treating psoriasis in pregnant women.)

A 47-year-old man with a history of coronary artery disease. heart failure, hypertension, and diabetes mellitus is admitted to the cardiac ICU with unstable angina and is awaiting coronary artery bypass graft surgery. He develops a severe retroperitoneal bleed from heparin bcjöre surgery and is administered protamine by a central intravenous line. The patient's other drugs include carvedilol 12.5 mg two times/day. insulin, aspirin 81 mg/day, clopidogrel 7S mg/day, and enalapril 10 mg two times/day, patient develops bronchospasm and generalized erythema, His systolic blood pressure decreases from 110 mm to 80 mm Hg; pulse rate is 120 beats/minute; and Hgb and Hct are 8.2 g/dL and 26%, respectively. Which of the following is the best initial therapy for this patient? A. Normal saline 2-L bolus and epinephrine 0,1 mg intravenously via central line over 5 minutes. B. Epinephrine I-mg intravenous push every to S minutes, 2 units oc packed red blood cells, and 2 units of fresh frozen plasma. C. Epinephrine 0.3 mg intramuscularly in the lateral thigh. reverse Trendelenburg positioning. and glucagon 2 mg intramuscularly. D. Two units of' packed red blood cells, epinephrine 2-mcg/ minute continuous infusion, glucagon.

Answer A: Normal saline 2-L bolus and epinephrine 0.1 mg intravenously over 5 minutes. (The patient is experiencing anaphylaxis and potentially anaphylactic shock, and he has a known significant bleed with hypotension. He likely has a combination of hypovolemic and distributive shock; therefore, intravascular volume replacement must be part of the initial medical management. Bolus administration of crystalloids is a more rapid approach to address intravascular volume initially than blood products (Answer A is correct). The dose and administration of epinephrine in Answer A is also correct because this patient already has a central line. Blood products may still be needed, but they are not appropriate as initial management (Answer B and Answer D are incorrect). Epinephrine I-mg intravenous push every 3-5 minutes would be correct if the patient were in cardiac arrest (Answer B is incorrect). Likewise, epinephrine 0.3 mg intramuscularly in the lateral thigh would be correct in the absence of central venous access (Answer C is incorrect). The patient was receiving ß-blocker therapy, which can worsen anaphylaxis and counter the effects of catecholamine therapy. Thus, glucagon would be appropriate. However, reverse Trendelenburg positioning would not be as effective for increasing blood pressure and may not be well tolerated in a patient with HF (Answer C is incorrect).)

A 57-year-old man with ischemic cardiomyopathy (LVEF 30%) presents to the ED with dyspnea, a 4-kg weight gain, and fatigue. His vital signs include BP 90/75 mm Hg, HR 92 beats/minute, respiratory rate 22 breaths/minute, and Sao 95% on 4L nasal cannula. Physical examination reveals a 15-cm J V D, crackles bilaterally, regular rhythm and rate, and 3+ pitting edema. Pertinent laboratory values include Na 126 mEq/L, K 3.9 mEq/L, BUN 50 mg/dL, and SCr 2.1 mg/dL (baseline 1.8 mg/dL). His home drugs include lisinopril 20 mg daily, carvedilol 12.5 mg twice daily, furosemide 40 mg twice daily, and spironolactone 25 mg daily. He is adherent to both his dietary restrictions and medications. Which one of the following is best to recommend for this patient? A. Increase carvedilol to 25 mg twice daily. B. Administer furosemide 40 mg intravenously. C. Discontinue spironolactone. D. Add nitroglycerin 5 mcg/min intravenously.

Answer B: Administer furosemide 40 mg intravenously. (Intravenous loop diuretics are recommended first-line therapy for patients who present with signs of fluid overload. Usually, the home dose of loop diuretic is at least doubled to provide a potent diuresis. Because the bioavailability of oral furosemide is around 50% of the intravenous dose, furosemide 40 mg intravenously would double his home regimen and be a reasonable place to start (Answer B is correct). Vasodilator therapy such as nitroglycerin may be considered in addition to loop diuretics to rapidly improve symptoms in patients with acute pulmonary edema or severe HTN, neither of which this patient has at this time. Loop diuretics should be tried first, however (Answer D is incorrect). Increasing carvedilol is appropriate only when patients are euvolemic; this patient has signs of fluid overload (Answer A is incorrect). Contraindications to spironolactone include a K level greater than 5.0 mmol/L and a SCr of 2.5 mg/dL (in men) or 2.0 mg/dL (in women). At this time, there is no clinical reason to discontinue spironolactone (Answer C is incorrect).)

A 70-year-old man receives a diagnosis of ischemic stroke, with onset of symptoms 3.5 hours ago. He has a medical history of H TN, DM, and deep venous thrombosis (DVT). He is currently taking warfarin and has an INR of 1.1. Which one of the following factors excludes this patient from receiving alteplase under the 3- to 4.5-hour expansion time window in ischemic stroke? A. His age (70 years). B. Current warfarin therapy. C. History of diabetes. D. Platelet count of 100,000 mm^3

Answer B: Current warfarin therapy. (Taking any oral anticoagulants, regardless of INR, is an exclusion criterion for the expanded time window (Answer B is correct). Exclusion criteria for the 3- to 4.5-hour expansion include patients older than 80 (Answer A is incorrect) and those with a history of both stroke and diabetes (Answer C is incorrect). Platelet count is not incorporated into the exclusion criteria for giving lytic agents within the time expansion window (Answer D is incorrect).)

A 57-year-old man (height 172 cm, weight 94 kg) with a history of HTN is brought to the ED by ambulance after 8 hours of substernal chest pressure at rest. During the past 2 weeks, patient has noticed fleeting twinges of chest pressure after exertion, but these episodes have been relieved by rest. While walking his dog today at 5:00 p.m., he developed heavy chest pressure. Once at home, he rested without relief of chest discomfort. At 6:30 p.m., he called 9-1-1. En route to the hospital, paramedics administered three sublingual nitroglycerin tablets and aspirin 325 mg orally without relief. The patient takes aspirin 81 mg once daily and metoprolol extended release 50 mg orally once daily. He denies any recent trauma or bleeding tendencies. His Vital Signs include BP 120/80 mm Hg and heart rate (HR) 55 beats/minute, and he is afebrile. Physical examination is significant for regular rate and rhythm and normal S1 and S2; a fecal occult blood test is negative. The patient has no signs acute heart failure (HF). Electrocardiography (ECG) performed 5 minutes after presentation to the ED reveals normal sinus rhythm With ST-segment elevation in the inferior leads. His chemistry panel and complete blood cell count (CBC) are normal; SCr is 1.0 mg/dL, and is elevated. The hospital does have a cardiac catheterization laboratory. Which one of the following would best reduce mortality in this patient? A. Alteplase, intravenous UFH bolus and infusion, and oral metoprolol. B. Enoxaparin intravenous bolus, followed by subcutaneous dosing, and oral metoprolol. C. Abciximab, intravenous UFH bolus and infusion, and clopidogrel. D Tenecteplase, clopidogrel intravenous bolus, followed by subcutaneous enoxaparin.

Answer D: Tenecteplase, elopidogrel intravenous bolus, followed by subcutaneous enoxaparin (The patient presents within 12 hours of the onset of chest discomfort, so reperfusion therapy is indicated. Because the hospital has no interventional cardiology facilities, options for reperfusion include interhospital transfer or administration of fibrinolytics. Low-molecular-weight heparins (LMWHs) are preferred to UFH with fibrinolytics because enoxaparin has shown a reduction in death or MI compared with UFH. Tenecteplase is a fibrinolytic option for reperfusion therapy, clopidogrel is given to reduce mortality, and enoxaparin is given to reduce death or MI (Answer D is correct). Therapy options without reperfusion therapy (either mechanical or pharmacologic) are not appropriate (Answer B and Answer C are incorrect). This patients baseline HR is less than 60 beats/minute, which is a risk factor for the development of heart block caused by β-blocker administration. β-blocker therapy may be deferred to later in the first 24 hours because early administration to hemodynamically unstable patients may precipitate cardiogenic shock. In addition, no clopidogrel is administered, and it is a less-than-optimal choice because UFH is administered (Answer A and Answer B are incorrect). Glycoprotein IIb/IIIa inhibitors are administered only for primary PCI in STEMI and not in combination with a fibrinolytic or when no reperfusion therapy is given (Answer C is incorrect).)

A 32-year old woman with persistent asthma has shortness of breath (SOB) and wheezing that occur twice weekly. Her medical history includes allergic rhinitis, depression, and polycystic ovary syndrom. She uses her albuterol hydrofluoroalkane (HFA) inhaler twice weekly, usually when outside at the park. She wakes up at night coughing about once weekly. Her current drugs include albuterol HFA 1 or 2 puffs every 4-6 hours as needed for SOB, fluticasone/salmeterol Diskus 250/50 1 puff twice daily, loratadine 10 mg once daily for "allergies", fluticasone nasal spray 50 mcg 2 sprays each nostril once daily, and metformin 500 mg twice daily. Sertraline 50 mg once daily was initiated 2 months before this presentation. She does not have a peak flow meter, but she follows her asthma action plan. The patient cannot remember the last time she had to visit the emergency department (ED) for an exacerbation. What best addresses the patient's current symptoms?

Add montekulast 10 mg once daily at bedtime. (This patient's asthma is not well controlled (symptoms occur more than 2 days/week, bedtime symptoms occur 1-3 times a week, and the patient requires short-acting β-agonist (SABA) use 2 days/week). The patient may benefit from the addition of montelukast (an alternative agent) because she has allergic rhinitis (already uses intranasal steroid and daily antihistamine) and reports that the symptoms usually occur when outdoors (pollen exposure), which may mean that an allergic component is worsening her symptoms. Montelukast should not be withheld from a patient who might benefit from it, even if the patient has concomitant depression.)

A randomized controlled trial assessed overall differences between two drug treatments. The trial enrolled 90 patients, who were then divided equally between treatments; all patients completed the trial. Of those who received drug A, 29 experienced the outcome. Of those who received drug B, 14 experienced the outcome. Which one of the following is the best estimate of the odds ratio for this study comparing the experience of individuals who received drug A, relative to those who received drug B? A. 40 B. 0.25 C. 1.1 D. 0.9

Answer A: 4.0 -------------------------------------------------- Drug || Outcome Yes || Outcome no || Total -------------------------------------------------- Drug A || 29 || 16 || 45 -------------------------------------------------- Drug B || 14 || 31 || 45 -------------------------------------------------- Total || 43 || 47 || 90 --------------------------------------------------- (The odds ratio is calculated by the odds of having the outcome with drug A (A/B) relative to the odds of having the outcome with drug B (C/D), or A/B over C/D, which converts to AD/BC (also known as cross-multiplying and dividing the observed number of events). In this case, this calculation translates to ([29/161/[14/311) or (29 x 31)/(16 x 14) = 4.01 (Answer A is correct). An odds ratio of 0.25 is incorrectly calculated by cross-multiplying and dividing BC by AD (Answer B is incorrect). An odds ratio of 1.1 is incorrectly calculated by cross-multiplying and dividing AB by CD (Answer C is incorrect). An odds ratio of 0.9 is incorrectly calculated by cross-multiplying and dividing CD by AB (Answer D is incorrect).)

Which one of the following scenarios does not require mandatory reporting to the FDA? A. A patient who received concurrent digoxin and amiodarone therapy and subsequently developed digoxin toxicity requiring the administration of digoxin immune fab. B. A hospitalized patient whose death was suspected to have been the result of an overdose of morphine brought about by infusion pump failure. C. A 25-year-old woman currently participating in a clinical trial comparing combined oral contraceptives who develops an acute deep venous thrombosis. D. A pharmaceutical company that receives a report from a physician that a 65-year-old man developed a subdural bleed while taking dabigatran.

Answer A: A patient who received concurrent digoxin and amiodarone therapy and subsequently developed digoxin toxicity requiring the administration of digoxin immune fab. (Reporting of adverse events and medication errors by health care professionals and consumers is voluntary in the United States. An adverse drug reaction (ADR) report to the FDA for the patient who developed digoxin toxicity would be voluntary, not required (Answer A is correct). Institutions such as hospitals are required to report deaths or serious injury potentially attributable to a medical device to the FDA and to the device manufacture, if known (Answer B is incorrect). Any ADR that occurs during a clinical trial is mandated to be reported to the FDA (Answer C is incorrect). Health care professionals and consumers may also report adverse events and/or medication errors to the products' manufacturers. If a manufacturer receives an adverse event report, it is required to send the report to the FDA as specified by regulations (Answer D is incorrect).)

A 76-year-old woman presents to the clinic with mild to moderate right knee pain related to osteoarthritis (OA). Her medical history is also significant for HTN, coronary artery disease, and hyperlipidemia. Her home drugs include carvedilol 25 mg twice daily, atorvastatin 20 mg daily, aspirin 81 mg daily. nitroglycerin 0.4 mg sublingually as needed for chest pain, and isosorbide dinitrate extended release 20 mg twice daily. Ranolazine 500 mg twice daily was just added to therapy regimen because of continued angina symptoms, patient's vital signs today include blood pressure 122/72 mm Hg and heart rate 66 beats/minute. She is allergic to penicillin and sulfonamides. Which of the following is the best recommendation for her OA-related pain? A. Acetaminophen 500 mg every 4 hours, B. Ibuprofen 800 mg every 4 hours. C. Celecoxib 200 mg twice daily. D. Tramadol 100 mg every 6 hours.

Answer A: Acetaminophen 500 mg every 4 hours. (According to the American College of Rheumatology osteoarthritis guidelines, acetaminophen should considered as first-line therapy for the management of mild to moderate knee or hip OA (Answer A is correct). Although nonselective NSAIDs are also effective for OA, the dose of ibuprofen exceeds the maximal daily dose of 3200 mg and should be avoided because of the risk of cardiovascular events in a patient with controlled disease (Answer B is incorrect). Celecoxib can be used in dosages of up to 400 mg/day; however, it should be avoided in patients with poorly controlled coronary artery disease and in those allergic to sulfonamides (Answer C is incorrect). Tramadol is indicated for moderate to moderately severe pain and should used at doses Of no more than 300 mg/day in patients older than 75 years (Answer D is incorrect).)

A 27-year-old man presents to the clinic for treatment of a first episode of genital herpes. He is unemployed and does not have prescription insurance. He has no known drug allergies or significant medical history. Which one of the following oral regimens is most appropriate for this patient? A. Acyclovir 400 mg three times/day for 7 days. B. Acyclovir 400 mg four times/day for 5 days. C. Famciclovir 250 mg three times/day for 7 days. D. Famciclovir 125 mg two times/day for 5 days.

Answer A: Acyclovir 400 mg three times/day for 7 days. (Acyclovir is the least expensive option of the three drugs approved for treating genital herpes. For treatment of a first episode of genital herpes, acyclovir 400 mg three times/day (Answer A is correct) or 200 mg five times/day for 7—10 days are two of the recommended regimens. One drawback to these options is the need for multiple daily doses. Acyclovir 400 mg four times/day for 5 days is not an appropriate frequency and duration of therapy (Answer B is incorrect). Famciclovir 250 mg three times/day for 7 days is also an appropriate regimen; however, it is more expensive than acyclovir. This patient is currently unemployed and lacks prescription insurance (Answer C is incorrect). Famciclovir 125 mg two times/day for 5 days may be used for treatment of recurrence; however, it is not an appropriate regimen for treating a first episode (Answer D is incorrect).)

You and your colleagues conduct a randomized, double-blind, controlled study of a drug. In constructing the preplanned tables, you find imbalances in the distribution of clinical and demographic characteristics between the experimental groups. Which one of the following is most likely to be of concern to your study team? A. Confounding. B. Immortal-time bias. C. Selection bias. D. Information bias.

Answer A: Confounding. (Imbalances between experimental groups are a necessary condition for confounding (Answer A is correct). Immortal-time bias is caused by including individuals who are not at risk of the event in the incidence rate denominator (Answer B is incorrect). Selection bias pertains to study participation (Answer C is incorrect). Information bias refers to errors in what is known about the study participants (Answer D is incorrect).)

A 9-year-old girl with cystic fibrosis (CF) is having difficulty gaining weight. She also has large-volume greasy stools several times each day. Her current enzyme dose provides lipase 10,000 units/kg/day. She consistently takes her enzymes immediately before eating. A nutritional assessment reveals she is ingesting adequate calories in her diet, about 30% of which are from fat. Which one of the following is best to recommend for improving this child's apparent malabsorption? A. Add a proton pump inhibitor (PPI) daily. B. Decrease pancreatic enzyme to lipase 7500 units/kg/day. C. Increase pancreatic enzymes to lipase 12,000 units/kg/day. D. Decrease daily fat intake.

Answer A: Add a PPI daily. (This patient is receiving the maximal recommended daily dose of lipase (10,000 units/kg). Some patients require higher doses, but the risk of fibrosing colonopathy increases as doses increase. Therefore, increasing lipase to 12,000 units/kg/day would not be recommended (Answer C is incorrect). Her daily dosage should not be decreased to 7500 units/kg because she has symptoms of malabsorption (Answer B is incorrect). If a decrease were needed, then a more gradual decrease would be recommended. Because patients with CF have decreased pancreatic secretion of bicarbonate, microencapsulated enzyme products may not release optimally in the intestine. Adding a PPI has improved steatorrhea in patients with CF and is best to recommend (Answer A is correct). Her current fat intake (30% of total calories) is appropriate and may be increased to 40%. Therefore, decreasing her fat intake would not be recommended (Answer D is incorrect).)

You are providing care to a patient with a history of multiple myeloma whose cells have engrafted 14 days after his stem cell reinfusion and will be discharged tomorrow. He has been receiving lorazepam 1 mg two times/day for nausea and oxycodone 10 mg two times/day for resolving mucositis. His laboratory values, which have been stable during the past 3 days, are: Na 131 mEq/L, K 3.3 mEq/L, Cl 99 mEq/L, HCO - 18 mEq/L, BUN 22 mg/dL, SCr 0.9 mg/dL, serum glucose 125 mg/dL, total serum calcium 7.5 mg/dL, and albumin 2 g/dL. Which one of the following is the most appropriate change for this patient's discharge prescriptions? A. Add potassium supplement. B. Increase lorazepam dose to 2 mg. C. Add long-acting oxycodone. D. Add calcium supplement.

Answer A: Add potassium supplement. (Even with resolving mucositis, most patients will have some element of diarrhea when recovering from high-dose chemotherapy. In addition, this patient has mild hypokalemia with a potassium concentration of 3.3 mEq/L. The patient will most likely continue to have electrolyte imbalances requiring replacement for a few weeks and should be continually assessed at follow-up appointments (Answer A is correct). With engraftment comes the resolution of mucositis and usually nausea/vomiting, so it is expected that the patient will need fewer antiemetics or narcotics at discharge (Answer B and Answer C are incorrect). Calcium supplementation is an option, but the corrected calcium is within an acceptable range, whereas the potassium is more critical (Answer C is incorrect).)

Which one of the following is the best example of an American College of Cardiology (ACC)/American Heart Association (AIIA) performance measure for the treatment of myocardial infarction (MI)? A. Administration of an angiotensin receptor blocker at hospital discharge in a patient with an ejection fraction of 35%. B. Administration of an excessive dose of eptifibatide to a patient with a creatinine clearance of 30 mL/minute. C. Administration of metoprolol 5 mg intravenously in the emergency department. D. Measurement of a baseline LDL-C concentration before administration ofa lipid-lowering agent.

Answer A: Administration of an angiotensin receptor blocker at hospital discharge in a patient with an ejection fraction of 35%. (A performance measure is selected for widespread use because of discussion and consensus, whereas a test measure is recommended, but not required, in order for data to be collected for future consideration as a performance measure. Excessive dosing of glycoprotein IIb/IIIa inhibitors is a test measure, not a performance measure (Answer B is incorrect). Because statin therapy is recommended even in the absence of a baseline LDL-C concentration, the test measure of baseline LDL-C measurement was removed from the ACC/AHA performance measures (Answer D is incorrect). Because the COMMIT trial showed that administering early ß-blockade to all patients does not reduce mortality and may increase mortality in patients with signs of heart failure at baseline, ACCIAHA removed the early administration of a ß-blocker as a performance measure (Answer C is incorrect). Either an angiotensin receptor blocker or an angiotensin-converting enzyme inhibitor is indicated in all patients with a reduced ejection fraction (40% or less) without contraindication at hospital discharge and is an ACC/AHA performance measure (Answer A is correct).)

A 45-year-old African American man presents to the clinic for an evaluation of H TN. His medical history is significant for seasonal allergies and osteoarthritis. His current drugs include diclofenac 4% gel applied to both knees twice daily and loratadine 10 mg daily. His self-monitored BP log shows readings of 138—150/88—96 mm Hg. Today in the clinic, his BP is 142/92 mm Hg, and HR is 72 beats/minute. His laboratory values are within normal limits. Which one of the following is best to recommend for this patient? A. Amlodipine 5 mg daily; titrate to goal BP less than 140/90 mm Hg. B. Carvedilol 3.125 mg twice daily; titrate to goal BP less than 140/90 mm Hg. C. Hydrochlorothiazide 12.5 mg daily; titrate to goal BP less than 130/80 mm Hg. D. Lisinopril 10 mg daily; titrate to goal BP less than 130/80 mm Hg.

Answer A: Amlodipine 5 mg daily; titrate to goal BP of less than 140/90 mm Hg. (According to the 2014 report by the panel members appointed to the Eighth Joint National Committee (JNC 8), dihydropyridine CCBs and thiazide diuretics are recommended as first-line agents in treating patients of African American descent. The goal BP Cor the general patient population without chronic kidney disease or DM is less than 140/90 mm Hg (Answer A is correct). Because ß-blockers are not recognized as useful first-line agents in patients without a compelling reason, thiazide-like diuretics, ACE inhibitors, ARBs, or CCBs should be considered first (Answer B is incorrect). Although hydrochlorothiazide is a first-line agent, no current evidence suggests that treatment to a goal BP of less than 130/80 mm Hg is more beneficial than less than 140/90 mm Hg. (Answer C is incorrect). Lisinopril is not the best option because of the lower BP target and because ACE inhibitors are less effective as monotherapy in reducing BP compared with dihydropyridine CCBs in African American patients (Answer D is incorrect).)

A 2-month-old boy who underwent surgical repair of a large ventricular septal defect now has clinical and echocardiographic signs of pulmonary arterial hypertension (PAH). On cardiac catheterization, his mean pulmonary arterial pressure (MPAP) was 65 mm Hg, PCWP (pulmonary capillary wedge pressure) was 10 mm Hg, and P VR (pulmonary vascular resistance) was 4 Wood units. On challenge with an Fi02 (fraction of inspired oxygen) of 100% and inhaled nitric oxide (NO) at a concentration of 20 ppm, his MPAP declined to 25 mm Hg. He was transferred back to the pediatric ICU (intensive care unit) on inhaled NO at a concentration of 20 ppm. Which one of the following drugs is the most appropriate initial therapy if this patient requires long-term PAH treatment? A. Amlodipine. B. Bosentan. C. Epoprostenol. D. Sildenafil.

Answer A: Amlodipine. (This patient has vasoresponsive PAH according to the American Academy of Cardiology criteria. According to the Joint American College of Cardiology Foundation and American Heart Association (ACCF/AHA) 2009 expert consensus guidelines on pediatric pulmonary hypertension, he should be treated with a calcium channel blocker because he has new-onset vasoresponsive disease, and calcium channel blockers have shown a long-term survival benefit in children (Answer A is correct). Sildenafil may be beneficial, but fewer scientific data are available to support its use in children; also, the FDA issued a statement warning against its use in children 1—17 years of age (Answer D is incorrect). Bosentan may be beneficial, but it requires patients to have monthly blood sampling to screen for hepatotoxicity. Furthermore, because this patient will be unable to swallow a tablet, he would need to have a suspension of bosentan prepared daily because the drug is unstable in aqueous solution. Amlodipine can be prepared as a I-mg/mL oral suspension that is stable for 56 days at room temperature (Answer B is incorrect). Epoprostenol may be beneficial; its success in the long-term treatment of children is well established. However, its use requires the placement of a central venous catheter and continuous intravenous infusion (Answer C is incorrect).)

A 58-year-old man who received a kidney transplant about 3 years ago comes to the clinic today for follow-up. He has no complaints. His current drugs include tacrolimus 2 mg twice daily, mycophenolate mofetil 500 mg twice daily, prednisone 2.5 mg daily, amlodipine 5 mg daily, metoprolol 50 mg twice daily, simvastatin 10 mg at bedtime, and sertraline 20 mg daily. He reports he had been traveling recently and did not take all of his drugs regularly. Laboratory tests are remarkable for an SCr of 4.3 mg/dL (increased from 1.4 mg/dL 3 weeks ago) and a tacrolimus trough level of 1.1 ng/mL. His biopsy reveals severe acute cellular rejection. This rejection is his second episode that is secondary to nonadherence. Which one of the following is the best recommendation for the treatment of this patient's current rejection episode? A. Antithymocyte globulin (rabbit) and corticosteroids. B. Belatacept. C. Corticosteroid bolus followed by taper. D. Tacrolimus dose increase.

Answer A: Antithymocyte globulin (rabbit) and corticosteroids. (Treatment of acute rejection, consisting of short courses of aggressive immunosuppression, is given in cases with evidence of graft dysfunction or pathologic findings to suggest immune recognition of allograft tissue. Drugs to treat rejection are selected primarily according to rejection characteristics such as the immunologic type (cellular vs. antibody mediated) and severity. In addition, the type of transplanted organ involved as well as the patient's history must be considered. In general, moderate to severe rejections are treated with agents typically classified as T- and/or B-cell depleting therapies, often with corticosteroids. Less-severe or mild rejections are treated with corticosteroid boluses, followed by a taper and possibly an increase in the number and/or dose(s) of maintenance immunosuppressive agents. This patient's biopsy-proven rejection is classified as a severe cellular rejection; thus, therapy with a T- and/or B-cell depleting therapy with corticosteroids is recommended (Answer A is correct). Corticosteroids alone are administered when treating a less severe or mild rejection (Answer C is incorrect). Belatacept is a maintenance immunosuppressant and thus not effective in reversing the acute rejection process (Answer B is incorrect). Increasing CNI dosing to achieve the upper threshold of the target ranges may be acceptable for a mild rejection, but not in this case of severe rejection (Answer D is incorrect).)

When describing the results of a randomized controlled clinical trial, the investigators report that subjects were analyzed by the actual intervention received. Which one of the following types of analysis does this approach best represent? A. As-treated analysis. B. Intention-to-treat analysis. C. Meta-analysis. D. Per-protocol analysis.

Answer A: As-treated analysis. (This study represents an as-treated analysis because subjects are analyzed by the actual intervention they received (Answer A is correct). An intention-to-treat analysis compares outcomes according to the initial group to which subjects were assigned, not on treatment eventually received (Answer B is incorrect). A meta-analysis pools the outcomes from several studies and would not be appropriate here (Answer C is incorrect). A per-protocol analysis evaluates only the subjects who complete the treatment to which they were randomized (Answer D is incorrect).)

A 55-year-old woman With a history of HTN, T2DM, and AF presents to the clinic for follow-up. Her home drugs include metformin 850 mg twice daily, lisinopril 20 mg daily. diltiazem 180 mg daily, and warfarin 5 mg daily. Her laboratory values are glucose 120 mg/dL, AIC 7.0%, SCr 1.3 mg/dL, BUN 30 mg/dL, K 4.2 mEq/L, Na 125 mEq/L, INR 2.6, TC 210 mg/dL, HDL-C 49 mg/dL, TG 200 mg/dL, and LDL-C 111 mg/dL. Vital signs today include BP 138/80 mm Hg and HR 65 beats/minute. Using the Pooled Cohort Equation, her 10-year ASCVD risk is 9.1%. Which one of the following, if added to her daily regimen, would best reduce this patient's ASCVD risk? A. Atorvastatin 40 mg. B. Rosuvastatin 10 mg. C. Pravastatin 20 mg. D. Simvastatin 40 mg.

Answer A: Atorvastatin 40 mg. (According to recent guidelines. this patient meets the criteria for a statin-benefiting group (40-75 years of age with diabetes and a 10-year ASCVD risk greater than 7.5%) thus, she would benefit from a high-intensity statin therapy. Atorvastatin 40 mg is the only option with a high-intensity statin dosage (Answer A is correct). Rosuvastatin 10 mg daily is an intermediate-intensity statin (Answer B is incorrect). Pravastatin 20 mg daily is a low-intensity statin dose (Answer C is incorrect). Simvastatin 40 mg is an intermediate statin dose and further, it is an inappropriate option in this patient because doses above 10 mg daily should be used in patients also taking diltiazem (Answer D is incorrect).)

Your institution is a study site for a large, multinational, randomized, placebo-controlled clinical trial. To evenly assign patients to the two treatments at each site, which one of the following types of randomization would be best for your study? A. Blocked. B. Covariate adaptive. C. Simple. D. Stratified.

Answer A: Blocked. (Blocked randomization is specifically used to control for differences within an individual study site (Answer A is correct). Simple randomization does not control for any specific factor (Answer C is incorrect). Stratified randomization is used to control for a confounding variable (Answer D is incorrect). In covariate adaptive randomization, a new participant is sequentially assigned to a particular treatment group by considering the specific covariates and previous assignments of participants. Covariate adaptive randomization uses the method of minimization by assessing the imbalance of a small sample size among several covariates. Because this is a large multinational trial, this method is not best applied in this case (Answer B is incorrect).)

A 61-year-old woman who experienced a seizure is brought to the emergency department for evaluation. She is initiated on phenytoin, and for the next 7 days in the hospital, she remains seizure free. The inpatient neurology team now plans to discharge the patient home on phenytoin 300 mg/day. The hospital uses the SPs tool before her discharge. The SPs tool evaluates problem medications, psychological issues, principal diagnosis, polypharmacy. low health literacy, patient support, prior hospitalizations, and palliative care. The patient's discharge medications include phenytoin, insulin glargine, glipizide, atorvastatin, lisinopril/hydrochlorothiazide, aspirin, and clopidogrel. The SPs tool notes that this hospitalization is the patient's first since her knee replacement IS months ago. After being discharged home, where she lives with her husband, she has a scheduled follow-up in the neurology clinic within I week. Which one of the following areas covered by the SPs assessment is most likely to show this patient to be at risk of a postdischarge adverse event? A. Problem drugs. B. Principal C. Patient support. D. Previous hospitalizations.

Answer A: Problem drugs. (Many validated tools to evaluate general and health literacy have been published in the literature. Because nonadherence and adverse events are higher among patients with low health literacy, a simple screening tool is useful for clinicians in assessing this risk factor. The patient's drug list includes insulin, aspirin, and clopidogrel; in addition to warfarin and digoxin, these three drugs increase the likelihood of adverse events after discharge (Answer A is correct). Cancer, stroke, diabetes/glycemic complications, chronic obstructive pulmonary disease, and heart failure have been identified as principal diagnoses likely to result in rehospitalization (Answer B is incorrect). Factors less likely to result in an adverse event are a support system for a patient whose spouse is at home and a previous hospitalization that was more than 6 months ago (Answer C and Answer D are incorrect).)

A 69-year-old woman (height 65 inches [165 cm], weight 65 kg) has patient group C/stage 3 chronic obstructive pulmonary disease (COPD), angina, hypertension, and depression. She also has a history of tobacco smoking for 35 years but is ready to quit smoking. Her home drugs include amlodipine 10 mg orally daily and a fluticasone 250 mcg/salmeterol 50 mcg inhaler twice daily. The patient's vital signs include the following: BP 180/90 mm Hg, HR 92 beats/minute, and respiratory rate (RR) 18 breaths/minute. Her CrCl is 50 mL/minute/1.73m^2. Which one of the following is the most appropriate choice for smoking cessation for this patient? A. Bupropion SR. B. Nicotine patch. C. Nicotine inhaler. D. Varenicline.

Answer A: Bupropion SR. (This patient, who wishes to quit smoking, has a history of depression. Of the choices listed, bupropion is least likely to have a drug-disease interaction or cause harmful adverse effects and may have the added benefit of improving her depression (Answer A is correct). Nicotine-based options are not preferred in this patient with a history of hypertension (currently uncontrolled, based on vital signs) and angina; both arc precautions to using nicotine patches or an inhaler (Answer B and Answer C are incorrect). Both cardiovascular disease and depression are precautions for using varenicline (Answer D is incorrect).)

A 63-year-old man has a medical history of hypertension, hyperlipidemia, and benign prostatic hyperplasia. He has been referred to the CKD clinic because his SCr has been increasing for the past several years. Today, his glomerular filtration rate is about 38 mL/minute/1.73m2. His laboratory values reveal calcium 8.6 mg/dL, albumin 4.0 g/dL, phosphorus 3.9 mg/dL, iPTH 159 pg/mL (increased from 87 pg/mL 6 months ago), and 25-hydroxyvitamin D 42 ng/mL. Which one of the following would best treat this patient's metabolic abnormalities? A. Calcitriol 0.25 mcg by mouth once a day. B. Doxercalciferol 4 mcg by mouth once a day. C. Cinacalcet 30 mg by mouth once a day. D. Sevelamer 403 mg by mouth three times a day with meals.

Answer A: Calcitriol 0.25 mcg by mouth once a day. (Newer guidelines focus on changes in iPTH rather than absolute values, so the increase in his iPTH is of concern. Initial therapy should focus on controlling hyperphosphatemia, hypocalcemia, and vitamin D deficiency. Because other metabolic abnormalities are not present, active vitamin D therapy is recommended to suppress PTH secretion. Calcitriol is activated vitamin D3 that will increase intestinal calcium absorption and decrease PTH secretion (Answer A is correct). Doxercalciferol is a possibility; however, it is more expensive than calcitriol (Answer B is incorrect). Cinacalcet would not be a good choice because this patient's calcium level is at the low end of the target range (Answer C is incorrect). He does not need a phosphate binder at this time because his current serum phosphorus concentration is 3.9 mg/dL, well below the target range for his stage 3 CKD (Answer D is incorrect).)

A clinical trial evaluated the use of interferon in the treatment of multiple sclerosis. Forty-two patients were enrolled in a randomized, placebo-controlled, crossover study. A review of study subjects revealed no statistical differences in age, sex, or length of disease. In evaluating the outcomes of this trial, which of the following effects is of greatest concern in this type of study design? A. Carryover effects. B. Cohort effects. C. Confounding effects. D. Run-in effects.

Answer A: Carryover effects. (In a crossover trial, the largest concern is that the effects of active therapy will carry over into the control group results; an example of this might be when interferon is randomized to be administered before placebo (Answer A is correct). Potential confounding and cohort effects would be minimized by randomization and the crossover design (Answer B and Answer C are incorrect). A run-in effect would require a placebo run-in period before randomization; this trial does not include such a period (Answer D is incorrect).)

A 31-year-old woman with no significant medical history presents to the ED with upper quadrant abdominal pain. Physical examination is suggestive of cholecystitis. Ultrasonography confirms the diagnosis. The patient is in mild distress with no alteration in vital signs. Which one of the following is best for this patient? A. Cefazolin 1 g intravenously every 8 hours. B. Imipenem 1 g intravenously every 8 hours. C. Gentamicin 5 mg/kg intravenously daily. D. Vancomycin 1 g intravenously every 12 hours.

Answer A: Cefazolin 1 g intravenously every 8 hours. (Uncomplicated cholecystitis is typically caused by gram-positive or gram-negative aerobic pathogens. Cefazolin provides coverage that is more focused than the alternative of imipenem. Anaerobic coverage is not necessary unless a biliary-enteric anastomosis is present (Answer A is correct; Answer B is incorrect). Gentamicin and vancomycin are lacking in coverage of the possible pathogen mix (Answer C and Answer D are incorrect).)

A 63-year-old man with a history of alcohol abuse and ascites is admitted to the hospital with possible spontaneous bacterial peritonitis. He has no active bleeding. Analysis of his ascitic fluid reveals protein 1.2 g/dL, and elevated WBC, and several bacteria. Which one of the following is best for this patient? A. Ceftriaxone 1 g intravenously daily. B. Piperacillin/tazobactam 4.5 g intravenously every 6 hours. C. Ampicillin/sulbactam 3 g intravenously every 6 hours. D. Ciprofloxacin 400 mg intravenously every 8 hours and metronidazole 500 mg intravenously every 8 hours.

Answer A: Ceftriaxone 1 g intravenously daily. (Patients with ascites are typically infected with gram-negative enteric pathogens or streptococci. Ceftriaxone covers these pathogens well without unnecessary broader coverage (Answer A is correct). Anaerobic activity is not required empirically, making piperacillin/tazobactam and ciprofloxacin and metronidazole unnecessarily broad (Answer B and Answer D are incorrect). Ampicillin/sulbactam, although a reasonable option for coverage of some gram-negative pathogens, is particularly poor in most institutions against Escherichia coli, the primary pathogen in spontaneous bacterial peritonitis (Answer C is incorrect).)

A 34-year-old woman has a history of migraine headaches. She was recently prescribed eletriptan 20 mg to take at the onset of a migraine episode. As a business executive, she is frequently required to travel out of the country. She has noticed that on these trips, eletriptan 20 mg does not last long enough for the migraine to be fully aborted. Which one of the following would be the best choice for this patient on long airline flights? A. Almotriptan. B. Frovatriptan. C. Rizatriptan. D. Sumatriptan.

Answer B: Frovatriptan. (Given that frovatriptan has the longest half-life (26 hours) of all the triptans, this agent would be ideal for her longer flights (Answer B is correct). The half-life of rizatriptan is 2—3 hours (Answer C is incorrect). The half-life of sumatriptan is 2.5 hours (Answer D is incorrect). The half-life of almotriptan is 3—4 hours (Answer A is incorrect).)

A 44-year-old man is hospitalized with moderate to severe Crohn disease (CD) in the colon and terminal ileum. His disease has not responded to oral prednisone 40 mg/day for 5 days or intravenous methylprednisolone 60 mg every 12 hours for 5 days. The patient has no known allergies or significant medical history. He continues to have a high temperature and severe symptoms. He has normal kidney and liver function and is without infection. Which one of the following is the best therapy to initiate in this patient? A. Certolizumab subcutaneously. B. Metronidazole orally. C. Hydrocortisone intravenously. D. Azathioprine orally.

Answer A: Certolizumab. (Moderate to severe CD often responds to corticosteroid therapy. In this cæse, the patient's disease has not responded to oral and intravenous corticosteroids, so administering another intravenous corticosteroid would not be expected to induce remission (Answer C is incorrect). Patients whose disease does not respond to corticosteroids or patients who are intolerant of corticosteroids should receive infliximab, adalimumab, or certolizumab unless they have a contraindication (Answer A is correct). Metronidazole is useful in mild to moderate disease or perianal disease (Answer B is incorrect). Azathioprine is not effective for induction therapy because of its long onset of action; instead, it is used for maintenance of remission (Answer D is incorrect).)

A 38-year-old woman has stage 5 CKD and is on continuous ambulatory peritoneal dialysis. Her prescription consists of four exchanges of 2 L each with a 6-hour dwell. She uses 1.5% dextrose peritoneal dialysis solution during the day, with 4.25% dextrose peritoneal dialysis solution for her nighttime dwell. She produces about 2 cups of urine per day. She is achieving adequate ultrafiltration and dialysis; however, her morning fasting glucose is 225 mg/dL. On repeat testing, her glucose concentration remains elevated. Which one of the following is best to recommend for this patient? A. Change her evening dialysis solution to icodextrin. B. Add regular insulin intraperitoneally to evening exchange. C. No change is necessary because this effect is transient. D. Add glipizide 2.5 mg orally once daily before breakfast.

Answer A: Change her evening dialysis solution to icodextrin. (The morning hyperglycemia may be related to the use of 4.25% dextrose peritoneal dialysis solution. Switching the solution to a nonglucose polymer such as icodextrin may eliminate the morning hyperglycemia (Answer A is correct). Administering insulin intraperitoneally results in insulin binding to polyvinyl chloride bags and tubing, requiring many adjustments in insulin dosage (Answer B is incorrect). Hyperglycemia is not commonly a transient effect and requires therapy modification (Answer C is incorrect). Glipizide should be considered only if hyperglycemia is significant and occurs throughout the day (Answer D is incorrect).)

A trial compares two groups of patients. One group receives a newly developed corticosteroid inhaler; the second group receives an existing corticosteroid inhaler. Which one of the following statistical tests would best compare the proportion of patients in each group who had an emergency department visit during the trial? A. Chi-square. B. Mann-Whitney U. C. Student t-test. D. Wilcoxon signed rank.

Answer A: Chi-square. (Chi-square is the appropriate statistical test to compare proportions or percentages of two independent groups when the outcome is measured on a nominal level (Answer A is correct). The Mann-Whitney U and Wilcoxon signed rank are tests used to compare two groups measured on an ordinal level (Answer B and Answer D are incorrect). The Student t-test is used to compare the means of two groups measured on a continuous level (Answer C is incorrect).)

A 34-year-old man receives a diagnosis of pulmonary Mycobacterium avium complex (MAC) infection. His medical history includes HIV infection, and he is antiretroviral-naive. The patient has no known drug allergies. Which one of the following is best to recommend for this patient? A. Clarithromycin plus ethambutol. B. Azithromycin plus rifampin. C. Ciprofloxacin plus rifabutin. D. Azithromycin plus ciprofloxacin.

Answer A: Clarithromycin plus ethambutol. (A macrolide plus ethambutol is the regimen of choice for MAC infections (Answer A is correct). Adding a rifamycin as a third agent (e.g., rifabutin because of a drug-drug interaction) can be considered, but ethambutol would be included in the regimen (Answer B is incorrect). Fluoroquinolones are considered alternative therapies and are not preferred to initial therapy with a macrolide plus ethambutol (Answer C and Answer D are incorrect).)

Which one of the following information resources would most efficiently provide a comparison of proton pump inhibitors, including dosage form, pharmacology, dosage, and adverse events? A. Clinical Pharmacology. B. Epocrates Online. C. Lexicomp Online. D. Micromedex.

Answer A: Clinical Pharmacology. (Clinical Pharmacology provides a tool by which various aspects of similar drugs can be compared (Answer A is correct). Epocrates Online and Lexicomp Online do not contain features allowing a user to compare items (Answer B and Answer C are incorrect). Micromedex allows a comparison of entire monographs, but not of individually chosen topics within the monograph. Micromedex allows a side-by-side comparison, but for only two medications at a time (not necessarily an entire class), so it is not as efficient as Clinical Pharmacology (Answer D is incorrect).)

You are contracted as a consultant for a group that is going to build an online health information Web site for consumers and professionals. The group recognizes the importance of providing a mechanism for visitors to quickly determine the quality of the information the site provides. Because of time and financial pressures, the Web site must be posted within 4 weeks. As the consultant, you are asked to recommend a quality assessment tool that will meet the group's needs. Which one of the following categories of quality assessment tools is the best option for this situation? A. Codes of conduct. B. Seals of quality. C. Filters. D. Third-party certification.

Answer A: Codes of conduct. (Quality assessment tools can be divided into four categories according to the method they employ to assess the quality of online health information. Although all rely on a set of assessment criteria, they differ in their application of these criteria. Codes of conduct are recommendations for Web site developers to follow when developing online health information (Answer A is correct). Developers who follow a code's recommendations can display the logo for the code on their site. Although this method places the burden of site assessment on the Web site developer, an external group is not required for assessment. Use of codes of conduct will allow your group to perform an internal quality assessment using established criteria, and visitors to the new Web site will see a logo indicating a quality assessment has occurred. Seals of quality, filters, and third-party certification are less ideal choices because they use an external evaluation process. Seals of quality are closely related to codes of conduct; however, a site displaying a seal of quality has applied for, and received, approval from an external organization to display its seal, indicating adherence to a set of quality criteria (Answer B is incorrect). Filters and third-party certification provide a higher level of quality. Filtering services allow users to search pre-evaluated sites that have been determined to meet existing quality criteria (Answer C is incorrect). Third-party certification services are similar to seals of quality; however, third-party certification services actively review sites displaying their seal (Answer D is incorrect).)

A 61-year-old man with extensive-stage small cell lung cancer has received one course of chemotherapy with carboplatin and etoposide. He develops a significant amount of nausea and vomiting during his first course of chemotherapy, together with febrile neutropenia and alopecia. Which one of the following is the best option for this patient's next cycle of chemotherapy? A. Continue carboplatin and etoposide with dose reductions. B. Continue carboplatin and etoposide at full dose; add a hematopoietic growth factor. C. Change the regimen to etoposide and cisplatin. D. Change the regimen to cyclophosphamide, doxorubicin, and vincristine.

Answer A: Continue carboplatin and etoposide with dose reductions. (The best course of action for this patient's second course of chemotherapy is to continue the same regimen with reduced doses (Answer A is correct). The goal is palliation rather than cure; therefore, adding a hematopoietic growth factor is not indicated, nor will it affect treatment outcomes (Answer B in incorrect). Treatment with the etoposide and cisplatin regimen is more emetogenic than the current regimen and is therefore not the best approach at this time (Answer C is incorrect). The three-drug regimen of cyclophosphamide, doxorubicin, and vincristine may be associated with more toxicities than the original regimen of carboplatin plus etoposide; therefore, switching to this regimen may be more problematic and is not optimal (Answer D is incorrect).)

A 64-year-old African American man received a deceased donor liver transplant 14 days ago secondary to NASH (nonalcoholic steatohepatitis) cirrhosis. His medical history is significant for diabetes, hypertension, hepatic encephalopathy, and gastric varices. Today, he presents to the endocrine clinic for uncontrolled blood sugar levels (home readings 180—220 mg/dL) after transplantation. His current drugs include aspirin 325 mg daily, mycophenolate mofetil 1000 mg twice daily, prednisone 20 mg daily, esomeprazole 40 mg twice daily, oxycodone 5—10 mg every 6 hours as needed for pain, docusate sodium 1 tablet twice daily, simvastatin 40 mg at bedtime, valganciclovir 450 mg twice weekly (on Mondays and Thursdays), nystatin swish and swallow 5 mL four times daily, sulfamethoxazole/trimethoprim single strength I tablet daily, insulin glargine 20 units twice daily, and regular insulin per sliding scale. Which one of the following is the most likely etiology for this patient's uncontrolled blood sugar levels? A. Corticosteroid-induced gluconeogenesis. B. CNI-induced beta-cell toxicity. C. Systemic infection leading to increased cortisol release. D. Mycophenolate mofetil—associated pancreatitis.

Answer A: Corticosteroid-induced gluconeogenesis. (Hyperglycemia and posttransplant diabetes are a source of frustration and complications after transplantation. Immunosuppressive drugs that likely contribute to uncontrolled blood sugar levels include corticosteroids and CNIs. Hyperglycemia is a known adverse effect of the CNIs and is more common with tacrolimus than with cyclosporine, secondary to direct beta-cell toxicity in the pancreas. Corticosteroid exposure induces gluconeogenesis, resulting in abnormal blood glucose regulation (Answer A is correct). Routine practice includes initiating blood sugar control therapies rather than immunosuppressive dose modification during the early posttransplant period. As months and years elapse, transplant recipients with continued uncontrolled blood sugar levels despite aggressive pharmacotherapy may consider conversion to another agent that does not produce hyperglycemia such as an mTOR (mammalian target of rapamycin) inhibitor or belatacept. Neither systemic infection leading to increased cortisol release nor mycophenolate mofetil—associated pancreatitis is associated with uncontrolled blood sugar levels (Answer C and Answer D are incorrect). Although CNIs can produce hyperglycemia, the patient's current regimen does not include a CNI (Answer B is incorrect).)

Your supervising director wishes to increase the number of clinical pharmacists engaged in anticoagulation and congestive heart failure services. He asks you to prepare a presentation outlining the relative costs associated with each service. Which one of the following economic evaluation methods is best to use when quantifying the value of a clinical pharmacy service? A. Cost-benefit analysis. B. Cost-effectiveness analysis. C. Cost-minimization analysis. D. Cost-utility analysis.

Answer A: Cost-benefit analysis. (A cost-benefit analysis is the best economic evaluation method to compare two or more programs when benefits should be translated to a dollar value. When quantifying the value of each pharmacy service, the cost of implementing and managing the program (e.g., the pharmacist's salary, laboratory tests) and the benefit of the program (e.g., decreased drug costs, decreased patient lengths of stay) can both be translated to dollar values (Answer A is correct). A cost-effectiveness analysis is the best economic evaluation method to apply when two or more treatment alternatives have different efficacy and safety profiles (Answer B is incorrect). Cost-minimization analysis should be used to compare agents in the same therapeutic class with documented equivalence in safety and efficacy (Answer C is incorrect). Cost-utility analyses can compare cost, quality, and quantity of patient-years and are best applied when evaluating life-extending treatment alternatives with serious adverse effects (i.e., chemotherapy) or when evaluating therapy options that reduce morbidity but not mortality (i.e., pain medications) (Answer D is incorrect).)

A 46-year-old man who is paraplegic is admitted to the hospital with fevers, chills, and increased WBCs. His medical history is significant for urinary tract infections and depression. He currently takes baclofen and fluoxetine and has no known drug allergies. His urine and blood cultures are growing Enterococcus spp. that are resistant to vancomycin and ampicillin and susceptible to nitrofurantoin, doxycycline, linezolid, and daptomycin. Which one of the following is the best antimicrobial therapy for this patient? A. Daptomycin. B. Doxycycline. C. Linezolid. D. Nitrofurantoin.

Answer A: Daptomycin. (Daptomycin is preferred because the patient has bacteremia and shows signs of urosepsis, including fever and elevated WBCs (Answer A is correct). Nitrofurantoin can be used for cystitis, and doxycycline should be reserved for cystitis and possibly upper urinary tract infection without urosepsis (Answer B and Answer D are incorrect). Linezolid should be avoided because the patient takes fluoxetine, and concurrent use may predispose patients to serotonergic syndrome (Answer C is incorrect).)

A 42-year-old woman (weight 60 kg [132 lb]) is admitted to the intensive care unit (ICU) after a motor vehicle crash. She develops acute renal failure secondary to hypovolemic shock and is currently receiving continuous venovenous hemodialysis (CVVHD). The patient also has a small bowel obstruction requiring parenteral nutrition (PN). The dialysis solution she receives is a lactate-based solution with dextrose 2% running at a rate of 1 L/hour. Trisodium citrate is being used for regional anticoagulation. The CRRT machine does not use a blood warmer. For optimal nutrition support, which of the following would be the best adjustment to the dextrose in this patient's PN regimen? A. Decrease by 200 g/day. B. Decrease by 480 g/day. C. Increase by 200 g/day. D. Increase by 480 g/day.

Answer A: Decrease by 200 g/day. (This patient is receiving CVVHD using a dialysis solution containing dextrose 2% at a rate of 1 L/hour, which will supply 480 g of dextrose per day. Because an estimated 35%-45% of dextrose in the CRRT dialysate is absorbed, 168-216 g of dextrose per day would be absorbed from the patient's dialysate. This dexstrose delivery should be accounted for in her nutrition regimen, necessitating a decrease in the dextrose administered through modalities other than CRRT. Decreasing the dextrose in the PN formulation by about 200 g/day is therefore the best option (Answer A is correct). Decreasing the dextrose in the PN by 480 g/day is incorrect because it represents 100% absorption of the dextrose being supplied through CRRT, although only 35%-45% is actually absorbed. Therefore, decreasing the dextrose delivery by 480 g/day would result in a caloric deficit (Answer B is incorrect). Increasing dextrose by either 200 g/day or 480 g/day is incorrect because dextrose in the PN formulation should be decreased, not increased, to avoid overfeeding and its inherent risks (Answer C and D are incorrect).)

A 21-year-old woman with GAD returns to your clinic for follow-up. Twenty weeks ago, her primary care physician initiated treatment with mirtazapine 45 mg at bedtime; 13 weeks ago, her regimen was changed to escitalopram 10 mg/day. Although she had a partial response to mirtazapine, she also gained an unwanted 10 lb in 6 weeks. Since the change to escitalopram 10 mg/day, her dose was titrated to the current dose of 40 mg/day and has been at this dose for I week. She notes significant improvement in her symptoms of irritability and worrying. Her baseline Hamilton Anxiety Rating Scale (HAM-A) score was 29, and today, her score is 9. She has experienced decreased libido since escitalopram was initiated. Her chart shows that she struggles with bulimia. She denies any problems with substance or alcohol abuse. She is not currently taking any other medications. Which one of the following is best to recommend for management of this patient's symptoms? A. Decrease escitalopram to 20 mg/day. B. Switch back to mirtazapine 45 mg/day. C. Add alprazolam 0.5 mg four times/day as needed. D. Continue current therapy.

Answer A: Decrease escitalopram to 20 mg/day. (The patient has had a good response to escitalopram, as evidenced by the decrease in her HAM-A score. At the current dose of 40 mg/day, she is experiencing decreased libido, which may improve with a decrease in dosage; therefore, a trial at a lower dose is warranted (Answer A is correct). Switching back to mirtazapine is not the best choice because she had achieved only a partial response to mirtazapine after 20 weeks of therapy and experienced unwanted weight gain. Furthermore, no substantial evidence from controlled studies supports the use of mirtazapine as a first- or second-line therapy (Answer B is incorrect). The patient has experienced a significant reduction in symptoms while taking escitalopram, so adding a benzodiazepine is not warranted, nor would it address her decreased libido (Answer C is incorrect). The patient is experiencing adverse sexual effects at this time, which should be acknowledged and managed appropriately (Answer D is incorrect).)

A 60-year-old African American man presents for follow-up evaluation of stage IV prostate cancer metastatic to the ribs, femur, and lumbar spine. He is experiencing severe bone pain and intermittent paresthesias in the lower limbs caused by spinal cord compression. Androgen deprivation therapy (ADT) is to be initiated. The patient does not wish to have an orchiectomy. Which one of the following is the best choice for ADT in this patient? A. Degarelix alone. B. Flutamide and leuprolide. C. Abiraterone plus prednisone. D. Goserelin alone.

Answer A: Degarelix alone. (Optimal ADT, as defined in the 2013 National Comprehensive Cancer Network guidelines, includes either medical castration with a luteinizing hormone—releasing hormone agonist or antagonist or surgical castration by bilateral orchiectomy. The two methods are considered equally effective. Combined androgen blockade, such as leuprolide combined with the use of flutamide, provides no proven benefit over the use of castration alone in patients with metastatic disease (Answer B is incorrect). Because the disease is not yet proven castrate resistant, the choice of abiraterone plus prednisone is incorrect (Answer C is incorrect). Degarelix is preferred to goserelin in this patient because of his severe bone pain and spinal cord compression, so there will be no chance of tumor flare (Answer A is correct; Answer D is incorrect).)

A 66-year-old woman with a medical history of HTN, stable angina, and T2DM presents to the clinic for follow-up. She had an MI 2 years ago that was treated successfully with one bare metal Stent IBMS) placed in the circumflex artery. At follow-up, she states that her exertional chest pain episodes are controlled and that they rarely occur. She has been able to resume all of her daily activities and is adherent to her diet and exercise regimen. Current drugs include aspirin 81 mg daily, clopidogrel 75 mg daily, atenolol 50 mg daily, amlodipine 5 mg daily, metformin 850 mg twice daily, and sublingual nitroglycerin as needed for chest pain. Today. BP is 134/84 mm Hg, and her HR is 58 beats/minute. Laboratory values include serum potassium 4.0 mEq/L, SCr 1.0 mg/dL, and AIC 7.1%. Which one of the following changes is to make to this patient's antiplatelet regimen? A. Discontinue clopidogrel. B. Increase aspirin to 325 mg daily. C. clopidogrel to 150 mg daily. D. Add cilostazol 100 mg twice daily.

Answer A: Discontinue clopidogrel. (Because this patient received a BMS, the recommended duration of dual therapy is for at least 1 month, and ideally for up to 1 year. Unlike with drug-eluting stents, the risk of late stent thrombosis is very uncommon With BMSs because they typically become re-endothelialized in about 2-4 weeks. Therefore, this patient has no reason to continue taking clopidogrel, and it should be discontinued (Answer A is correct and Answer C is incorrect). The use of aspirin doses greater than 81 mg has not shown improved efficacy (Answer B is incorrect). Dual antiplatelet therapy 1 year after successful BMS implantation not necessary (Answer D is incorrect).)

A 35-year-old woman has had rheumatoid arthritis (RA) for 10 years; her disease has been in remission for 6 months. She currently takes methotrexate 7.5 mg weekly and etanercept 50 mg weekly. The patient now states that she wants to become pregnant and asks about continuing her RA treatment during pregnancy. Which one of the following is the best recommendation for this patient? A. Discontinue methotrexate and continue etanercept. B. Discontinue methotrexate and etanercept and begin leflunomide. C. Discontinue etanercept and continue low-dose methotrexate. D. Discontinue all drugs.

Answer A: Discontinue methotrexate and continue etanercept. (Patients with long-standing RA are likely to need maintenance therapy with disease-modifying antirheumatic drugs or biologics, even after achieving a short-term remission. Thus, discontinuing methotrexate, a known teratogen, to allow a possible pregnancy is reasonable while continuing etanercept, which largely appears safe in pregnancy (Answer A is correct; Answer C is incorrect). It would inappropriate to discontinue all drug treatment at once because the patient would likely suffer a flare of her RA symptoms (Answer D is incorrect). Substituting leflunomide would be inappropriate because it is also teratogenic (Answer B is incorrect).)

A 66-year-old carpenter presents to the emergency department with a puncture wound. When asked about previous tetanus vaccinations, he is uncertain whether he has ever received this particular vaccination. Which one of the following is most appropriate for this patient? A. Administer 0.5 mL of tetanus-diphtheria toxoid (Td) booster vaccine. B. Administer 0.5 mL of Tdap vaccine now and then complete the primary series with Td. C. No Td or Tdap vaccination is currently necessary. D. Perform laboratory tests to determine the need for Td vaccination.

Answer B: Administer 0.5 mL of Tdap vaccine now and then complete the primary series with Td. (In patients with an uncertain history of a complete primary vaccination series, a primary series should be administered. A primary series includes a one-time dose of Tdap, which should be administered first in the series, followed by two doses of Td vaccine, with the first Td vaccine at least 4 weeks after the Tdap vaccine and the third dose 6—12 months after the first Td vaccine (Answer B is correct). Therefore, vaccination is necessary for this patient (Answer C is incorrect). No laboratory testing is required before administering the Td vaccine (Answer D is incorrect). This patient should have a complete series of the Tdap/Td vaccine (Answer A is incorrect).)

A 43-year-old man (height 5'9", weight 102 kg [225 lb]) is referred to the clinic by his primary care physician for treatment of depression. His target symptoms for therapy are moderately depressed mood, difficulty falling asleep, fatigue, and problems focusing. He denies excessive guilt, changes in psychomotor activity, and thoughts of self-harm. His Quick Inventory of Depressive Symptomatology—Self Report (QIDS-SR) score is 13. His medical history is significant for type 2 diabetes mellitus, peripheral neuropathy, and coronary artery disease. His current drugs are simvastatin 40 mg at bedtime, lisinopril 10 mg/day, metoprolol 12.5 mg twice daily, and capsaicin cream topically to his feet twice daily. Current vital signs include blood pressure 128/78 mm Hg and heart rate 72 beats/minute. He has never been treated for depression. Which one of the following would be the best initial antidepressant for this patient? A. Duloxetine. B. Mirtazapine. C. Phenelzine. D. Paroxetine.

Answer A: Duloxetine. (Duloxetine is the most appropriate choice because it has minimal drug-drug interactions with the patient's current regimen, and there are few concerns with worsening his comorbid medical conditions. In addition, it should help with both the patient's depressive episode and his peripheral neuropathy (Answer A is correct). Mirtazapine might be beneficial for both conditions as well, but because of the patient's weight issues and related medical problems, it would not be the best choice (Answer B is incorrect). Phenelzine is not recommended as first-line therapy because of significant drug-drug and drug-food interactions (Answer C is incorrect). Paroxetine potently inhibits CYP2D6 and therefore blocks the metabolism of metoprolol. In addition, paroxetine would likely not be as beneficial for this patient's neuropathy (Answer D is incorrect).)

You wish to find information about panic attacks with triphasic contraceptive agents. Which one of the following resources, listed in search order of first to last, would be best to find this information? A. Facts & Comparisons eAnswers; Meyler's Side Effects of Drugs; PubMed. B. Embase; prescribing information. C. MEDLINE; Drug Information Handbook; American Hospital Formulary Service (AHFS) Drug Information. D. AHFS Drug Information; Drug Information Handbook.

Answer A: Facts & Comparisons eAnswers; Meyler's Side Effects of Drugs; PubMed. (Tertiary resources should be searched first (AHFS, Facts & Comparisons eAnswers, Meyler's Side Effects of Drugs, prescribing information, Drug Information Handbook, Lexicomp Online), followed by a secondary database search (MEDLINE, PubMed, International Pharmaceutical Abstracts, Embase) for primary information (Answer A is correct; Answer B and Answer C are incorrect). Searching only tertiary sources without a subsequent secondary database search for primary information is likely to result in incomplete information (Answer D is incorrect).)

A 54-year-old man with a history of alcoholism and peptic ulcer disease presents to the emergency department (ED) with severe melena suggestive of gastrointestinal bleeding. On admission, his blood pressure is 90/40 mm Hg and pulse is 120 beats/minute. Laboratory values include Hgb 10.2 g/dL and Hct 31%. While receiving an infusion of 0.9% sodium chloride (NaCl), the patient complains of dizziness and then becomes unresponsive. No pulse or blood pressure is detected. Telemetry shows sinus rhythm at a rate of 72 beats/minute. Which of the following is the best initial intervention for this patient? A. Five cycles ofCPR. B. Intravenous epinephrine 1 mg every 3—5 minutes. C. One shock of 200 joules with a biphasic defibrillator. D. Atropine 1 mg every 3—5 minutes with maximal total dose of 3 mg.

Answer A: Five cycles of CPR. (This patient has pulseless electrical activity (PEA), which is a cardiac arrest rhythm. According to the 2010 AHA guidelines for CPR and ECC, for the treatment of PEA, five cycles (about 2 minutes) of CPR should be delivered before administering drug therapy (Answer A is correct). A vasopressor may be administered if PEA persists despite initial attempts at CPR. In the treatment algorithm for PEA, either epinephrine 1 mg can be administered intravenously every 3—5 minutes or one dose of vasopressin 40 units can be given intravenously to replace the first or second dose of epinephrine (Answer B is incorrect). In the most recent guideline update, atropine has been removed from the cardiac arrest algorithm because available evidence suggests that routine use during PEA or asystole is unlikely to have a therapeutic benefit (Answer D is incorrect). Defibrillation has no role in the treatment of PEA because it is not considered a shockable rhythm (Answer C is incorrect).)

The director of nursing for the emergency department is concerned about the number of mixed prescription medication overdoses coming into the department. The director would like the emergency department pharmacist to do a three-part continuing education program for the nurses and paramedics. The first part of the series should discuss recent changes in the general management of poisonings and overdoses as well as the differences in treating patients with gastric lavage, activated charcoal, and/or hemoperfusion. Which one of the following resources would provide the best and most current information for a basic overview of general poisonings? A. Goldfrank 's Manual of Toxicologic Emergencies. B. Casarett Doull's Toxicology: The Basic Science of Poisons. C. IDENTIDEX. D. Poisindex.

Answer A: Goldfrank's Manual of Toxicologic Emergencies. (Goldfrank 's Manual of Toxicologic Emergencies provides excellent review information on acute poisoning management, including mixed overdose situations. It also has concise reviews on managing patients with gastric lavage, activated charcoal, and hemoperfusion (Answer A is correct). Casarett & Doull's Toxicology textbook focuses on more chronic aspects of poisonings; this book may be an alternative for other aspects of the three-part series, but it is not necessarily useful for this first topic (Answer B is incorrect). IDENTIDEX may provide help in identifying the tablets, if available, in an overdose situation, but it does not provide information that could be used to create an education program (Answer C is incorrect). Poisindex is a great source for the immediate overdose situation; however, it may not provide the information in a concise review format for the basis of an education program (Answer D is incorrect). Poisindex does provide management information on overdose both in general and for specific overdoses and would be a good alternative to Goldfrank's Manual to supplement the information or find in-depth information on a particular drug overdose.)

In designing the layout of your new community pharmacy, you include a consultation room for providing immunizations. Which one of the following is most important to consider in designing the physical workspace for providing immunization services? A. Cost of vaccine. B. Auditory privacy. C. Location to pharmacy main entrance. D. Access to refrigerator.

Answer B: Auditory privacy. (Although a refrigerator to hold vaccinations must be close within the practice site, it does not need to be in the actual immunization consult area (Answer D is incorrect). The cost of the vaccine is not a consideration when designing the physical space to provide immunizations (Answer A is incorrect). Location to the pharmacy entrance may be a consideration when designing a workplace; however, a location that provides auditory privacy is most important (Answer C is incorrect). The Health Insurance Portability and Accountability Act (HIPAA) requires auditory privacy for patient consultation (Answer B is correct).)

A 65-year-old man (weight 60 kg [132 lb]) has stage 3 chronic kidney disease (CKD) secondary to long-standing hypertension. His laboratory values today are Hgb 9.8 g/dL (increased from 10.4 g/dL 3 months ago), ferritin 65 ng/mL, and transferrin saturation (TSAT) 12%. He has no symptoms at this time. Which one of the following would be best to recommend for this patient? A. Initiate ferrous sulfate 325 mg orally three times/day. B. Initiate iron sucrose 200 mg intravenously for five doses in a M-day period. C. Initiate epoetin alfa 3000 units subcutaneously three times/week. D. Initiate darbepoetin subcutaneously 25 mcg/week.

Answer A: Initiate ferrous sulfate 325 mg orally three times/day. (Because this patient is asymptomatic and mildly iron deficient, with a TSAT of less than 20% and serum ferritin less than 100 ng/mL, and because he is not on dialysis, oral iron replacement is the best choice (Answer A is correct). Intravenous iron should be reserved for patients not responding to oral therapy or for those on dialysis (Answer B is incorrect). In patients with CKD who are not on hemodialysis, it is recommended that erythropoiesis-stimulating agents (ESAs) not be initiated if the Hgb is 10 g/dL or greater. The decision to initiate therapy for patients with an Hgb less than 10 g/dL is individualized according to the rate of Hgb decline, presence of symptoms, and prior response to iron supplementation. Although this patient's Hgb recently fell below 10 g/dL, he is asymptomatic, and iron supplementation may correct the anemia. In addition, ESAs should not be initiated until iron stores have been adequately repleted (Answer C and Answer D are incorrect).)

A man has a new diagnosis of Burkitt lymphoma. His laboratory values include SCr 0.9 mg/dL, total bilirubin 1.4 mg/dL, and LDH 2100 U/L. His medical history is significant for HIV infection, and his current CD4' count is 44 cells/mm^3. Which one of the following is the best treatment option to recommend for this patient? A. Initiate highly active antiretroviral treatment and chemotherapy. B. Start chemotherapy and initiate highly active antiretroviral treatment once remission is achieved. C. Initiate highly active antiretroviral treatment and start chemotherapy once the CD4+ count normalizes. D. Proceed to autologous transplant.

Answer A: Initiate highly active antiretroviral treatment and chemotherapy. (This patient should be treated with highly active antiretroviral therapy and chemotherapy simultaneously (Answer A is correct). Clinical trials have shown that patients with AIDS can be safely and effectively treated with chemotherapy and antiretroviral drugs simultaneously and have improved responses compared with chemotherapy alone. On the basis of this same evidence, starting chemotherapy and initiating highly active antiretroviral treatment once remission is achieved is incorrect (Answer B is incorrect). The patient has a highly aggressive lymphoma and requires treatment urgently; thus, highly active antiretroviral therapy alone is inappropriate (Answer C is incorrect). An autologous transplant may be an option once the disease responds to treatment, but combination chemotherapy with antiretroviral therapy must be initiated before the patient is considered for a transplant (Answer D is incorrect).)

A randomized controlled trial is conducted in three groups of adults to assess how treatment for multiple sclerosis affects patient functioning. Investigators assess global functioning with an ordered, Likert-type scale. Which one of the following statistical tests would be most appropriate to assess differences in functioning between the groups? A. Kruskal-Wallis. B. Multiple ANOVA. C. ANOVA. D. Analysis of covariance (ANCOVA).

Answer A: Kruskal-Wallis. (The Kruskal-Wallis test is the nonparametric analog to the ANOVA test and is used when the outcome data are ordinal (Answer A is correct). The multiple ANOVA test is used for normally distributed data, taken from more than two groups, with multiple outcomes (Answer B is incorrect). The ANOVA test is used for normally distributed data taken from more than two groups (Answer C is incorrect). The ANCOVA procedure is used when data are normally distributed and include multiple independent variables, some of which are measured on a continuous scale (Answer D is incorrect).)

A 45-year-old woman (height 64 inches, weight 88.6 kg) who has had T2DM for 5 years presents for her annual physical examination. Her vital signs and laboratory show the following: blood pressure 134/79 mm Hg, heart rate 84 beats/minute, and microalbumin/creatinine ratio 11.1. Fasting laboratory results reveal TC 184 mg/dL, TG 210 mg/dL, HDL-C 45 mg/dL, LDL-C 97 mg/dL, non-HDL-C 139 mg/dL, AIC 9.8%, scr 1.2 mg/dL, and K 4.5 mEq/L. She has not smoked for 8 years. She currently takes metformin 1000 mg twice daily, atorvastatin 80 mg/day, and lisinopril 40 mg/day. Which one of the following would best improve this patient's AIC and cardiovascular (C V) risk factors? A. Lose 5-9 kg with lifestyle modification. B. Start a fish oil supplement. C. Change atorvastatin to pravastatin 40 mg/day. D. Start aspirin 325 mg/day.

Answer A: Lose 5-9 kg with lifestyle modification. (Lifestyle intervention resulting in a weight loss of 5%—9% has shown a significant reduction in AIC and several CV disease risk factors (Answer A is correct). Fish oil supplements have not been shown to improve AIC (Answer B is incorrect). Changing to pravastatin 40 mg daily is a less potent therapy and it will not Improve her CV risk factors or AIC (Answer C is incorrect). Although aspirin therapy could be considered to reduce her CV risk, it will not improve her AIC. The American Diabetes Association (ADA) states that aspirin may be used as primary prevention in those at increased CV disease risk (10%), but it is not recommended in women younger than 60 with low CV risk (less than 5%). The U.S. Preventive Services Task Force states that aspirin use is not recommended in women younger than 60 whose 10-year CV disease risk is less than 5%. Using the Western States Stroke Consortium tool, this patient's 10-year risk of stroke is 2.8%. In addition, the recommended dose of aspirin is 75—162 mg daily, not 325 mg as listed here (Answer D is incorrect).)

A woman with a history of acute myeloblastic leukemia seeks evaluation 89 days after an allogeneic hematopoietic stem cell transplant (HSCT) because she has bloody diarrhea, and desquamating rash covers 100% of her body. A confirmed biopsy reveals grade III graft-vs.-host disease (GVHD). Which one of the following is the most appropriate initial treatment option for this patient? A. Methylprednisolone 2 mg/kg/day. B. Mycophenolate 1000 mg two times/day. C. Antithymocyte globulin 2.5 mg/kg daily for 3 days. D. Infliximab 10 mg/kg weekly for 4 weeks.

Answer A: Methylprednisolone 2 mg/kg/day. (Corticosteroids are the mainstay for the treatment of acute GVHD, with methylprednisolone 2 mg/kg/day being the recommended first-line choice for grades III and I V; Level 1 A Evidence (Answer A is correct). Infliximab is incorrect for this patient now but is appropriate as second-line therapy because tumor necrosis factor inhibitors induce response in patients who have acute GVHD refractory to corticosteroids (Answer D is incorrect). Mycophenolate is also a second-line therapy option for steroid-resistant GVHD (Answer B is incorrect). Antithymocyte globulin is sometimes used for prophylaxis of GVHD after HSCT or bone marrow transplantation but is not a first-line therapy for acute grade III GVHD (Answer C is incorrect).)

The following data were obtained from Kaplan-Meier plots of survival for treatments I and 0. The y-axis reflects the percentage of individuals in the study who survive (or who do not have the event of interest) over a specified time (x-axis). (Couldn't put graph) ------------------------------------------------------------ Log Rank Test for Equality of Survivor Functions ---------------------------------------- Treatment || Events Observed || Events Expected ---------------------------------------- Treatment 1 || 7 || 5.23 ---------------------------------------- Treatment 0 || 5 || 6.77 ---------------------------------------- Total || 12 || 12.00 ----------------------------------------------------------- Which one of the following best explains what the log-rank test assesses? A. Overall differences between the Kaplan-Meier curves for the treatment groups. B. Differences between groups, assuming that differences early in the study are more important than differences later in the study. C. Differences between groups, assuming that differences late in the study are more important than differences early in the study. D. Differences between groups at the median survival time.

Answer A: Overall differences between the Kaplan-Meier curves for the treatment groups. (The log-rank test assesses overall differences between the Kaplan-Meier curves for the treatment groups (Answer A is correct). The log-rank test does not assume that differences early in the study are more important than differences later in the study (Answer B is incorrect). (The Peto test is used to incorporate larger weights for events early in the study.) Because the log-rank test looks at all time points equally, assuming that differences late in the study are more important than differences early in the study would instead require a test that weighted late events more than events at other times (Answer C is incorrect). Finally, the log-rank test looks at all points along the curve and does not test only for differences between groups at the median survival time (Answer D is incorrect).)

You are assigned to a workgroup in a managed care organization to evaluate barriers to improving the Healthcare Effectiveness Data and Information Set (HEDIS) measure "Persistence of ß-Blocker Treatment After a Heart Attack." Which one of the following would best explain a low HEDIS rate for this measure? A. Pharmacies have not been billing all ß-blocker prescriptions to the health plan. B. Patients continue to take ß-blockers despite intolerable adverse effects. C. Patients are using their pharmacies' automated refill programs. D. Patients have very low copayments on ß-blockers

Answer A: Pharmacies have not been billing all ß-blocker prescriptions to the health plan. (If the pharmacist bills the health plan for the drug, the data are available for HEDIS reporting. If the patient pays cash for the generic, the health plan has no evidence of the claim, and the HEDIS rate will be artificially low (Answer A is correct). Another barrier to care is the intolerable adverse effects of ß-blockers (e.g., dizziness, fatigue, exercise intolerance). If patients do not take their ß-blockers because of the adverse effects, the HEDIS rate can be negatively affected. If patients take their medications despite the adverse effects, this action improves the HEDIS rate (Answer B is incorrect). Automated refill programs may help patients adhere to their medication regimens; thus, such programs may actually improve HEDIS rates (Answer C is incorrect). Although high copayments may be a barrier to care, especially in patients with a limited income, a low copayment does not represent a barrier to care. A low copayment is not a probable explanation for a low HEDIS rate (Answer D is incorrect).)

The following table represents the report cards issued by the National Committee for Quality Assurance (NCQA) for four health care plans. -------------------------------------------------------------------------- || Plan A || Plan B || Plan C || Plan D ---------------------------------------------------------- Access to Service || ** || *** || **** || **** ---------------------------------------------------------- Qualified Providers || *** || ** || ** || **** ---------------------------------------------------------- Staying Healthy || ** || ** || **** || **** ---------------------------------------------------------- Getting Better || ** || *** || *** || **** ---------------------------------------------------------- Living With Illness || ** || *** || **** || **** ---------------------------------------------------------- Accreditation Status || Accredited || Accredited || Commendable || Excellent ----------------------------------------------------------------------- Which plan is most likely to benefit from improving programs to assist patients in managing their asthma therapy and diabetes care? A. Plan A. B. Plan B. C. Plan C. D. Plan D.

Answer A: Plan A. ("Living with Illness" is a measure developed by the NCQA to evaluate health plan activities that help people manage chronic illness. "For example: Does the plan have programs in place to assist patients in managing chronic conditions such as asthma? Do diabetic patients who are at risk of blindness receive eye exams as needed?" Two stars show that the health plan meets basic requirements. Because Plan A has the fewest stars in this category, it would most likely benefit from implementing or improving programs that focus on chronic illnesses (Answer A is correct). Plan B has received three stars in the "Living with Illness" category, meaning it has met rigorous requirements for consumer protection and quality improvement (Answer B is incorrect). Plans C and D have received four stars, meaning they have exceeded rigorous requirements for consumer protection and quality improvement (Answer C and Answer D are incorrect).)

When data are clustered on the low end of a range of values (e.g., 1, 1, 1, 2, 2, 2, 3, 3, 9, 10), which one of the following best describes this distribution? A. Positive (right) skew. B. Negative (left) skew. C. Normal (balanced) skew. D. Nonmeasurable skew.

Answer A: Positive (right) skew. (A normal distribution has no skew; the mean is exactly at the peak value (Answer C is incorrect). Data on the low end pull the peak to the left, leaving a long tail on the right side of the peak value (i.e., a right shift). The mean of the data set is to the right of the peak value (Answer A is correct). A negative skew would occur when the data on the high end pull the peak to the right, leaving a long tail on the left side of the peak (i.e., a left shift). The mean of that data set would be to the left of the peak value (Answer B is incorrect). The degree to which a data set has skew can be determined by the formula 3 x (mean — median)/SD (Answer D is incorrect).)

A 55-year-old woman is being discharged home today, 5 days after she was admitted to the internal medicine service for pancreatitis. She has received her reconciled discharge drug list and has undergone counseling from a pharmacist on all of her drugs. In addition, a nurse discussed with her the laboratory orders, for which she will need to follow up as an outpatient. The patient will receive a follow-up telephone call by a pharmacist in several days and a patient satisfaction survey related to the hospitalization. Which one of the following negative outcomes will most likely be prevented in this patient because of these discharge interventions? A. Potential adverse drug events (ADEs). B. Duplicate laboratory orders. C. Poor medication adherence. D. Patient dissatisfaction.

Answer A: Potential ADEs. (In one study. pharmacist counseling at discharge and by telephone in 3—5 days significantly reduced preventable ADEs in the 30-day post-hospitalization period (Answer A is correct). In the same study, although the pharmacists inquired about laboratory appointments, no documented outcome was related to this inquiry (Answer B is incorrect). The study reported no difference between the intervention and control arms with respect to medication adherence and patient satisfaction (Answer C and Answer D are incorrect).)

A 6.8-year-old woman with a history of angina and H TN recently received a diagnosis of Graves disease and is markedly symptomatic. She experiences anxiety, heat intolerance, frequent bowel movements, and unintended weight loss. She is to undergo radioactive iodine (R A1) therapy in 2 weeks. Which one of the following is the best initial therapy for her pharmacologic management? A. Propranolol 40 mg orally four times/day. B. Methimazole 10 mg orally two times/day. C, Lugol's solution 5 drops orally three times/day. D. Propylthiouracil (PTU) 100 mg orally three times/day.

Answer A: Propranolol 40 mg orally four times/day. (This patient with Graves disease is markedly symptomatic and has a history of angina. Prompt symptom reduction that will not interfere with the planned RAI therapy is indicated. ß-Blockers take effect within hours of administration and do not interfere with the efficacy of RAI (Answer A is correct). Lugol's solution has a faster onset of action than thioamides, but it does not reach maximal effect for several days. In addition, taking iodides before RAI therapy will prevent RAI uptake by the thyroid gland, rendering its use ineffective (Answer C is incorrect). Thioamides such as methimazole and P TU inhibit organification and coupling but do not affect stored thyroid hormone levels; thus, several weeks are required before a reduction is seen in hyperthyroid symptoms (Answer B and Answer D are incorrect).)

A 16-year-old adolescent girl presents to the clinic for an annual examination. When asked about her vaccine history, she states she has not received the human papillomavirus (HPV) or HBV vaccine and is interested in receiving them. Her medical history shows that she has not had genital warts. Which one of the following is the best treatment approach for this patient? A. Series of HPV quadrivalent vaccine at 0, 2, and 6 months concurrently with HBV vaccine. B. HBV vaccine at 0, 1, and 6 months. C. HPV bivalent vaccine at 0, 1, and 6 months. D. HPV quadrivalent vaccine at 0, 1, and 6 months.

Answer A: Series of HPV quadrivalent vaccine at 0, 2, and 6 months concurrently with HBV vaccine. (This patient should be provided a series with either the quadrivalent vaccine or the bivalent vaccine concurrently with the recombinant HBV vaccine. The quadrivalent vaccine is the best choice because it protects against both genital warts and cancers. Although coverage with either HPV vaccine would be appropriate, the quadrivalent vaccine preferred. In addition, providing the appropriate administration schedule and quadrivalent vaccine protection against genital warts, while not missing an opportunity to administer the HBV vaccine, are all important factors in vaccinating this patient. Because the patient is relatively young and has no history of genital warts, she is likely to benefit from immunization against additional HPV strains. It is advisable for her to receive the HBV vaccine at the same time (Answer A is correct; Answer C and Answer D are incorrect). The patient should also receive protection with the HPV vaccine, not just the HBV vaccine (Answer B is incorrect).)

A 75-year-old woman with hypertension, Crohn disease, asthma, and type 2 diabetes mellitus is enrolled in a Medicare Part D prescription drug plan. She takes eight drugs on a chronic basis, and her drug expenditure for the calendar year was S3459 as of October. She is interested in receiving MTM services, if she qualifies. According to Centers for Medicare & Medicaid Services (CMS) regulations, which one of the following is most likely to disqualify this woman from receiving MTM services through her Medicare Part D prescription drug plan? A. She has only three of the seven core disease states. B. She takes only eight drugs chronically. C. Her drug expenditure was only $3459 for the first 9 months of the year. D. She is only 75 years of age.

Answer A: She has only three of the seven core disease states. (The CMS requires plans to target patients with at least four of the seven core chronic disease states. This patient has four chronic disease states, but only three fall into the seven core disease states outlined by CMS, so she would not be eligible for MTM services under most plans (Answer A is correct). More than 66% of Part D plans provide MTM services to patients taking eight or more drugs, so she would be eligible based on the number of drugs she is taking (Answer B is incorrect). In 2010, CMS lowered the drug expenditure amount for eligible patients from S4000 to $3000, so she would be eligible based on her annual drug expenditure (Answer C is incorrect). Patients older than 65 years are eligible for Medicare Part D benefits, so her age would not make her ineligible for these services (Answer D is incorrect).)

You have received promotional materials from a new drug wholesaler. Your pharmacy director asks you to order stocks of ß-blockers and angiotensin-converting enzyme inhibitors for your facility from the new company. After reviewing the promotional materials, you are unable to find any record of this wholesale drug distributor (WDD) being licensed or accredited. Which agency would be the best resource for further information on this supplier? A. State Board of Pharmacy. B. The FDA. C. National Association of Boards of Pharmacy (NABP). D. Drug Enforcement Agency (DEA).

Answer A: State Board of Pharmacy. (The State Board of Pharmacy is the regulatory body for the W DDs (Answer A is correct). If a WDD is conducting business in a state, it must be licensed in that state. Most states also require a W DD to be licensed in the state in which it is shipping drugs. Without the state licenses, the W DD is violating the law. The FDA is involved in the approval of drugs but not wholesale distributors (Answer B is incorrect). The NABP is an association of all the boards and is not a government agency; thus, has no legal authority (Answer C is incorrect). The DEA may be involved if the drug is a controlled substance or an anabolic steroid, which it is not in this case (Answer D is incorrect).)

A pharmacy's new vaccine coordinator is examining current procedures regarding compliance with Occupational Safety and Health Administration (OSHA) bloodborne pathogen standards. Which one of the following conditions best illustrates an example of noncompliance with current OSHA standards? A. The pharmacy's most recent exposure control plan (ECP) was completed 3 years ago. B. The pharmacy annually completes employee training for OSHA bloodborne pathogen standards. C. The pharmacy offers employees a hepatitis B vaccine within 7 days of assignment. D. Employees are not required to supply their own personal protective equipment while working.

Answer A: The pharmacy's most recent ECP was completed 3 years ago. (According to OSHA's bloodborne pathogen standards, the ECP should be updated annually at minimum; therefore, an ECP updated 3 years ago would not meet OSHA requirements (Answer A is correct). The OSHA bloodborne pathogen training for employees must be completed on an annual basis; this standard is in compliance (Answer B is incorrect). All employees who have job responsibilities such that they may exposed to bloodborne pathogens should be offered the hepatitis B vaccine within 10 days of starting employment; offering vaccination within 7 days meets the standard (Answer C is incorrect). Answer D meets compliance standards because employers must provide personal protective equipment free of charge to employees (Answer D is incorrect).)

After 5 years of a successful antimicrobial stewardship program at your institution, the hospital administrators are recommending its discontinuation and a shift in resources to other areas within the medical center. One explanation given for their recommendation is that antibiotic costs only modestly decreased in the past year compared with previous years. They believe the success of the program has become stagnant. Which of the following best describes the response of the stewardship team to this initiative? A. Studies have shown a substantial increase in drug costs after an antimicrobial stewardship program is discontinued; therefore, the program should be continued. B. Studies have shown that prescribing habits remain changed with continued cost savings after discontinuing an antimicrobial stewardship program; therefore, resources can be shifted to other areas. C. Studies have shown that fewer resources are needed for an antimicrobial stewardship program after the program has been in place for more than 5 years; therefore, the program should be continued with fewer staff and resources. D. Studies have shown that gram-negative resistance rates increase substantially after discontinuing an antimicrobial stewardship program; therefore, the program should be continued.

Answer A: Studies have shown a substantial increase in drug costs after an antimicrobial stewardship program is discontinued; therefore, the program should be continued. (A study by Standiford et al. describes antibiotic costs within a tertiary care academic medical center before the implementation of an antimicrobial stewardship program, during the program, and after discontinuation of the program. The program was discontinued after 7 years to shift resources to hire additional infectious dis- eases physicians to increase infectious diseases consults. After the program was discontinued, antibiotic costs significantly increased, equivalent to $2 million over 2 years (Answer A is correct; Answer B is incorrect). In addition, a study by Beardsley et al. has shown continued financial benefits of a stewardship program for more than 10 years. No data suggest an established program can be continued with fewer staff and resources (Answer C is incorrect). Moreover, no data are available evaluating the effect of discontinuing a stewardship program on changes in gram-negative resistance rates (Answer D is incorrect).)

You are aiding a physician in the appropriate antibiotic selection for an elderly man who has a medical history of health care—associated MRSA pneumonia. The disease has not responded to vancomycin with a trough of 18 mcg/mL. The susceptibilities for the patient's MRSA return and show a minimum inhibitory concentration (MIC) of 2 mcg/mL for vancomycin. Which one of the following is best to recommend for this patient? A. Switch vancomycin to linezolid. B. Increase the vancomycin dose to target a trough of 20—25 mcg/mL. C. Switch vancomycin to daptomycin. D. Switch vancomycin to ceftaroline.

Answer A: Switch vancomycin to linezolid. (Discontinuing vancomycin therapy should be considered when patients do not respond to therapy, when adequate serum levels cannot be achieved, or when the MIC is 2 mcg/mL or greater. With MICs greater than 1.5 mcg/mL, failures in vancomycin therapy have occurred, and targeting an area under the curve (AUC)/MIC greater than 400 is not very feasible (Answer B is incorrect). Daptomycin is effective against MRSA, but it should not be used for the treatment of pneumonia because of the effects on the drug by lung surfactant (Answer C is incorrect). Ceftaroline is approved for skin and soft tissue infections and CAP; however, data on its use for health care—associated pneumonia, especially compared with currently recommended therapies, are limited (Answer D is incorrect). The best option is to change vancomycin to an appropriate alternative agent such as linezolid (Answer A is correct).)

A woman who takes carbamazepine calls you to ask whether Tegretol XR tablets "are as good as" Carbatrol XR capsules. Her insurance provider no longer covers Carbatrol XR, and she was forced to change products. You find an open-label, crossover, pharmacokinetic study comparing the two dosage forms. Which type of statistical testing would be most appropriate for evaluating the differences in the maximum concentration (Cmax) and area under the curve (AUC)? A. Two one-sided tests at the 0.05 level of significance using a 90% Cl. B. Two one-sided tests at the 0.025 level of significance using a 95% Cl. C. A single two-sided test at the 0.05 level of significance using a 95% Cl. D. A single two-sided test at the 0.01 level of significance using a 99% Cl.

Answer A: TWO one-sided tests at the 0.05 level of significance using a 90% Cl. (Use of two one-sided statistical tests for each parameter (Cmax and AUC) with a 90% CI is considered appropriate for bioequivalence studies (Answer A is correct). The current practice of carrying out two one-sided tests at the 0.05 level of significance ensures that if the two products truly differ by as much as (or more than) allowed by the equivalence criteria, the chance that they will be approved as equivalent is no more 5%. Two-sided statistical tests and the higher 95% CI are not required (Answer B, Answer C, and Answer D are incorrect).)

A 65-year-old man presents to his physician with symptoms that are determined to be erectile dysfunction (ED). He has a history of hyperlipidemia, gastroesophageal reflux disease, and glucose intolerance. His home drugs include atorvastatin 20 mg orally daily, omeprazole 20 mg orally daily, and aspirin 81 mg orally daily as tolerated. He asks about medications to help with his symptoms but notes that he does not want to have to plan his intimate moments. Which one of the following therapies would be best for this patient? A. Tadalafil. B. Vardenafil. C. Yohimbine. D. Bupropion.

Answer A: Tadalafil. (Tadalafil may be dosed daily without respect to timing of sexual intercourse (Answer A is correct). Vardenafil should be taken 60 minutes before sexual intercourse (Answer B is incorrect). Yohimbine and bupropion are not first-line therapies for ED (Answer C and Answer D are incorrect).)

A patient describes his frustration with using the Internet for obtaining health information. He is aware of the potential for finding inaccurate information. However, he values the ease with which he can find information. He goes on to describe additional frustration with finding tools for assessing the quality of online health information. He wants to go to any site he desires and quickly have an indication of quality with minimal effort on his part. Which one of the following is the best assessment tool to fit this patient's need? A. The Health on the Net (HON) toolbar. B. American Medical Association guidelines. C. Healthfinder. D. QUality Information ChecKlist (QUICK).

Answer A: The HON toolbar. (Your patient has identified a common problem: the Internet offers good information, but patients may have trouble easily identifying it. It is important that patients understand the different roles that the various evaluation services play. The HON Foundation, like all seals of quality services, maintains a staff that performs periodic reviews of sites displaying its logo. Sites receiving favorable reviews by the HON Foundation are able to display the HON logo as an active hyperlink. Patients who have installed the HON toolbar have a visual indication when they visit health sites that have been granted the HON logo. The HON toolbar automatically provides an indication of information quality with no effort on the part of the user (Answer A is correct). The American Medical Association guidelines are recommendations for its Web site. Other sites can use these guidelines, but visitors to these sites have to search for an indication that these guidelines were followed, and the patient described a desire to go to any site and have a quick indication of information quality (Answer B is incorrect). As a filtering service, Healthfinder requires users to start their searches on the Healthfinder Web site; thus, this will not meet the patient's need of searching any site (Answer C is incorrect). Some information assessment tools are designed to target specific populations. QUICK is intended to educate young children on the need and process for evaluating online health information and will not meet this patient's need (Answer D is incorrect).)

You are asked by a colleague to interpret reliability results for this abstract: "The Hypertension Care Survey was designed to measure psychosocial factors related to hypertension and its treatment. This study sought to determine the reliability and validity of the scales on the self-administered survey in a Hispanic population with hypertension. Hispanic patients (n = 300) were recruited at three retail pharmacies in southern Florida. Scale reliabilities calculated by Cronbach's coefficient U revealed reliabilities ranging from 0.25 to 0.34 in the Hispanic population. Mean values on the various scales of the survey were drastically different from values found when administered previously to white populations. Correlation coefficients between scale scores within the Hispanic population and external measures were low (e.g., r = 0.15)." Which one of the following is the best interpretation of the study results? A. The scales cannot be considered reliable in this population because of the low-coefficient scores. B. Because the correlation coefficients are low, reliability is probably acceptable but validity is not. C. Interrater reliability was not assessed; therefore, it is difficult to make any judgments about the reliability of the survey. D. The coefficient a score range is less than I, so it is acceptable; therefore, these scales are considered highly reliable in a Hispanic population.

Answer A: The scales cannot be considered reliable in this population because of the low-coefficient a scores. (The Cronbach coefficient a measures the reliability of a scale. An U value greater than 0.70 is desired. In this abstract, the a value is less than 0.34, indicating that the measure is not consistent in this population (Answer A is correct; Answer D is incorrect). The correlation coefficients provided in the abstract are a measure of criterion validity, not reliability. Thus, validity may be a problem, but reliability as shown by the Cronbach is not acceptable (Answer B is incorrect). Because this survey was self-administered and multiple interviewers were not involved in this study, no measure of interrater reliability was assessed (Answer C is incorrect).)

A placebo-controlled study evaluated the ability of a new sedative hypnotic to decrease patients' time awake at night. The investigators used a significance level of 0.05 and required 50 patients in each arm for a power of 80% to find a decrease in time awake of 1 hour. Which one of the following best describes the probability of making a type II error in this study? A. 5% B. 20% C. 50% D. 80%

Answer B: 20% (The probability of making a type II error is equal to the beta error rate. Power is calculated from the beta error rate (power = 1 — ß), which is 20% in this case (Answer B is correct). Five percent is the level of significance set for this study (Answer A is incorrect). The number of patients in each arm of the study is 50, but this is not a percentage risk of type II error (Answer C is incorrect). Eighty percent is the power of the study (Answer C is incorrect).)

A 55-year-old-woman is admitted to the ICU with sepsis secondary to a urinary tract infection (UTI). She is started on empiric therapy with ceftriaxone. On day 2, the patient is afebrile and her WBC is 6 x 10^3 cells/mm^3 Subsequent culture and sensitivity data support her empiric treatment. Which one of the following is the best treatment duration for this patient? A. 3 days. B. 7 days. C. 14 days. D. 6 weeks.

Answer B: 7 days. (For infections in which the source control is adequate, such as a UT I, antibiotic therapy for 7 to 10 days is appropriate (Answer B is correct). Although 3-day therapy is appropriate for an uncomplicated UT I, the complication of sepsis necessitates a longer treatment course (Answer A is incorrect). Treatment courses of 14 days and 6 weeks are too long given current recommendations, the patient's clinical responsiveness, the most likely organisms, and adequate source control (Answer C and Answer D are incorrect).)

Which one of the following patients would be the most appropriate candidate to receive pharmacologic prophylaxis against Pneumocystis jiroveci pneumonia (formerly P. carinii pneumonia [PCP])? A. A 40-year-old HIV-positive man with no history of opportunistic infections who has not yet taken antiretrovirals and who has a CD4+ count of 248 cells/mm^3. B. A 28-year-old woman with lupus nephritis taking prednisone 20 mg by mouth daily and cyclophosphamide 500 mg intravenously every 2 weeks for the past 6 months. C. A 58-year-old man with rheumatoid arthritis taking prednisone 5 mg by mouth daily and methotrexate 15 mg by mouth once weekly for past 2 weeks. D. A 65-year-old man 2 years after a renal transplant on chronic immunosuppression with mycophenolate mofetil 500 mg by mouth twice daily and tacrolimus 1 mg by mouth twice daily.

Answer B: A 28-year-old woman with lupus nephritis taking prednisone 20 mg by mouth daily and cyclophosphamide 500 mg intravenously every 2 weeks for the past 6 months. (Prophylaxis for PCP is indicated in patients with HIV who have a history of PCP infection, oropharyngeal candidiasis, or a CD4+ cell count less than 200 cells/mm3 (Answer A is incorrect). Prophylaxis for PCP is indicated in patients without HIV infection receiving a glucocorticoid therapy dose equivalent to prednisone 20 mg or more daily for 1 month or longer, plus another cause of immunosuppression such as hematologic malignancy or another immunosuppressive drug (Answer B is correct). The PCP prophylaxis for solid organ transplant patients is typically indicated 6—12 months posttransplant (longer periods may be warranted for lung transplant patients) and during periods of high doses of immunosuppressants. A patient with rheumatoid arthritis who is taking prednisone 5 mg does not need PCP prophylaxis (Answer C is incorrect). Recent Kidney Disease/lmproving Global Outcomes guidelines recommend PCP prophylaxis for 3—6 months after a kidney transplant. A patient who is 2 years posttransplantation with lower-dose immunosuppression is not a candidate for PCP prophylaxis (Answer D is incorrect).)

Each of the following patients has an acute coronary syndrome (ACS) and is scheduled to undergo percutaneous coronary intervention (PCI). Which one is most likely to derive the greatest benefit from prasugrel compared with clopidogrel? A. A 75-year-oId woman (weight 70 kg) with NSTEMI B. A 45-year-old man (weight 80 kg) with type 2 diabetes mellitus (T2DM) and ST-segment elevation myocardial infarction (STEMI). C. A 77-year-old woman (weight 50 kg) with T2DM NSTEMI. D. A 50-year-old man (weight 90 kg) with STEMI and prior stroke.

Answer B: A 45-year-oId man (weight 80 kg) with T2DM and STEMI. (In the TRITON-TIMI 38 trial of patients with STEMI, the primary end point of cardiovascular (CV) death, myocardial infarction (MI), or stroke at 15 months was reduced with prasugrel compared with clopidogrel patients with low body weight (i.e., less than 60 kg) had a higher rate of thrombolysis in MI major non-coronary artery bypass graft bleeding (Answer C is incorrect). Patients older than 75 were also at increased bleeding risk (Answer A and Answer C are incorrect). Patients with DM had the greatest benefit in TRITON-TIMI 38 with a 4.8% absolute risk reduction in the 15-month rate of CV death, MI, or stroke with no increased risk of bleeding. Prior stroke is a contraindication to prasugrel because the patients in TRITON-TIMI 38 experienced no benefit but had an increased risk of intracranial hemorrhage compared with clopidogrel (Answer D incorrect). Therefore, the younger patient with T2DM is at lowest risk compared with an older patient and a patient without DM (Answer B is correct).)

To improve vaccination rates at the hospital, the pharmacy department has started a standing order vaccination program. You are responsible for screening all inpatients at risk of developing invasive pneumococcal disease. For which one of the following patients would it be best to recommend PPSV23 before discharge? A. An 80-year-old man with congestive heart failure who received PPSV23 at age 68. B. A 45-year-old woman with asthma. C. A 3-day-old girl born to a woman who is immunocompromised. D. A 62-year-old man with pneumonia who received PPSV23 at age 56.

Answer B: A 45-year-old woman with asthma. (The Centers for Disease Control and Prevention (CDC) updated the recommendations for prevention of invasive pneumococcal disease to include vaccination of patients 19—64 of age who have asthma. The 45-year-old woman with asthma should receive a pneumococcal vaccine before discharge (Answer B is correct). Revaccination is indicated 5 years after the first dose only for individuals 19—64 years of age with functional or anatomic asplenia and for individuals who are immunocompromised. The patient in Answer A received one dose of pneumococcal vaccine after age 65, and revaccination is not indicated (Answer A is incorrect). Use of PPSV23 is indicated only in patients older than 18 (Answer C is incorrect). The patient in Answer D has no immunocompromised condition that would warrant revaccination before age 65 (Answer D is incorrect).)

The Affordable Care Act established the Medication Management Services in the Treatment of Chronic Disease program, which will provide grants for such programs. Which one of the following patients is most likely to be targeted by this program? A. A 45-year-old man with hypertension receiving acetaminophen, as needed; hydrochlorothiazide; and sildenafil, as needed. B. A 66-year-old woman with atrial fibrillation receiving warfarin. C. A 51-year-old man with hypercholesterolemia receiving simvastatin, fenofibrate, and lisinopril. D. A 35-year-old man with a family history of diabetes, hypertension, and hyperlipidemia who has hypertension, lives a sedentary lifestyle, and takes trandolapril daily.

Answer B: A 66-year-old woman with atrial fibrillation receiving warfarin. (The Medication Management Services in the Treatment of Chronic Disease program will provide grants or contracts to eligible entities to implement such services provided by pharmacists as part of a collaborative interdisciplinary team. These grant programs will target patients who (l) take four or more prescribed medications, including over-the-counter drugs and dietary supplements; (2) take any high-risk drugs, such as warfarin as taken by the 66-year-old woman (Answer B is correct); (3) have two or more chronic diseases; or (4) have undergone a transition of care or other factor likely to create a high risk of drug-related problems. The other patients described in the answer options do not meet these criteria (Answer A, Answer C, and Answer D are incorrect).)

You conducted a nationwide mail survey of pharmacists. There are 250,000 registered pharmacists in the United States on a list you purchased from a market research firm. You selected a random sample of 1000 pharmacists to survey and mailed the survey with a return envelope and a follow-up postcard 2 weeks later to the sample. Of these 1000 surveys, 20 were returned because of incorrect address and 325 were returned from responding pharmacists. Of the 325 surveys returned, 30 were unusable. Which one of the following is the best interpretation of the usable response rate? A. Only 30 of the surveys returned were unusable, so a usable response rate of 91% is acceptable. B. About 295 usable surveys were returned from this mail survey, yielding a usable response rate of 30%, which is acceptable. C. Because not all surveys were delivered, an accurate response rate cannot be determined. D. Less than 50% of the mailed surveys were returned, so a usable response rate of 36% is not acceptable.

Answer B: About 295 usable surveys were returned from this mail survey, yielding a usable response rate of 30%, which is acceptable. (To calculate the usable response rate, the numerator is the number of usable responses, which is 295 (i.e., 325— 30). The denominator is the number of surveys mailed and received, which is 980 (i.e., 1000 — 20). Thus, the response rate is 295/980, which is 30% (Answer B is correct). For mail surveys, response rates of 10%—20% are often seen, making a 30% response rate acceptable, assuming no response bias or other problems with the survey methodology occurred. A usable response rate calculated to be 36% is incorrect and suggests that less than 50% of the surveys were returned; this response rate is not acceptable (Answer D is incorrect). A 91% usable response rate also does not state the correct usable response rate (Answer A is incorrect). It is common to have some surveys incorrectly addressed and not deliverable. This occurrence does not preclude the researcher from calculating a response rate (Answer C is incorrect). If the number of nondeliverables is extremely high, the researcher should investigate why and determine whether bias has been introduced. In addition, many nondeliverables will decrease the number of people receiving surveys, making it more difficult to achieve an acceptable response rate.)

A 61-year-old woman presents with PD and peak-dose dyskinesias. In the past several months, she has undergone several changes in her antiparkinson regimen to achieve an acceptable balance between dyskinesias and symptom control. Her current antiparkinson regimen consists of carbidopa/levodopa controlled release 50/200 mg three times daily, ropinirole 2 mg three times daily, and benztropine I mg three times daily. Her other drugs include folic acid 1 mg once daily, paroxetine 20 mg once daily, and clonazepam 1 mg once daily at bedtime. Which one of the following is best to recommend for the management of this patient's dyskinesias? A. Add rasagiline. B. Add amantadine. C. Increase the ropinirole dose. D. Increase the benztropine dose.

Answer B: Add amantadine. (Amantadine is effective in the management of peak-dose dyskinesia (Answer B is correct). Adding a monoamine oxidase B inhibitor or increasing the dopamine agonist dosage worsens dyskinesias (Answer A and Answer C are incorrect). Increasing the dose of an anticholinergic agent is not effective for managing dyskinesias (Answer D is incorrect).)

A 64-year-old African American man presents to his ophthalmologist today for a follow-up examination. His medical history includes T2DM for 24 years, H TN for 26 years, and hyperlipidemia for 10 years. The patient reports taking metoprolol 50 mg two times/day, hydrochlorothiazide 25 mg/day, lisinopril 10 mg/day, glimepiride 8 mg/day, metformin 1000 mg two times/day, aspirin 81 mg/day, atorvastatin 40 mg/day, and latanoprost I drop in each eye at bedtime. He lives alone, drinks two beers per week, and quit smoking I year ago (but smoked 1 pack/day for 45 years). His mother (deceased) had T2DM, headaches, and vision loss, and his father (deceased) had hyperlipidemia and cardiovascular disease. At his last visit 3 months ago, latanoprost was initiated after ocular examination revealed a normal visual field test and intraocular pressure (IOP) of 34 mm Hg in both eyes (baseline IOP in both eyes of 25 mm Hg). Today, his dilated examination reveals some optic nerve damage, and the IOPs are now 36 mm Hg in each eye, with the visual field test revealing a slight loss of peripheral vision. Which of the following is best to recommend for this patient? A. Continue current therapy. B. Add dorzolamide. C. Recommend laser trabeculoplasty. D. Add oral acetazolamide.

Answer B: Add dorzolamide. (Given the elevation of IOP in this patient, he will likely require combination therapy to reach the goal IOP of less than 21 mm Hg, or 30%—50% lower than the pretreatment level (Answer A is incorrect). Adding dorzolamide to the patient's latanoprost dose is the most appropriate recommendation at this time (Answer B is correct). Adding an oral agent such as acetazolamide is not the best choice for treatment of open-angle glaucoma (Answer D is incorrect). Laser trabeculoplasty is an option for patients whose drug regimens have failed to sufficiently control IOP; therefore, the patient should receive at least 6 months of pharmacologic treatment before being considered for laser surgery (Answer C is incorrect).)

A 68-year-old man (height 6'2", weight 113 kg [249 lb]) is admitted to the emergency department (ED) with confusion, fever, and a productive cough. About 2 weeks ago, he was treated as an outpatient for bronchitis with a 7-day course of cefuroxime axetil. His medical history includes depression, asthma, and thrombotic thrombocytopenic purpura, and he is postsplenectomy but has no history of confusion or dementia. He is allergic to vancomycin (hives). The patient's home drugs include sertraline 50 mg/day, a multivitamin daily, and an albuterol inhaler as needed. Vital signs include temperature 1010F (38.30C), blood pressure (BP) 141/82 mm Hg, heart rate (HR) 108 beats/minute, and respiratory rate (RR) 24 breaths/minute. Laboratory tests show WBC 14.9 x 10^3 cells/mm3, platelet count 100,000 cells/mm3, Na 135 mEq/L, K 3.2 mEq/L, BUN 15 mg/dL, SCr 1.1 mg/dL, and serum glucose 143 mg/dL. Results of a lumbar puncture show cerebrospinal fluid (CSF) WBC 2.2 x 10^3 cells/mm 3 (reference value 0-5), neutrophils 94% (0%—6%), monocytes 5% (0%—30%), eosinophils 1% (()%—5%), protein 573 mg/dL (15—45 mg/dL), and glucose less than I mg/dL. Blood and CSF Gram stain yield gram-positive diplococcus. Head computed tomography (CT) shows sinusitis; radiography is negative. The patient is initiated on dexamethasone. Which one of the following, in addition to intravenous ceftriaxone, is the best empiric regimen for this patient? A. Vancomycin 2.5 g intravenously. B. Vancomycin 2.5 g intravenously and acyclovir 1000 mg intravenously. C. Ampicillin 2 g intravenously. D. Moxifloxacin 400 mg intravenously.

Answer D: Moxifloxacin 400 mg intravenously. (The patient's CSF is positive for gram-positive diplococcus, which is most likely Staphylococcus pneumoniae. He has been exposed to a course of oral cephalosporin, which may increase the risk of a cephalosporin-resistant strain. Moxifloxacin has activity against penicillin-resistant and third-generation cephalosporin—resistant S. pneumoniae. Moxifloxacin also has adequate central nervous system (CNS) penetration and is further recommended when vancomycin, ceftriaxone, or cefotaxime are unable to be used according to the guideline (Answer D is correct). Moxifloxacin should be used in combination with a ceftriaxone or cefotaxime until culture results are available. Because the patient has a documented allergy to vancomycin (hives) and no indication for acyclovir therapy, vancomycin and acyclovir are poor choices (Answer A and Answer B are incorrect). The use of ampicillin is not recommended empirically for S. pneumoniae (Answer C is incorrect).)

A 5-year-old boy is brought to the physician's office by his father, who inquires about influenza vaccination for the child. The boy's medical history includes allergic rhinitis. His drug and food allergies include penicillin (rash) and eggs (mild hives). Neither reaction included shortness of breath or anaphylaxis. Which one of the following is most appropriate for this patient? A. Administer 10% of the LAIV4 dose and observe for 30 minutes; if no reaction occurs, then administer the remaining 90% of the dose. B. Administer 100% of the II V 3 dose and observe for a reaction for at least 30 minutes after vaccination. C. Use skin prick testing to determine the risk of reaction before administering the influenza vaccine. D. Do not administer any influenza vaccine to this patient because the risks outweigh the benefits.

Answer B: Administer 100% of the II V 3 dose and observe for a reaction for at least 30 minutes after vaccination. (The benefits of influenza vaccination in pediatric patients have been well documented. In addition, results of recent studies suggest that influenza vaccine can be safely administered to patients with a previous reaction of hives after egg consumption. The health care provider administering the vaccine should be aware of the signs and symptoms consistent with an allergic reaction and should be well versed in appropriate management if a reaction occurs. When administered to this patient population, it is prudent to include an observation time of at least 30 minutes after the vaccination to observe for a possible reaction. Consequently, the most appropriate recommendation for this patient is to administer full-dose II V 3 and then observe for at least 30 minutes (Answer B is correct). Given these factors, a decision not to administer any influenza vaccine to this patient is not optimal (Answer D is incorrect). Skin prick testing is not highly predictive of allergic reactions and should not be used to determine risk of reaction (Answer C is incorrect). Split-dose administration has been used successfully in some studies; however, studies have evaluated the use of II V 3, not LAIV4. In this context, administering 10% of the LAIV4 dose and observing for .30 minutes before administering the remainder is not the best answer (Answer A is incorrect).)

A 61-year-old man is transported to a level I trauma center after a motor vehicle crash. The patient's medical history is significant for hypertension and an ischemic stroke 4 years ago with no residual deficits. He is intubated, placed on mechanical ventilation, and resuscitated according to the trauma center's protocol. The initial neurologic examination reveals a postresuscitation Glasgow Coma Scale score Of 8, reactive pupils, and localizing of all extremities on painful stimulation. Computed tomography reveals a small epidural hematoma, a nondisplaced parietal skull fracture, and a left acetabulum fracture. The neurosurgeon determines that no surgical intervention is warranted at this time, although an intracranial pressure (ICP) monitoring catheter is inserted. The patient is receiving intravenous propofol at I mg/kg/hour and fentanyl at 75 mcg/hour. His initial ICP is 17 mm Hg. During the next 2 hours, his ICP ranges from 20 mm Hg to 26 mm Hg and his mean arterial pressure (MAP) ranges from 100 mm Hg to 110 mm Hg. His sodium concentration is 141 mEq/L. Which of the following is best to recommend for this patient? A. No acute therapy for ICP control is needed. B. Administer hypertonic saline intravenously. C. Administer lactated Ringer's solution intravenously D. Begin cisatracurium intravenous infusion at 80 mcg/hour.

Answer B: Administer hypertonic saline intravenously. (Administering hypertonic saline is a recommended therapy of increased ICP, and doses and concentrations used have been variable (Answer B is correct). Mannitol is also an approved agent for treatment of elevated ICP. The elevated osmolarity of hypertonic saline and mannitol contributes to the osmotic gradient established in the serum relative to the brain. Intracellular fluid follows this gradient and decreases the volume; hence the pressure exerted in the enclosed intracranial vault. The threshold for treatment of an elevated ICP is 20 mm Hg according to current guidelines; therefore, this patient should receive treatment for his elevated ICPs of 20—26 mm Hg (Answer A is incorrect). The use of lactated Ringer's would not address the patient's elevated ICP because it is isotonic and does not establish an osmotic gradient (Answer C is incorrect). Neuromuscular blocking drugs should not be used routinely in patients with traumatic brain injury with increased ICP based on evidence that they increase complications (e.g., pneumonia, prolonged paralysis) and length of stay (Answer D is incorrect).)

A 55-year-old with a medical history of nonischemic dilated cardiomyopathy (LVEF 25%), schizophrenia, and paroxysmal atrial fibrillation (AF) presents to clinic with palpitations, fatigue, and shortness of breath. Her ECG that is in AF with a ventricular rate of 125 beats/minute and a QT of 400 milliseconds. Her home drugs include enalapril 10 mg twice daily orally, metoprolol extended release 200 mg daily orally, spironolactone 25 mg daily orally, quetiapine 200 mg twice daily orally, and warfarin 3mg daily orally. She states that she is adherent to her drugs. Laboratory values include SCr 0.8 mg/dL, INR 2.7 (stable for the past 2 months). Her Vital Signs include BP 115/70 mm Hg and HR 120 beats/minute. Which one of the following would best promote cardioversion of AF and maintenance of normal sinus rhythm in this patient? A. Dofetilide 500 mcg twice daily for 2 weeks; then 125 mcg daily. B. Amiodarone 400 mg twice daily for 2 weeks; then 200 mg daily. C. Digoxin 0.5 mg x 1; then 0.25 mg daily. D. Propafenone 600 mg x 1; then 300 mg twice daily.

Answer B: Amiodarone 400 mg twice daily for 2 weeks; then 200 mg daily. (Flecainide, dofetilide, propafenone. ibutilide (all class l), or amiodarone (class IIa) are options for use in pharmacologic cardioversion of AF. However, options are limited when a patient has structural heart disease or a reduced ejection fraction (i.e., less than 40%). In this patient with HF with reduced ejection fraction, only dofetilide and amiodarone are options. Dofetilide is dosed on the basis of the patient's CrCl, but no loading dose is required. Dosing would be reduced only if the CrCl changes or if the QT interval is prolonged during monitoring of initiation (first 3 days). The dosing of dofetilide is not appropriate (Answer A is incorrect). Amiodarone does require a loading dose, and 400 mg twice daily for 2 weeks is reasonable. The usual maintenance dose of amiodarone for the treatment of AF is 200 mg daily (Answer B is correct). Digoxin and/or sotalol may be harmful when used for pharmacologic cardioversion; thus, neither is recommended (Answer C is incorrect). Propafenone would be a reasonable option if this patient had a preserved ejection fraction (Answer D is incorrect).)

After undergoing surgery for tetralogy of Fallot, a 2-month-old boy presents with junctional ectopic tachycardia. He is sufficiently sedated and is provided magnesium and potassium supplementation intravenously. He is also hemodynamically stable. Overdrive atrial pacing was unsuccessful at returning him to normal sinus rhythm. Which one of the following is most appropriate to use next in treating this patient's cardiac arrhythmia? A. Adenosine. B. Amiodarone. C. Dobutamine. D. Lidocaine.

Answer B: Amiodarone. (Initially, proper sedation and electrolyte management (Mg and K) are tried with the goal of minimizing catecholamine release and further exacerbation of the arrhythmia. Overdrive atrial pacing helps in about 30% of cases to return to goal heart rate. However, in this case, the patient requires further pharmacotherapy management. Amiodarone is the first-line antiarrhythmic agent to treat junctional ectopic tachycardia and is given in intermittent doses every 30 minutes until a response is achieved, with a maximal total dose of 15 mg/kg (Answer B is correct). Adenosine is not optimal in this case because the patient does not have supraventricular tachycardia, in which presentation on and off is rapid (Answer A is incorrect). Dobutamine, a ß-agonist, should not be used because it could further exacerbate the patient's tachycardia (Answer C is incorrect). Lidocaine is a second-line agent for sustained ventricular tachycardia or fibrillation but is not indicated for use in junctional ectopic tachycardia (Answer D is incorrect).)

A 69-year-old married man with glioblastoma multiforme has cancer that recurred about 6 months after adjuvant therapy with temozolomide and radiation. His medical history is significant for uncontrolled type 2 diabetes mellitus, amputation of the right foot because of a foot infection, and chronic back pain from spinal instrumentation secondary to a motor vehicle crash. His current drug regimen includes lisinopril, metformin, phenytoin, insulin, and morphine. The recurrence of glioblastoma multiforme is being treated with irinotecan plus bevacizumab every 14 days. Which of the following is the best antidiarrheal regimen for this patient while he is taking irinotecan plus bevacizumab? A. Atropine 0.25 mg intravenously before irinotecan; then diphenoxylate-atropine 2 tablets orally every 6 hours for 4 days, followed by every 6 hours as needed. B. Atropine 0.25 mg intravenously before irinotecan; then loperamide 4 mg orally every 4 hours (24 mg day maximum), starting with the first sign of diarrhea. C. Atropine 0.5 mg intravenously as needed; then loperamide 4 mg orally every 6 hours (16 mg/day maximum), starting on day 2. D. Loperamide 4 mg orally every 4 hours starting the morning of the irinotecan infusion and continuing until no stools occur in 12 consecutive hours.

Answer B: Atropine 0.25 mg intravenously before irinotecan; then loperamide 4 mg orally every 4 hours (24 mg/day maximum), starting with the first sign of diarrhea. (Irinotecan-associated diarrhea at two distinct times, owing to two discrete pathophysiologic processes. Early-onset diarrhea occurs within 24 hours of irinotecan administration, but it may manifest during the infusion. [Delayed-onset diarrhea at a very high incidence with irinotecan and manifests beyond 24 hours after infusion. Early-onset diarrhea is caused by cholinergic overdrive from irinotecan, and administration of an anticholinergic drug is essential, with atropine being the drug of choice. However, cholinergic phenomena are not guaranteed. Because this is the first dose of irinotecan, atropine may be given irinotecan or as needed if symptoms arise. Delayed-onset diarrhea occurs from toxicity to gastrointestinal epithelium, for which loperamide has a preponderance of data supporting use. is usually not scheduled after the first dose of irinotecan, but on the first signs of diarrhea, and is continued until resolution of diarrhea and development of formed Therefore, atropine 0.25 mg intravenously before irinotecan and then loperamide 4 mg orally every 4 hours (24 mg/day maximum), starting with the first sign of diarrhea, is the answer (Answer B is correct). Diphenoxylate-atropine oral tablets are not the preferred agent for chemotherapy-induced diarrhea because of a lack of supporting data and the theoretical benefit of loperamide because it undergoes enterohepatic recirculation in the gastrointestinal tract (Answer A is incorrect). For administration of regimens that do not include irinotecan, the total daily dose of loperamide is arbitrarily capped at 16 mg/day to prompt patients to seek medical attention for diarrhea not responsive to this daily dose. The total daily dose of loperamide after irinotecan is 24 mg/day. Therefore, capping the loperamide dose at 16 mg is incorrect (Answer C is incorrect). Beginning loperamide before administering irinotecan and continuing until no loose occur for 12 hours is incorrect because atropine is not included for acute diarrhea, and loperamide should not initiated until diarrhea occurs.)

A hospital pharmacy manager is conducting a quality improvement project. He wants to measure physician knowledge of asthma guidelines at two points: (I) before implementing the quality improvement interventions and (2) 6 months after implementation. The pharmacy manager develops an asthma guideline knowledge test. Which one of the following is the best approach to conducting this study? A. Cross-sectional survey design with a control group; use a self-administered survey; mail the survey to all physicians in the state; and mail the follow-up survey to all responding physicians in 6 months. B. Before-and-after study design; use a self-administered survey; randomly select hospital physicians to complete the survey; and follow up with physicians in 6 months to resurvey. C. Panel study design; develop an Internet survey; post recruitment flyers around the hospital that instruct physicians to complete the Internet survey today; and then do the follow-up survey in 6 months. D. Before-and-after study design; use a self-administered questionnaire; randomly select physicians, nurses, and pharmacists in the hospital to complete the survey; and follow up with participants in 6 months to resurvey.

Answer B: Before-and-after study design; use a self-administered survey; randomly select hospital physicians to complete the survey; and follow up with physicians in 6 months to resurvey. (This scenario emphasizes that all four aspects of survey design are intertwined and that all aspects must match the study purpose. The proposal to use a before-and-after study design that is appropriate for the study purpose is the best option because a random sample of physicians, which is the population of interest, will be selected (Answer B is correct). The study purpose requires both a pre- and post-survey design, and the pharmacy manager is interested in only physicians at the hospital, not physicians across the state (Answer A is incorrect). Use of a convenience sampling technique is not the best approach, and it is not practical to ask respondents to remember to complete the survey again in 6 months. It is also unclear whether the Web site allows only physicians to complete the survey (Answer C is incorrect). A sample that includes physicians, nurses, and pharmacists is not optimal because this group is not the population of interest (Answer D is incorrect).)

A 27-year-old woman is referred to an endocrinologist by her gynecologist after experiencing irregularities in her menstrual cycle. Laboratory tests show a thyroid-stimulating hormone (TSH) level of 1.9 mlU/L and a prolactin level of 247 ng/mL. Subsequent magnetic resonance imaging of the pituitary identifies a solitary 6-mm adenoma. The patient is interested in becoming pregnant at age 29—30. Which of the following is the best recommendation for this patient? A. Bromocriptine 5 mg twice daily. B. Cabergoline 0.25 mg twice weekly. C. Clomiphene 50 mg once daily for 5 days. D. Metformin 1000 mg twice daily.

Answer B: Cabergoline 0.25 mg twice weekly. (The hypothalamus does produce a hormone that stimulates prolactin production; rather, its production is inhibited by dopamine secreted by the hypothalamus. Prolactin-secreting pituitary tumors secrete an excessive amount of prolactin, which in turn causes a decrease in luteinizing hormone and follicle-stimulating hormone production by the pituitary This decrease results in decreased endogenous sex hormone production (i.e., estrogen in women and testosterone in men) and may lead to abnormal menstrual cycles and infertility. Treatment of hyperprolactinemia involves the use of dopamine agonist therapy, which normalizes prolactin levels. Of the choices listed, only cabergoline and bromocriptine are dopamine agonists. Cabergoline is recommended as initial therapy over bromocriptine because of its greater efficacy, ability to normalize prolactin levels, and ability to reduce the size of the adenoma (Answer B is correct). Current guidelines recommend that any dopamine agonist therapy be discontinued when pregnancy is confirmed to minimize risk to the developing fetus. Should a patient require therapy during pregnancy because of tumor growth and/or increased symptoms, bromocriptine would be the preferred agent. Because this patient is planning to start a family for 2—3 years, cabergoline would be preferred to bromocriptine at this time because of its better efficacy. In addition, the starting dose of bromocriptine should not exceed 2.5 mg day, and dese should adjusted to avoid gastrointestinal adverse effects. The starting dose of 5 mg twice daily is too high (Answer A is incorrect). Although metformin and clomiphene are used in the management of infertility, neither agent would address the underlying cause of this patient's menstrual irregularities (i.e., prolactinemia) (Answer C and Answer D are incorrect).)

You are studying the relationship between the use of decongestants and hemorrhagic stroke, a rare event in young, otherwise healthy women. Which one of the following study designs would be most appropriate? A. Cohort. B. Case-control. C. Randomized controlled trial. D. Prospective.

Answer B: Case-control. (Case-control studies are well suited to studying rare events and are generally more efficient than cohort studies (Answer B is correct). In this case, the outcome of interest is rare, so a large cohort would have to be followed for a long time. In addition, collecting information on all non-cases in the source population would require a large amount of resources. Cohort studies are generally less efficient than case-control studies for rare events (Answer A is incorrect). Equipoise about this question no longer exists (or is at least in doubt); as a result, a randomized controlled trial is not the best choice of study design for this question (Answer C is incorrect). The term prospective (or retrospective) refers only to the timing of the information and events of the study (Answer D is incorrect).)

A 74-year-old woman who resides in a nursing home is admitted to the hospital with pyelonephritis. On presentation, she is vomiting and confused. Which one of the following is best to recommend for intravenous empiric therapy in this patient? A. Ampicillin/sulbactam. B. Cefepime. C. Cefepime plus vancomycin. D. Vancomycin.

Answer B: Cefepime. (Patients who are admitted to the hospital with pyelonephritis should be initiated on intravenous antibiotics, particularly if they show signs of sepsis or are unable to take oral medications. Even in nursing homes, the primary causes of pyelonephritis are enteric gram-negative rods, although Pseudomonas aeruginosa may be a cause of infection as well, particularly in catheterized patients. Cefepime covers all of these organisms (Answer B is correct). Resistance to ampicillin/sulbactam is high in many enteric gram-negative rods, particularly in long-term care facilities (Answer A is incorrect). S. aureus is not a common cause of pyelonephritis in any setting and does not need to be empirically treated with vancomycin (Answer C and Answer D are incorrect).)

A 69-year old man with coronary artery disease and severe COPD is hospitalized for an acute exacerbation, characterized by dyspnea, sputum volume, and purulence. During his last hospitalization (2 months ago), he was treated with levofloxacin for community-acquired pneumonia. His vital signs today include temperature 99.6^oF (37.5^oC), BP 130/78 mm Hg. HR 92 beats/minute, and RR 20 breaths/minute. Blood and sputum cultures are pending. Which one of the following is most appropriate to initiate in this patient? A. Cefazolin. B. Cefepime. C. Levofloxacin. D. Vancomycin.

Answer B: Cefepime. (The most likely bacterial pathogens encountered in COPD exacerbations include Haemophilus influenza, Moraxella catarrhalis, and Streptococcus pneumoniae However, this patient has risk factors for infection with Pseudomonas aeruginosa—namely, severe COPD, concomitant cardiac disease, age older than 65, and antimicrobial therapy in the past 3 months. Cefepime has coverage against the common pathogens encountered in COPD exacerbations as well as Pseudomonas coverage (Answer B is correct). Cefazolin and vancomycin provide primarily gram-positive activity against Staphylococcus and Streptococcus spp. and no coverage against Pseudomonas (Answer A and Answer D are incorrect). Although levofloxacin would typically be a good choice for treating suspected pseudomonal bacterial infection in hospitalized patients with an acute COPD exacerbation, this patient was recently treated with levofloxacin for community-acquired pneumonia, so local resistance patterns and recent use may not support this quinolone choice (Answer C is incorrect).)

A 39-year-old man (weight 85 kg [187 lbs)) with no significant medical history presents to the ED with suspected community-acquired pneumonia. His symptoms began 2 days ago with shortness of breath, purulent sputum production, and fever. On physical examination, the patient is alert and oriented. Vital signs are temperature 102.70 F (39.30 C), pulse rate 102 beats/minute, respiratory rate 26 breaths'minute, and blood pressure 74/40 mm Hg. Chest radiography shows left lower lobe consolidation consistent with pneumonia. Blocxi and respiratory samples are collected. His WBC is 21.3 x cells/rnrn3 and lactate 4.8 mmolJL. The ED physician places a central venous catheter and initiates aggressive fluid resuscitation with normal saline. After 3 L of normal saline boluses, the patient's mean arterial pressure remains below 60 mmHg, and he has had 30 mL of urine output in 2 hours. Which of the following would be most helpful to guide emergency resuscitation for this A. Pulse rate. B. Central venous pressure (CVP). C. Hematocrit. D. Central venous oxygen saturation (ScvO2).

Answer B: Central venous pressure (CVP). (A stepwise approach in the management of severe sepsis and septic shock aids in determining which information is needed at each stage. Given that central line placement is under way, the next decision is whether additional fluid administration is needed as determined by the CVP (Answer B is correct). This data point needs evaluation before the Hct and ScvO2 can be interpreted for the hemodynamic resuscitation of the patient. The CVP is also the most readily obtainable target for fluid resuscitation (Answer C and Answer D are incorrect). Pulse rate is not a standardized resuscitation end point and is commonly confounded by other clinical factors (Answer A is incorrect).)

A 2-year-old boy (height 2'9", weight 13 kg [28.6 lbs], body surface area 0.56 m2) was rescued from a burning building 6 hours ago. He has second-degree, superficial, and deep partial-thickness burns on his extremities, upper torso, and face. The total body surface area burned is determined to be 40%. Progressive mouth swelling and airway edema suggest inhalation injury, and the patient is placed on mechanical ventilation for airway protection. He is receiving lactated Ringer's solution at 178 mL/hour. Vital signs are heart rate 165 beats/minute, respiratory rate 30 breaths/minute, temperature 98.60 F (370 C), blood pressure 85/65 mm Hg, and SaO2 94%. His urine output is 5-8 mL/hour during the past 2 hours. The most recent laboratory values are sodium 136 mEq/L, K 4 mEq/L, HC03 16 mEq/L, SCr 0.6 mg/dL, and serum glucose 45 mg/dL. Which one of the following would be the best change to this patient's fluid regimen? A. Discontinue lactated Ringer's solution; start dextrose 5%/0.45% sodium chloride at 214 mL/hour. B. Change to lactated Ringer's solution at 156 mL/hour; add dextrose 5%/Iactated Ringer's solution at 48 mL/ C. Keep lactated Ringer's solution at 178 mL/hour; add dextrose 50/0/0.450/0 sodium chloride at 48 mL/hour. D. Keep lactated Ringer's solution at 178 mL/hour; add albumin 5% at 10 mL/hour.

Answer B: Change to lactated Ringer's solution at 156 mL/hour; add dextrose 5%/lactated Ringer's solution at 48 mL/ hour. (This patient's current fluid resuscitation is lactated Ringer's at 178 which is calculated using the Parkland formula for resuscitation volume (i.e., 130 mlJhour for the first 8 hours) plus maintenance volume of 48 mlJhour. However, the patient's tachycardia, SaO2, HC03-, and urine output, and increased SCr all suggest that he is not well hydrated. The low serum glucose of 45 mg/dL is also noteworthy, suggesting that he is failing to generate enough to meet metabolic demands. Therefore, changing the fluid regimen to lactated Ringer's 156 ml/hour (a 20% increase in the resuscitation volume rate) and changing the maintenance component from lactated Ringer's to Dextrose 5%/lactated Ringer's at 48 mL/hour is the best option (Answer B is correct). Changing the regimen to dextrose 5%/0.45% sodium chloride at 214 mlJhour is not appropriate, while it is a 200/0 increase in total fluid rate, it changes all the volume to a dextrose-containing, non-isotonic fluid (Answer A is incorrect) The option of maintaining lactated Ringer's solution at 178 mL/hour and adding dextrose 50/0/0.450/0 scxlium chloride at 48 mL/hour is not preferred because the increase in the resuscitation volume is in excess of 200/0 and while dextrose is added, it is delivered in a non-isotonic solution and would not be the best option to meet the patient's volume needs (Answer C is incorrect). The regimen in answer D is not optimal, because the total fluid rate has not significantly changed, and the addition of albumin at 6 hours postburn is generally not recommended because of massive ongoing capillary leak. Albumin is typically added to the regimen if the patient's serum albumin is low (e.g., less than 2 g/dL) at 18 hours or longer postburn (Answer D is incorrect).)

The Anticoagulation Practice-Based Research Network (PBRN) has 50 members from 30 clinics across 10 states. Members of the Anticoagulation PBRN include pharmacists, nurses, and physicians who provide medical management for patients taking anticoagulants. The Anticoagulation PBRN is launching a project to identify the best method of warfarin management to reach therapeutic targets by different health care professionals (e.g., pharmacists, nurses, physicians). Which one of the following would be the best type of design for this study? A. Longitudinal. B. Cluster randomized. C. Randomized controlled. D. Epidemiologic.

Answer B: Cluster randomized. (The cluster randomized design is best for assessing clusters of patients by clusters of clinicians (e.g., pharmacists, nurses, physicians) from multiple sites through the Anticoagulation PBRN (Answer B is correct). In cluster randomized trials, groups of individuals rather than individuals are randomized to different interventions. Cluster randomized trials are often used to evaluate the group effect of an intervention or to avoid "contamination" across interventions when trial participants are managed within the same setting. Longitudinal studies may or may not apply randomization; without randomization, the study findings will not reach a high evidence level (Answer A is incorrect). In addition, longitudinal studies and epidemiologic studies do not apply randomization or consider potential variances among patients and clinicians (Answer D is incorrect). Traditional randomized controlled studies are ideal for efficacy trials, but not for effectiveness studies like the proposed study (Answer C is incorrect).)

A 6-year-old boy with no known drug allergies presents with an itchy, red left eye with watery discharge. On ocular examination, the cornea appears intact without any ulceration. The patient also has a sore throat and a temperature of 100.40F (380C). Which one of the following is the most appropriate course of treatment for this patient? A. Erythromycin eye ointment four times/day for 7 days. B. Cold compresses and artificial tears as needed. C. Ciprofloxacin eyedrops four times/day for 10 days. D. Trifluridine eyedrops nine times/day for 7 days.

Answer B: Cold compresses and artificial tears as needed. (Because this patient has red, itchy eyes with watery discharge, he likely has conjunctivitis. Discharge that is watery rather than mucoid points to a viral cause. In addition, the constellation of symptoms (sore throat, fever, and conjunctivitis) suggests that pharyngoconjunctival fever is present, which is likely caused by an adenovirus. For viral conjunctivitis, symptomatic treatment with cold compresses and artificial tears to maintain lubrication is all that is typically required (Answer B is correct). Use of erythromycin ointment has been proposed to prevent possible bacterial superinfection, but it is not usually recommended because there is no strong evidence to support this indication (Answer A is incorrect). Given that the clinical picture points strongly to a viral pathogen, antibiotic eyedrops such as ciprofloxacin will not be beneficial (Answer C is incorrect). Likewise, no evidence supports the use of topical antivirals such as trifluridine (Answer D is incorrect).)

An 8-year-old girl with SCD has been undergoing chronic transfusion for more than 2 years and has received a cumulative packed RBC total of 140 mL/kg. Her serum ferritin concentration is 1700 ng/mL. She has a history of severe renal impairment (SCr 2 times the normal age limit). Her parents have difficulty making appointments for her because the family lives in a rural community. Which of the following is the most appropriate therapy recommendation for this patient? A. Deferoxamine. B. Deferasirox. C. Dimercaptosuccinic acid. D. Hydroxyurea.

Answer B: Deferasirox. (This patient's current laboratory values of RBC cumulative dose and serum ferritin indicate the need for iron chelation therapy. Hydroxyurea increases Hgb F levels and would not address her iron chelation needs (Answer C is incorrect). Dimercaptosuccinic acid is a lead chelating agent (Answer D is incorrect). Deferoxamine is a parenteral iron-chelating agent that binds to trivalent ferric ions to form the renally cleared ferrioxamine. Severe renal impairment is a contraindication for use of this agent (Answer A is incorrect). Also important to consider is the patient's ability to adhere to a prescribed regimen. Deferoxamine, administered parenterally with required infusions weekly, would be a challenge for this patient because her family lives in a rural area. Deferasirox is an oral agent and can be used in patients with renal impairment with dose adjustment (Answer B is correct).)

A 54-year-old man (height 5'5", weight 78 kg [172 lb]) has diabetes and end-stage renal disease; he is treated with hemodialysis on Mondays, Wednesdays, and Fridays (access by tunneled catheter). Today, he presents to the hospital with fevers and chills and is admitted to the intensive care unit. His home drugs include insulin glargine, insulin aspart, multivitamin, calcitriol, sevelamer, and furosemide. Blood cultures are 2 of 2 positive for Candida, with the species as yet unidentified. His physician prescribes caspofungin 50 mg intravenously daily. Three days later, the patient remains stable with vital signs of BP 110/72 mm Hg, HR 98 beats/minute, RR 23 breaths/minute, 02 saturation 97%, and Tmax 39.30C. Cultures are as follows: ------------------------------------------------------------------- Day 1: blood, peripheral — Candida krusei (2/2 bottles) (susceptibilities pending); blood, dialysis catheter — C. krusei (2/2 bottles) (susceptibilities pending); Day 2: blood, peripheral — yeast (2/2 bottles); blood, dialysis catheter — yeast (2/2 bottles); Day 3: blood, peripheral — yeast (2/2 bottles); blood, dialysis catheter — yeast (2/2 bottles). ------------------------------------------------------------------ Which one of the following is best to recommend for this patient? A. Continue caspofungin with no changes. B. Continue caspofungin and recommend removal of dialysis catheter and placement of a temporary line for dialysis. C. Change caspofungin to liposomal amphotericin B 200 mg intravenously daily and recommend removal of dialysis catheter and placement of a temporary line for dialysis. D. Continue caspofungin and recommend initiation of a voriconazole catheter lock solution.

Answer B: Continue caspofungin and recommend removal of dialysis catheter and placement of a temporary line for dialysis. (The best antifungal therapy for C. krusei is an echinocandin, so changing to liposomal amphotericin B is not a good option, especially because liposomal amphotericin B has reduced sensitivity, and a higher dose (5 mg/kg) would be needed (Answer C is incorrect). Line-related candidemia infections require removal of the catheter for complete resolution of the infection, so this should occur first (Answer B is correct). Continuing caspofungin is appropriate therapy; however, without a dialysis line catheter change, the patient is unlikely to clear the infection (Answer A is incorrect). Using only a voriconazole catheter lock solution would not be as effective as removing the current dialysis catheter (Answer D is incorrect).)

A patient with bipolar disorder presents with symptoms that include feelings of hopelessness, worthlessness, psychomotor slowing, and marked anxiety. He has been adherent to lithium therapy, which has decreased the intensity and frequency of cycling episodes of mania. His lithium serum concentration is 1.1 mEq/L. He has no other significant medical history. Which one of the following would be best to recommend for this patient? A. Discontinue lithium and initiate aripiprazole. B. Continue lithium and augment with quetiapine. C. Continue lithium and augment with ziprasidone. D. Discontinue lithium and initiate venlafaxine.

Answer B: Continue lithium and augment with quetiapine. (The patient appears to be describing the depressive symptoms of bipolar depression. Depressive symptoms are more common than manic symptoms as a manifestation of bipolar disorder. Augmentation with quetiapine is the best option (Answer B is correct). Quetiapine has a labeled indication for the acute treatment of bipolar depression. Initiating aripiprazole or augmenting with ziprasidone is not the best choice; neither agent has a labeled indication for bipolar depression. Aripiprazole lacks data to support its use as monotherapy, and ziprasidone lacks data to support its use as monotherapy or adjunctive therapy (Answer A and Answer C are incorrect). Use of antidepressants as monotherapy is not indicated in bipolar disorder. In addition, use of a tricyclic antidepressant and venlafaxine has been associated with higher rates of switch to mania or hypomania compared with use of other antidepressants (Answer D is incorrect).)

A 9-year-old girl is evaluated at your clinic for asthma. A pulmonary function test shows her FEV(1) (forced expiratory volume in I second) to be 68% of predicted. The patient and her mother report that she has asthma symptoms throughout the day and night and that she uses her albuterol inhaler two or three times/day. During the past 2 months, she has completed two courses of oral steroid "burst" therapy. Her current asthma regimen is albuterol as needed and low-dose fluticasone twice daily The physician is prescribing a steroid burst for 5 days. Which one of the following is best to recommend for this patient's long-term inhaled therapy? A. Continue low-dose fluticasone and albuterol. B. Continue low-dose fluticasone, add salmeterol, and continue albuterol. C. Increase fluticasone to high dose and continue albuterol. D. Increase fluticasone to high dose, add salmeterol, and continue albuterol.

Answer B: Continue low-dose fluticasone, add salmeterol, and continue albuterol. (This patient's asthma is poorly controlled, and therapy should be stepped up by adding a long-acting ß-agonist (LABA). The patient is currently at step 2 therapy and needs to step up one or two steps because of her poor asthma control (Answer B is correct). Continuing low-dose fluticasone and albuterol is inappropriate because it does not change current therapy for a patient with poorly controlled asthma (Answer A is incorrect). High-dose inhaled corticosteroids (ICS) are used during step 5 therapy, which would be a three-step increase for this patient (Answer C is incorrect). Increasing to high-dose fluticasone, adding salmeterol, and continuing albuterol is also an increase in three steps to step 5 therapy (Answer D is incorrect). Currently recommended step 3 therapy regimens include low-dose ICS plus LABA (in patients 5 years or older) or medium-dose ICS.)

A 39-year-old man has type I bipolar disorder (diagnosed at age 22), with the last episode being mania with psychotic features. For the past 10 years, he has been stable on oxcarbazepine 300 mg every morning and 600 mg at bedtime, venlafaxine XR 150 mg every morning, and olanzapine 30 mg at bedtime. Although currently in full remission, he scheduled an appointment today to discuss treatment alternatives. He is concerned about the metabolic issues associated with olanzapine and states he has gained 130 lb during the past decade. He fears changing his therapy because of a rebound of manic and psychotic symptoms. His medical history includes hypertension, hyperlipidemia, type 2 diabetes mellitus, sleep apnea, and morbid obesity. He has been treated in the past with risperidone, lithium, divalproex, and ziprasidone. Which one of the following is best for this patient? A. Continue oxcarbazepine and venlafaxine; switch to quetiapine. B. Continue oxcarbazepine and venlafaxine; switch to aripiprazole. C. Discontinue oxcarbazepine, venlafaxine, and olanzapine; switch to divalproex and risperidone. D. Continue oxcarbazepine, venlafaxine, and olanzapine; augment with topiramate.

Answer B: Continue oxcarbazepine and venlafaxine; switch to aripiprazole. (Use of atypical antipsychotics is associated with weight gain, diabetes, and dyslipidemia. The 2004 American Diabetes Association (ADA) Consensus Development Conference concluded that clozapine and olanzapine have an increased effect on weight gain, risk of diabetes, and worsening of the lipid profile compared with atypical antipsychotics. Risperidone and quetiapine also have an increased effect on weight gain; however, the ADA Consensus found the data on diabetes risk and lipid profile inconclusive. Aripiprazole, ziprasidone, and the new atypical antipsychotics are neutral with respect to their metabolic risks. Switching atypical antipsychotic drugs can significantly improve metabolic risks. According to the ADA Consensus, the atypical antipsychotic with the least metabolic risk is aripiprazole (Answer B is correct). Quetiapine also carries a high risk of weight gain and metabolic issues, and the patient's goals might not be met (Answer A is incorrect). Discontinuing the current drug regimen and switching to divalproex and risperidone is also an acceptable combination; however, divalproex and risperidone both have the potential to cause weight gain, and both together might prohibit weight loss (Answer C is incorrect). In theory, augmenting with topiramate would target bipolar disorder and weight loss; unfortunately, evidence of efficacy for either goal is limited (Answer D is incorrect). According to a meta-analysis, the average weight loss with topiramate was between 5.4 kg (12 lb) and 7.2 kg (16 lb), whereas this patient's weight-loss goal is more than 100 lb. To achieve this goal, the best option is to remove the agent most likely exacerbating the problem (olanzapine).)

A 37-year-old woman with epilepsy (weight 65 kg [143 lbs]) has been prescribed carbamazepine for several years. The patient smokes, regularly drinks alcohol, and has difficulty remembering to take her drugs. She has experienced more frequent seizures at home. A friend concerned about her current behavior brings her to the ED. On arrival, she is afebrile and is given intravenous thiamine, benzodiazepine, and phenytoin boluses. She is transferred to the ICU for further care because of continued seizures with no recovery of consciousness. Her laboratory values include sodium 133 mEq/L, Cl 103 mEq/L, SCr 1.1 mg/dL, BG 147 mg/dL, K 3.7 mEq/L, HCO3 23 mEq/L, BUN 6 mg/dL, WBC 10.8 x 103/mm^3, Hgb II g/dL, and platelet count 314,000/mm3. Vital signs: BP 100/50 mmHg, HR 80 beats/minute. Her serum drug screen was negative for the usual toxins. Which one of the following is best to recommend for this patient? A. Rapid intravenous infusion of valproic acid 40 mg/kg. B. Continuous infusion of midazolam at 7 mg/hour after a bolus of 10 mg. C. Continuous infusion of propofol at 30 mcg/kg/min after a bolus of 120 mg. D. Continuous infusion of pentobarbital at 3 mg/kg/hour after a bolus of 10 mg/kg.

Answer B: Continuous infusion of midazolam at 7 mg/hour after a bolus of 10 mg. (Per the 2012 guidelines, after failed attempts to control status epilepticus with bolus therapy (benzodiazepines and phenytoin in this case), treatment recommendations are to use continuous infusion antiepileptics. Because this patient is nonadherent and regularly uses alcohol, she will benefit from general anesthesia with a benzodiazepine for treatment of refractory seizures (Answer B is correct). For refractory status epilepticus in adults, valproic acid may be tried before administering general anesthetic drugs. Valproic acid can be rapidly infused; if it fails, then general anesthesia can be initiated. However, no trials have tested this approach. Her alcohol history also makes this option less attractive (Answer A is incorrect). Propofol may also be used in this patient; however, for suspected ethanol withdrawal, benzodiazepines remain the drugs of choice. The safety profile of Propofol should also be considered with its risk of propofol infusion syndrome and higher risk of hypotension when compared with midazolam (Answer C is incorrect). Finally, the prolonged half-life and the hemodynamic effects of pentobarbital make it less attractive than midazolam (Answer D is incorrect).)

A 48-year-old woman with a history of hypertension, depression, and hot flushes presents to the clinic for her annual checkup. Her current drugs include hydrochlorothiazide 25 mg daily, sertraline 100 mg daily, estradiol 50 mcg plus norethindrone acetate 0.25-mg transdermal patch, and a multivitamin daily. Laboratory values today show vitamin D 35 ng/mL; a chemistry panel is within normal limits. A dual-energy x-ray absorptiometry (DEXA) scan I month ago shows a T-score of spine -2.5, femoral neck -2.7, and total hip -2.3. Which one of the following is best to recommend regarding this patient's bone health? A. Alendronate 35 mg by mouth weekly. B. Denosumab 60 mcg subcutaneously every 6 months. C. Raloxifene 60 mg by mouth daily. D. Zoledronic acid 10 mg intravenous infusion every year.

Answer B: Denosumab 60 mcg subcutaneously every 6 months. (The correct answer is denosumab 60 mcg subcutaneously every 6 months (Answer B is correct). Current American Association of Clinical Endocrinologists guidelines recommend alendronate, risedronate, denosumab, or zoledronic acid as first-line treatment options for osteoporosis, which is indicated by the patient's DEXA values. The 35-mg weekly alendronate dose is for the prevention, not treatment, of osteoporosis (Answer A is incorrect). Raloxifene is a second-line agent that could worsen the patient's hot flushes (Answer C is incorrect). The approved zoledronic acid dose is 5 mg by intravenous infusion annually, not 10 mg as listed here (Answer D is incorrect).)

A 72-year-old woman asks whether she still needs alendronate. After taking this agent for 5 years, she admits discontinuing it 2 months ago. She still takes calcium 600 mg and vitamin D 400 units of combination product with food twice daily. The patient has no history of fracture; her FRAX scores are 1.2% for hip fracture and 14% for any major osteoporotic fracture. Results of her DXA are as follows: Region || Current T-Score || Baseline T-Score ------------------------------------------------------- L1-L4 || -2.2 || -2.8 ------------------------------------------------------- Femoral Neck || -1.9 || -2.6 ------------------------------------------------------- Total Hip || -1.8 || -2.4 Which one of the following is best to recommend for this patient? A. Reinitiate alendronate. B. Do not reinitiate alendronate. C. Switch to zoledronic acid. D. Switch to denosumab.

Answer B: Do not reinitiate alendronate. (Because this patient has low bone mass, has been on therapy for 5 years, and is at low risk of an osteoporotic fracture, she appears to be a candidate for bisphosphonate discontinuation (Answer B is correct). In the FLEX trial 2010 and 2012 reanalyses, patients with low bone mass who discontinued alendronate therapy had no significant difference in vertebral or nonvertebral fracture risk compared with those who continued therapy. In addition, the AACE and the FDA support discontinuation after 3—5 years of therapy. The AACE guidelines recommend reinitiating therapy in 1—2 years only in patients considered at high risk who have initially received 10 years of bisphosphonate therapy (Answer A is incorrect). This patient's T-scores suggest she has low bone mass. Fracture and bone mineral density data are lacking to support a switch from a bisphosphonate to other therapies in patients with low bone mass after initial osteoporosis therapy (Answer C and Answer D are incorrect).)

A 73-year-old woman presents to the ED with extreme dyspnea on exertion when walking from room to room at home. She has experienced lethargy and a gradual reduction in exercise tolerance for the past 3 months. She also admits feeling cold most of the day. She has been adherent to her dietary restrictions and drugs. Her weight has been stable at 85 kg during this time. She has a history of alcohol-induced nonischemic cardiomyopathy (LVEF 15%). Vital signs on admission include BP 82/56 mm Hg (without orthostasis), HR 95 beats/minute, and respiratory rate 18 breaths/minute. On physical examination, she has a stable 6-cm J V D, no pulmonary edema, no ascites, and no lower extremity edema. Laboratory analysis reveals Na 135 mEq/L, K 4.1 mEq/L, Cl 99 mEq/L,BUN 42 mg/dL (baseline 34 mg/dL), SCr 1.8 mg/dL (baseline 1.4 mg/dL), and Mg 1.8 mEq/L. She has been stable on her oral HF drugs, which include metoprolol succinate 25 mg twice daily, enalapril 5 mg twice daily, and spironolactone 25 mg once daily. Which one of the following medication changes would most likely benefit this patient? A. Milrinone 0.5 mcg/kg/minute infusion. B. Dobutamine 5 mcg/kg/minute infusion. C. Nitroglycerin 10 mcg/minute infusion. D. Furosemide 40 mg intravenous bolus.

Answer B: Dobutamine 5 mcg/kg/minute infusion. (This patient has progressive worsening of HF leading to acute decompensation characterized by a "cold and dry" profile with symptomatic hypotension; therefore, she is hypoperfused. Because she has no signs or symptoms of being "wet," a loop diuretic would not be an appropriate choice because furosemide could deplete her intravascular volume (Answer D is incorrect). Nitroglycerin would not be an appropriate option at this time given her BP level (Answer C is incorrect). Her worsening kidney function, reduced exercise tolerance, and feeling of being cold are likely caused by the reduction in cardiac output; thus, inotropic therapy is warranted. Because of its mechanism of action, milrinone may be chosen over dobutamine in patients on stable ß-blockers, however, it has an increased risk of hypotension, and its effects may be prolonged in patients with kidney dysfunction. This patient is already hypotensive and has kidney dysfunction, and the milrinone dose is not renally adjusted (Answer A is incorrect). Renal adjustment for this patient's CrCl of around 37 mL/minute/l.73m2 would be 0.375 mcg/kg/minute. At this time, dobutamine would be the best choice to improve cardiac output and limit the risk of drug-induced hypotension, despite the patient's being on a ß-blocker at home (Answer B is correct).)

A 52-year-old man presents to his primary care provider with symptoms of ED. He states that his symptoms have been mild for years but that they are now more bothersome. His scores on the SHIM (Sexual Health Inventory for Men) are in the mild to moderate range of severity. His medical history includes osteoarthritis; his social history is notable for smoking (2 packs/day) and social alcohol use. The patient's home drugs include acetaminophen every 6 hours as needed for joint pain and glucosamine/chondroitin once daily. Which one of the following laboratory tests would be most helpful in further evaluating this patient's ED? A. SCr level. B. Fasting lipid panel. C. Free thyroxine level. D. Follicle-stimulating hormone level.

Answer B: Fasting lipid panel. (Evaluating a fasting lipid panel at least every 5 years is appropriate for any adult as a screening mechanism for cardiovascular disease (CVD). Because ED can present as a symptom of CVD, screening for and treating dyslipidemia would be helpful for this patient as part of the ED workup (Answer B is correct). Checking an SCr might be appropriate, but the patient has no risk factors for an elevated SCr (Answer A is incorrect). Evaluating a thyroid-stimulating hormone level, but not a free thyroxine level, is appropriate in the workup of ED (Answer C in incorrect). Finally, a follicle-stimulating hormone would not be helpful in assessing a patient with ED (Answer D is incorrect).)

You have been asked to design a study in which the expected prevalence of a drug effect is very low. You decide to compare the presence or absence of the effect in individuals who took the drug with individuals who did not. Which one of the following statistical tests would be the most appropriate to use? A. Z-test. B. Fisher exact test. C. t-test. D. Analysis of variance (ANOVA).

Answer B: Fisher exact test. (The Fisher exact test is used for comparing categorical data and is appropriate when the expected value of at least one cell of a 2 x 2 table is less than 5. Because the expected prevalence of the disease in question is very low, this is the appropriate test (Answer B is correct). The Z-test is used when the data are normally distributed, the variance is known, and the data are continuous; in this case, the data are categorical, not continuous (Answer A is incorrect). The t-test is used when data are continuous, are relatively normally distributed, and the variance is unknown (Answer C is incorrect). The ANOVA is a generalization of the t-test for more than two groups (Answer D is incorrect).)

A 49-year-old woman with a 10-year history of type 2 diabetes mellitus presents to the clinic with burning pain accompanied by numbness and tingling in her feet bilaterally. She reports that the pain is constant and rates it as 6/10. Monofilament testing shows that she has decreased sensation in both feet. Which one of the following would best manage this patient's pain? A. Acetaminophen 500 mg orally every 4 hours. B. Gabapentin 100 mg orally three times daily. C. Lidoderm 5% topical patch applied every 48 hours. D. Oxycodone 5 mg orally every 4 hours.

Answer B: Gabapentin 100 mg orally three times daily. (Anticonvulsants such as gabapentin are commonly used for the management of diabetic peripheral neuropathy (Answer B is correct). Lidocaine patches are also effective for localized neuropathic pain; however, they ars applied daily for 12 hours and then removed for 12 hours, and they may be difficult to apply to the feet, even when cut to size (Answer C is incorrect). Acetaminophen is not effective for neuropathic pain (Answer A is incorrect). Oxycodone and other opioids should be considered for patients with severe neuropathic pain that has not been relieved by other analgesics (Answer D is incorrect).)

In a regression model, individuals with cancer had an odds ratio of 1.5 (95% CI, 1.2—2.0) compared with individuals without cancer. Which statement is the best interpretation of these results? A. Having cancer increased the likelihood that the person experienced the outcome by 150%. B. Having cancer increased the likelihood that the person experienced the outcome by C. Having cancer decreased the likelihood that the person experienced the outcome by 150%. D. The estimated odds ratio did not reflect a statistically significant result.

Answer B: Having cancer increased the likelihood that the person experienced the outcome by 20%—100%. (The data are consistent with odds ratios from 1.2 to 2.0, as shown in the 95% CI (Answer B is correct). An odds ratio of 1.2 is interpreted as a 20% increase in the likelihood that exposed individuals had the outcome, whereas an odds ratio of 2.0 is interpreted as a 100% increase in the likelihood that exposed individuals had the outcome compared with unexposed individuals. An odds ratio of 1.5 is not interpreted as a 150% increase in the likelihood of the outcome (Answer A is incorrect). An odds ratio of 2.5 would represent a 150% increase, or an increase of 2.5 times the baseline odds. Odds ratios above 1.0 are interpreted as increases in the likelihood of experiencing the outcome (Answer C is incorrect). Odds ratios below 1.0 are interpreted as a decrease in the likelihood of experiencing the outcome. The CI for the odds ratio did not cross 1.0, so the results are interpreted as statistically significant (Answer D is incorrect).)

A study examines the effect of pharmacist involvement on a general medical team. One reported finding is teams that include pharmacists make fewer errors. The estimates are presented as mean errors plus or minus the standard error of the mean (SEM). Which one of the following best describes the researchers' use of SEM as the measure of variability? A. Appropriate because it estimates the spread of the data. B. Inappropriate because it estimates the variability of means estimated from repeated iterations of the study, not of the individual observations. C. Appropriate because it overestimates the variability of the data. As a result, readers can comfortably believe that the authors' findings are even more dramatic than reported. D. Inappropriate because it estimates the variability of the individual observations, not of the means estimated from repeated iterations of the study.

Answer B: Inappropriate because it estimates the variability of means estimated from repeated iterations of the study, not of the individual observations. (The SEM estimates variability of the means from multiple iterations of the study (Answer B is correct). The SEM does not estimate the spread of the data (Answer A is incorrect). In addition, the SEM does not overestimate the data variability. Because the SEM is calculated by dividing the standard deviation (SD) by the square root of the number of observations, this measure underestimates the variability in the data (Answer C is incorrect). The SEM estimates the variability of the means derived from reiterations of the study, not of the individual observations Answer D is incorrect).)

A 36-year-old man with a medical history significant for HIV infection (diagnosed in 2002) presents to the ED with difficulty breathing and chest pain that has worsened during the past few weeks. His home drugs include tenofovir 300 mg daily, emtricitabine 200 mg daily, raltegravir 400 mg twice daily, and ranitidine 75 mg once daily. Laboratory values today show HIV RNA less than 50 c/mL, CD4+ count 404 cells/mm3, CD4 29%, WBC 10.5 x 10^3 cells/mm3, platelet count 245,000/mm3, and Hgb 9.9 g/dL. A chemistry panel is within normal limits. Chest radiography reveals patchy consolidation within the lung parenchyma and a darkened lesion within the right lung, suggestive of cavitation. Sputum samples (3 of 3) are positive for acid-fast bacilli; culture and susceptibility are pending. The patient receives a diagnosis of TB, and treatment with isoniazid plus rifampin plus pyrazinamide plus ethambutol is planned. With the initiation of anti-TB therapy, which of the following recommendations should be made regarding this patient's current antiretroviral regimen? A. Decrease raltegravir to 400 mg daily. B. Increase raltegravir to 800 mg twice daily. C. Replace raltegravir with ritonavir 100 mg twice daily. D. Schedule raltegravir I hour before or 2 hours after the rifampin dose.

Answer B: Increase raltegravir to 800 mg twice daily. (The raltegravir dose should be increased to 800 mg twice daily while the patient is taking rifampin therapy to account for the induction and subsequent significant reduction in raltegravir AUC (Answer B is incorrect). Decreasing the dose would only decrease the patient's exposure to the drug and undermine his antiretroviral therapy (Answer A is incorrect). The patient's HIV is well controlled, and a change in antiretroviral therapy is not warranted. In addition, the dose is too low, and ritonavir is subject to the same type of interaction with rifampin as is raltegravir. Ritonavir labeling recommends that alternative antimycobacterial agents such as rifabutin be considered in patients with HIV infection who are treated with ritonavir. However, some experts, as well as the CDC (Centers for Disease Control and Prevention) guidelines, suggest that the usual antituberculous dosages of rifampin can be used with ritonavir 400—600 mg twice daily in combination with one or more NRTIs (nucleoside reverse transcriptase inhibitors). However, rifampin should not be used with low-dose ritonavir (i.e., 100 mg twice daily given as a pharmacokinetic booster of other protease inhibitors). Replacing raltegravir with ritonavir 100 mg twice daily would not be optimal (Answer C is incorrect). Scheduling administration around the rifampin dose does not mitigate the interaction (Answer D is incorrect).)

A 34-year-old woman is admitted to the hospital for a mastectomy after receiving a diagnosis of advanced-stage breast cancer. She has no other significant medical history. The patient underwent successful surgery on the morning of admission, and the following day, she was initiated on venous thromboembolism (VT E) prophylaxis with low-dose UFH twice daily. On day 3 of hospitalization, routine monitoring revealed that her platelet count had dropped to 145,000/mm3 from a baseline of 220,000/mm3. She has no signs or symptoms of VT E. Which one of the following is best to recommend for this patient? A. Discontinue low-dose UFH, initiate argatroban, and order a heparin platelet factor 4 antibody test. B. Increase the low-dose UFH therapy to three times daily. C. Discontinue low-dose UFH; then order a heparin/platelet factor 4 antibody test and wait for results before initiating anticoagulation. D. Continue her current low-dose UFH regimen.

Answer B: Increase the low-dose UFH therapy to three times daily. (The VT E prophylaxis options for patients with cancer undergoing surgery include low-dose UFH given three times daily, a LMWH, or fondaparinux. This patient was receiving UFH only twice a day and would need more aggressive prophylaxis (Answer B is correct). The suggestion of heparin-induced thrombocytopenia (HIT) in this patient is low. The timing of thrombocytopenia onset is not consistent with HIT (less than 5 days from exposure with no previous exposure), no signs of thrombosis are evident, the platelet count did not decrease by more than 50%, and an alternative cause of thrombocytopenia, advanced-stage cancer, is present (Answer A is incorrect). Heparin should not be interrupted if the suggestion of HIT is low, and if the suggestion of HIT is high enough, alternative anticoagulation should be initiated immediately (Answer C is incorrect). If low-dose UFH is used for V TE prophylaxis in patients with cancer undergoing surgery, it must be administered three times daily (Answer D is incorrect).)

You are preparing a pharmacoeconomic analysis of the costs incurred in caring for patients with congestive heart failure. Which one of the following cost categories best typifies lost productivity? A. Direct nonmedical cost. B. Indirect cost. C. Intangible cost. D. Opportunity cost.

Answer B: Indirect cost. (Indirect costs are those resulting from morbidity and mortality, including lost productivity or premature death. These costs are valued as real money that is not directly paid for the treatment of an illness or disease (Answer B is correct). Direct nonmedical costs are costs for nonmedical services that are the result of illness or disease but do not involve purchasing medical services (Answer A is incorrect). Pain, suffering, or both related to a medical condition represent intangible costs (Answer C is incorrect). Opportunity costs represent the money spent on one resource that now cannot be spent for another purpose (i.e., the next best use of those resources is forgone) (Answer D is incorrect).)

A colleague tells you about a poster presented at an annual national pharmacy meeting. The poster addressed the advantages and disadvantages of the Baxter intravenous pump specifically for pediatric patients in the intensive care unit. Which one of the following would be the best source for finding this poster? A. Iowa Drug Information Service. B. International Pharmaceutical Abstracts. C. MEDLINE. D. Embase.

Answer B: International Pharmaceutical Abstracts. (International Pharmaceutical Abstracts contains information specific to pharmacy as well as information presented at selected national pharmacy scientific meetings. Because this question requires information specific to an intravenous pump that was presented at a national meeting, the best source would contain information specific to pharmacy (Answer B is correct). MEDLINE does not routinely index meeting abstracts (Answer C is incorrect). Embase is a database similar to MEDLINE with more international coverage abstracts and is a good source for agents available in Europe because generic names are now indexed according to the name preferred by the FDA and the EMEA (European Medicines Agency) (Answer D is incorrect). Iowa Drug Information Service contains information on medications but contains only journal articles chosen by the editorial staff from the indexed journals. Information is limited to articles that pertain to drug use in humans, so meeting abstracts are not routinely included (Answer A is incorrect).)

A 32-year-old woman has a new diagnosis of relapsing-remitting multiple sclerosis (MS). She has not taken any previous MS therapy and presents to the clinic today to discuss her therapy options. The medical resident would like to consider a trial of glatiramer acetate. Which one of the following statements best describes the use of glatiramer acetate as a first-line agent in this patient? A. It is not a first-line therapy for relapsing-remitting MS. B. It is an appropriate first-line therapy option for this patient. C. It is an appropriate first-line therapy option, but it should be given concurrently with interferon beta-1b. D. It may be used in this patient only after a trial of dalfampridine.

Answer B: It is an appropriate first-line therapy option for this patient. (Glatiramer acetate is a viable, cost-effective, first-line, single-therapy agent for a relapsing-remitting condition (Answer B is correct; Answer A and Answer C are incorrect). Dalfampridine is used to improve walking and mobility issues related to MS, and there is no requirement for its administration before glatiramer; however, the two drugs may be used together (Answer D is incorrect).)

An adult is brought to the ED in respiratory distress from a severe asthma attack with underlying pneumonia. The patient requires intubation. Vital signs include the following: blood pressure 80/48 mm Hg, heart rate 98 beats/minute, and temperature 102.10 F (38.90 C). Which one of the following is the best choice for induction in this patient? A. Etomidate. B, Ketamine. C. Propofol. D. Sodium thiopental.

Answer B: Ketamine. (Ketamine is preferred in this setting on the basis of its bronchodilatory properties and because it does not contribute to increased hypotension (Answer B is correct). Propofol is well known to cause hypotension and should be avoided (Answer C is correct). Etomidate is an option and does not contribute to hypotension; however, it does not have bronchodilatory properties; therefore ketamine is preferred in this patient (Answer A is incorrect). Sodium thiopental also causes hypotension and should be avoided (Answer D is incorrect).)

A 45-year-old Asian man with a diagnosis of alcohol dependence is admitted to the emergency department with chest pains. He states that both of his parents were alcohol-dependent and that he started drinking heavily when he was age 14. Which one of the following genetic polymorphisms related to alcoholism is this patient most likely to have? A. 2D*4 polymorphism of cytochrome P450 (CYP) 2D6. B. LL alleles of serotonin transporter gene promoter polymorphism (5'-HTTLPR). C. μ-Opiate receptor gene (OPRMI) polymorphism, Asn40ASP. D. Aldehyde dehydrogenase (ALDH)2*2 polymorphism.

Answer B: LL alleles of serotonin transporter gene promoter polymorphism (5'-HTTLPR). (The 5'-HTTLPR serotonin transporter is putatively associated with early-onset alcohol dependence (Answer B is correct). The 2D*4 polymorphism of CYP2D6 is related to the conversion of opioids to an active metabolite, morphine (Answer A is incorrect). The OPRMI polymorphism, Asn40ASP, influences response to naltrexone (Answer C is incorrect). The ALDH2*2 allele variation is the best-characterized genetic factor that protects against the development of alcohol dependence. This polymorphism results in a slower metabolism of acetaldehyde and allows it to accumulate, causing hypotension, flushing, nausea, tachycardia, and vomiting. If the patient is alcohol-dependent, he probably does not have this polymorphism (Answer D is incorrect).)

A 71-year-old woman (weight 68 kg [150 lb]) has developed spells in which she loses consciousness and has movements of her mouth. These episodes last for less than 1 minute and occur several times a week. She has a medical history of atrial fibrillation. Her current drugs include warfarin that is dose adjusted to maintain an INR of 2—3, lisinopril 5 mg/day, and hydrochlorothiazide 25 mg/day. Her vital signs, laboratory values, and other test results are BP 135/85 mm Hg; HR 82 beats/minute; Na 138 mEq/L; K 4.0 mEq/L; Cl 102 mEq/L; CO2 23 mEq/L; SCr I mg/dL; BUN 12 mg/dL; and INR 2.3. Her electroencephalography results are consistent with a diagnosis of partial seizures. Which one of the following antiepileptic drugs (AEDs) would be best for this patient? A. Carbamazepine. B. Lamotrigine. C. Oxcarbazepine. D. Phenytoin.

Answer B: Lamotrigine. (All the agents listed are for the treatment of partial seizures. Of the available and lamotrigine are only selections that have been prospectively studied in older adults. Carbamazepine and phenytoin are not optimal for this patient because they are to induce the metabolism of warfarin (Answer A and D are incorrect). Oxcarbazepine is not a good choice because of an increased risk of hyponatremia in elderly patients and the for an interaction warfarin similar to that with carbamazepine and (Answer C is incorrect). In addition, in a retrospective study, oxcarbazepine was consistently less well tolerated than AEDs- Thus, lamotrigine is the best option this patient (Answer B is correct).

A 30-year-old nulliparous woman (height 5'5", weight 115 kg [253.5 lb]) presents to the clinic for contraceptive care. She does not smoke and drinks alcohol once or twice weekly. Today, her blood pressure is 125/85 mm Hg. She is not interested in becoming pregnant in the near future. She has a history of nonadherence to daily contraceptive regimens and is interested in finding a contraceptive option that will reduce her menorrhagia. Which one of the following is the most appropriate contraceptive choice for this patient? A. Low-dose combined oral contraceptive. B. Levonorgestrel intrauterine device (IUD). C. Combined hormonal contraceptive transdermal patch. D. Progestin-only oral contraceptive.

Answer B: Levonorgestrel IUD. (The levonorgestrel IUD is the best choice for this patient, who is obese (Answer B is correct). The IUD will meet her contraceptive efficacy and safety needs and provide the noncontraceptive benefit of reducing menorrhagia with time. Contraceptives containing estrogen may increase her risk of developing venous thromboembolism (VT E) because of her obesity and are not the most appropriate options because she is nonadherent to daily contraceptive regimens (Answer A and Answer C are incorrect). The progestin-only oral contraceptive may be a safe option in obese women; however, it will not reduce this woman's menorrhagia, and she may experience decreased efficacy because she has been nonadherent to daily tablets. Progestin-only oral contraceptives must be taken within a 3-hour time interval to maintain efficacy (Answer D is incorrect).)Answer B: Levonorgestrel IUD. (The levonorgestrel IUD is the best choice for this patient, who is obese (Answer B is correct). The IUD will meet her contraceptive efficacy and safety needs and provide the noncontraceptive benefit of reducing menorrhagia with time. Contraceptives containing estrogen may increase her risk of developing venous thromboembolism (VT E) because of her obesity and are not the most appropriate options because she is nonadherent to daily contraceptive regimens (Answer A and Answer C are incorrect). The progestin-only oral contraceptive may be a safe option in obese women; however, it will not reduce this woman's menorrhagia, and she may experience decreased efficacy because she has been nonadherent to daily tablets. Progestin-only oral contraceptives must be taken within a 3-hour time interval to maintain efficacy (Answer D is incorrect).)

An 8-week-old infant girl (weight 7 kg [15.5 lb]) is admitted to the hospital with a temperature of 102.5^oF (39.20C), irritability, and poor feeding. She has a hydrocephalus that necessitated the placement of a ventriculoperitoneal shunt at 4 weeks. Analysis of her CSF shows WBC 1.744 x 103 cells/mm3, neutrophils 90%, lymphocytes 10%, protein 455 mg/dL, and glucose less than 20 mg/dL. The patient's SCr is 0.9 mg/dL. She is initiated on vancomycin and meropenem. Culture of CSF is positive for vancomycin-resistant Enterococcus faecalis sensitive to linezolid, daptomycin, and tigecycline. Which one of the following is best to recommend for this patient? A. Daptomycin 10 mg/kg intravenously every 24 hours. B. Linezolid 10 mg/kg intravenously every 8 hours. C. Quinupristin/dalfopristin 7.5 mg/kg intravenously every 8 hours. D. Tigecycline 2 mg/kg intravenously every 8 hours.

Answer B: Linezolid 10 mg/kg intravenously every 8 hours. (Linezolid intravenously penetrates the CSF, and reports of its successful use are documented in pediatric patients (Answer B is correct). Daptomycin intravenously does not achieve high concentrations in the CSF, and use in children with ventriculitis is limited (Answer A is incorrect), although case reports in adult patients are documented. Quinupristin/dalfopristin does not have activity against E. faecalis (Answer C is incorrect). Tigecycline is not approved for use in patients younger than 18 years. Although a single case has been reported, tigecycline dosing is not well established in children, and CNS penetration is limited. This agent should be used only as a last resort (Answer D is incorrect).)

A nurse from the emergency department calls to ask why the preferred administration route for parenteral promethazine is intramuscular rather than intravenous. She knows the drug should not be administered through a peripheral vein. Which one of the following is the best MEDLINE search strategy to limit erroneous results? A. MeSH terms injections, intravenous AND MeSH term promethazine. B. MeSH terms injections, intravenous, with a subheading, "adverse event," AND MeSH term promethazine. C. MeSH terms injections, intravenous, with a subheading, "adverse event," AND key word "promethazine." D. Key word "intravenous injections" AND key word "promethazine."

Answer B: MeSH terms injections, intravenous, with a subheading, "adverse event," AND MeSH term promethazine. (Use of Medical Subject Headings (MeSH) terms together with a subheading will provide the most accurate and efficient search strategy (Answer B is correct). Although use of MeSH terms without the subheading will provide sufficient information, use of a subheading in this case will result in a more effective search strategy (Answer A and Answer C are incorrect). A key word search will provide more erroneous information than a directed MeSH search (Answer D is incorrect).)

A 37-year-old woman (weight 59 kg [130 lb]) presents to the ambulatory care clinic with a 7-week history of new abdominal cramping and two to four loose bloody stools per day. She has no known allergies and reports taking only acetaminophen as needed for headaches. Colonoscopy findings are consistent with ulcerative colitis (UC) with superficial inflammation that is continuous from the rectum to the splenic flexure. The patient is currently afebrile with a BP of 116/78 mm Hg and an HR of 80 beats/minute. Laboratory findings are within normal limits. Which one of the following is best to recommend for this patient? A. Mesalamine suppository rectally 500 mg twice daily plus MMX mesalamine (Lialda) 2.4 g orally daily. B. Mesalamine enema rectally 2 g/day plus sulfasalazine I g orally four times/day. C. Hydrocortisone enema rectally 100 mg/day plus mesalamine 1.6 g orally three times/day. D. Prednisone 60 mg orally daily plus sulfasalazine I g orally four times/day.

Answer B: Mesalamine enema rectally 2 g/day plus sulfasalazine 1 g orally four times/day. (The patient presents with new-onset UC, which is classified as mild to moderate distal disease confined to the rectum and extending proximally to the splenic flexure. Aminosalicylates are the mainstay of initial therapy, and a combination of topical and oral aminosalicylates is more effective than either alone. This patient's leftsided disease requires the use of an aminosalicylate enema, which would treat the splenic flexure (Answer B is correct), versus a suppository, which would be effective only for proctitis (Answer A is incorrect). Although topical corticosteroids may be used, they are not as effective as topical aminosalicylates (Answer C is incorrect). Oral corticosteroid use is reserved for patients whose disease is refractory to aminosalicylates or for patients who have moderate to severe disease or signs of systemic illness (Answer D is incorrect).)

A 67-year-old man with significant heart disease recently received a diagnosis of Cushing syndrome caused by an adrenal tumor. He receives twice-daily high-dose lansoprazole for severe gastroesophageal reflux disease. He is not a surgical candidate because of his heart disease. Which one of the following is best to initiate in this patient? A. Ketoconazole. B. Metyrapone. C. Eplerenone. D. Spironolactone.

Answer B: Metyrapone. (Eplerenone and spironolactone are aldosterone antagonists used to treat hyperaldosteronism, not Cushing syndrome (Answer C and Answer D are incorrect). Ketoconazole and metyrapone are used to treat Cushing syndrome, but ketoconazole requires an acidic stomach pH for absorption (Answer A is incorrect). Because this patient is receiving high-dose proton pump inhibitor therapy, metyrapone is the best choice (Answer B is correct).)

A 74-year-old man is admitted to the hospital with an infected prosthetic hip that was placed 20 days ago. Surgery is performed, the area is washed out, and a joint spacer is placed. Samples of the joint space reveal S. aureus, with sensitivity to all tested antibiotics except penicillin and oxacillin. After an intravenous course of antibiotics, the patient requires initiation of oral therapy for chronic suppression. Which one of the following is best to recommend for this patient? A. Amoxicillin. B. Minocycline. C. Ciprofloxacin. D. Rifampin.

Answer B: Minocycline. (Infections of prosthetic joints are complicated, and the treatment course of choice is removal, washout (with a joint spacer often added for mobility), antibiotics for a prolonged course, and reinsertion of a new joint. Staphylococci are the most common cause of these infections. Ciprofloxacin is the preferred treatment for osteomyelitis associated with Pseudomonas; however, this is not true for this patient (Answer C is incorrect). Additional consideration must be given to the long-term use of fluoroquinolones because they are associated with tendinopathy and corrected QT (QTc) prolongation. Amoxicillin is not active against penicillin-resistant staphylococci (Answer A is incorrect). Rifampin is active, but resistance can emerge quickly when it is used as monotherapy (Answer D is incorrect). Minocycline is an appropriate option for oxacillin-resistant S. aureus (Answer B is correct).)

A 67-year-old man with a history of H TN, T2DM, and mitral stenosis secondary to rheumatic heart disease presents to the cardiology clinic for follow-up. His HR is 82 beats/minute, but his rhythm is determined to be irregularly irregular during physical examination. Electrocardiography reveals AF. On previous echocardiography, a dilated left atrium secondary to mitral stenosis was noted; this is currently being medically managed. Results of his CBC, activated PTT (aPTT), PT/INR, and basic metabolic panel are within normal limits, and his fasting glucose is 123 mg/dL. The patient currently takes aspirin 81 mg daily, simvastatin 40 mg at bedtime, lisinopril 20 mg daily, and metformin 1000 mg twice daily. Given his medical history, which one of the following is this patient's strongest single risk factor for stroke? A. Age B. Mitral stenosis. C. Diabetes. D. HTN

Answer B: Mitral stenosis. (Atrial fibrillation is associated with an increased long-term risk of stroke, HF, and all-cause mortality. Risk increases with age, severity of underlying heart disease, and presence of comorbid conditions such as HF, HTN, and diabetes. The rate of ischemic stroke in patients with nonrheumatic AF averages 5% per year, but in patients with a history of rheumatic heart disease and AF, stroke risk increases 17-fold, and the attributable risk is 5 times greater than in those with nonrheumatic AF. Because this patient has a history of rheumatic heart disease, his strongest single risk factor for stroke is mitral stenosis (Answer B is correct). Age, diabetes, and HTN are all considered moderate risk factors compared with his mitral stenosis (Answer A, Answer C, and Answer D are incorrect).)

A 40-year-old woman with newly diagnosed stage Ill breast cancer is scheduled to receive her first cycle of doxorubicin plus cyclophosphamide administered every 21 days. You provide chemotherapy counseling, including administering predictive tests for both acute and delayed chemotherapy-induced nausea and vomiting (CINV). On questioning, the patient denies alcohol or illicit substance use, and you learn she has a history of getting seasick on a cruise ship last year, is quite nervous regarding her first cycle of chemotherapy, and slept poorly the night before. Which of the following premedication regimens is best for this patient? A. Ondansetron 16 mg orally, dexamethasone 10 mg intravenously, and lorazepam I mg intravenously. B. Palonosetron 0.25 mg intravenously, dexamethasone 12 mg intravenously, and aprepitant 120 mg orally on day 1 and 80 mg orally on days 2 and 3. C. Ondansetron 16 mg intravenously, dexamethasone 10 mg intravenously, lorazepam I mg orally, and scopolamine 1.5-mg patch applied for 3 days. D. Palonosetron 0.25 mg intravenously, lorazepam I mg intravenously, and prochlorperazine 10 mg intravenously.

Answer B: Palonosetron 0.25 mg intravenously, dexamethasone 12 mg intravenously, and aprepitant 120 mg orally on day 1 and 80 mg orally on days 2 and 3. (Both tools for predicting acute and delayed CINV were studied and validated in a study group of predominantly female patients with breast cancer. Pertinent details in this patient include stage III disease (increased risk of both acute and delayed CINV), receipt of anthracycline (increased risk of both acute and delayed CINV), no reported alcohol use (lack of protective effect on acute CINV), age younger than 41 (increased risk of delayed CINV), and poor night's sleep (increased risk of delayed CINV). In this context, her antiemetic regimen should be aggressive for both acute and delayed CINV. Palonosetron 0.25 mg intravenously, dexamethasone 12 mg intravenously, and aprepitant 120 mg orally on day 1 and 80 mg orally on days 2 and 3 is correct; the combination of a serotonin-3 antagonist and dexamethasone is required for preventing acute CINV, and aprepitant prevents CINV (Answer B is correct). Ondansetron 16 mg orally, dexamethasone 10 mg intravenously, and lorazepam 1 mg intravenously (regimen A) and ondansetron 16 mg intravenously, dexamethasone 10 mg intravenously, lorazepam I mg orally, and scopolamine 1.5-mg patch applied for 3 days (regimen C) are aggressive for acute CINV, but not delayed CINV. (Answer A and Answer C are incorrect.) Additionally, scopolamine adds minimal benefit to the regimen for delayed CINV. Palonosetron 0.25 mg intravenously, lorazepam 1 mg intravenously, and prochlorperazine 10 mg intravenously is not aggressive enough for either acute or delayed CINV (Answer D is incorrect).)

A 65-year-old Asian woman presents to an emergency department with a 24-hour history of black, tarry stools; confusion; dizziness; and hematemesis. She has a medical history of osteoarthritis, hypertension, hyperlipidemia, and myocardial infarction. Her current drugs include ibuprofen 400 mg twice daily, carvedilol 12.5 mg twice daily, clopidogrel 75 mg/day, atorvastatin 20 mg/day, linisprol 10 mg/day. She has no known drug allergies and denies ever taking azithromycin. Endoscopy reveals a 2-cm antral ulcer with a visible vessel. Her vital signs include BP 95/50 mm Hg and HR 102 beats/minute. Pertinent laboratory values include Hgb 7.4 g/dL, Hct 22.8%, SCr 1.4 mg/dL, and BUN 20 mg/dL. Endoscopic intervention is successful, and she subsequently tests positive for Helicobacter pylori infection. Which one of the following is best to recommend for this patient? A. Omeprazole 20 mg twice daily, amoxicillin 1000 mg twice daily, and clarithromycin 500 mg twice daily. B. Pantoprazole 40 mg twice daily, amoxicillin 1000 mg twice daily, and clarithomycin 500 mg twice daily. C. Esomeprazole 40 mg twice daily, metronidazole 500 mg twice daily, and clarithomycin 500 mg twice daily. D. Omeprazole 20 mg twice daily, bismuth subsalicylate 262.4 mg four times/day, metronidazole 250 mg four times/day, and tetracycline 500 mg four times/day.

Answer B: Pantoprazole 40 mg twice daily, amoxicillin 1000 mg twice daily, and clarithomycin 500 mg twice daily. (For the patient who requires H. pylori eradication therapy, triple therapy triple therapy with amoxicillin, clarithromycin, and a PPI is recommended as the first-line treatment. Because this patient requires clopidogrel therapy, omeprazole is best avoided because of the potential for drug interaction (Answer A is incorrect). In patients who require clopidogrel therapy, pantoprazole is the preferred PPI with the least potential for drug interaction and should be used as part of the H. pylori eradication regimin (Answer B is correct). Esomeprazole also has the potential for drug interaction with clopidogrel and metronidazole is only preferred in patients with a penicillin allergy (Answer C is incorrect). Quadruple therapy may be appropriate for H. pylori eradication therapy if clarithomycin resistance is suspected because of the patient's previous exposure to a macrolide antibiotic drug. This patient denies ever taking azithromycin, and the frequency of drug dosing (four times/day) may decrease adherence to a quadruple regimen (Answer D is incorrect).)

A 53-year-old postmenopausal woman requests hormone therapy for the uncomfortable hot flashes she experiences 10 times/day as well as dyspareunia associated with vaginal dryness. Her medical history is significant for hypertension and hypothyroidism. Her uterus is intact, and her last menstrual period was at age 51. Her social history is significant for current tobacco use (I pack/day for 32 years). Her current drugs include lisinopril, levothyroxine, and a nonprescription vaginal moisturizer. Which one of the following hormone regimens is best for this patient? A. 17ß-Estradiol topical emulsion (0.25%) applied once daily. B. 17ß-Estradiol vaginal ring (0.0075 mg/24 hours) inserted once every 3 months plus a 200-mg micronized progesterone oral capsule once daily for 12 days/month. C. 17ß-Estradiol/levonorgestrel (0.045 mg/0.015 mg/24 hours) transdermal patch applied once weekly. D. 0.625-mg conjugated estrogen/2.5-mg medroxyprogesterone acetate oral tablet once daily.

Answer C: 17ß-Estradiol/levonorgestrel (0.045 mg/0.015 mg/24 hours) transdermal patch applied once weekly. (The 17ß-estradiol and levonorgestrel combination patch may be used to treat both vasomotor and vulvovaginal symptoms (Answer C is correct). Because the patient is a smoker and thus at greater risk of VT E, estradiol given nonorally is the safer option because it has less effect on clotting factors than the oral form, which has first-pass liver metabolism. Because the patient has a uterus, she will require the endometrial protection of a progestin, so 17ß-estradiol topical emulsion as monotherapy is not the best choice (Answer A is incorrect). The lowest-dose 17ß-estradiol vaginal ring is intended to provide only local drug concentrations and will not produce systemic estradiol concentrations sufficient to treat vasomotor symptoms, regardless of using micronized progesterone (Answer B is incorrect). The conjugated estrogen/medroxyprogesterone acetate tablet could subject her to greater V TE risk because she smokes (Answer D is incorrect).)

A study is being planned to evaluate the pediatric use of a drug that has been safely used in adults for several years. For the particular indication, disease progression is similar in adults and children. In addition, the drug concentration-response in children is expected to be similar to that in adults. Which one of the following best describes the studies that will be required for the drug to receive FDA-approved labeling for use in children? A. Pharmacokinetic and concentration-response studies. B. Pharmacokinetic and safety trials. C. Safety trials only. D. Pharmacokinetic trials only.

Answer B: Pharmacokinetic and safety trials. (The Pediatric Decision Tree, noted in the following citation for FDA exposure-response guidance, outlines the studies that must be performed. Because safety is never extrapolated and must always be performed, neither pharmacokinetic and concentration-response studies nor pharmacokinetic trials only are appropriate choices (Answer A and Answer D are incorrect). Because exposure will be matched to adult drug exposure, a pharmacokinetic study in the pediatric populations of interest will be needed to establish the proper exposure in the pediatric population (Answer C is incorrect). Both pharmacokinetic and safety trials will be needed (Answer B is correct).)

A trial is conducted in 60 healthy volunteers to determine the pharmacokinetic and pharmacodynamic parameters of a new antihypertensive medication. Which one of the following best describes this type of trial? A. Preclinical study. B. Phase I drug trial. C. Phase II drug trial. D. Phase III drug trial.

Answer B: Phase I drug trial. (A phase I drug trial represents the initial introduction of an Investigational New Drug (IND) into humans and typically involves 20—100 healthy volunteers. The goal of this type of study is to gain information on the pharmacokinetic and pharmacodynamic properties of the drug to help design well-controlled and robust phase II drug trials (Answer B is correct). Preclinical trials are conducted in animals to determine toxicology and the effects of fetal outcomes, which may or may not translate to human fetal adverse events (Answer A is incorrect). A phase II study is a controlled clinical study conducted in no more than several hundred subjects—that serves to evaluate the drug's effectiveness for a particular indication in patients with the disease or condition under investigation, as well as determine the common short-term adverse effects and risks associated with the drug (Answer C is incorrect). A phase III drug trial involves the administration of the investigational drug to a range of several hundred to several thousand patients in different clinical settings to determine its safety, efficacy, and appropriate dosage. The goal is to gather necessary additional information about effectiveness and safety for evaluating the overall benefit-risk relationship of the drug and to provide an adequate basis for physician labeling (Answer D is incorrect).)

A 63-year-old man is admitted to the medical ICU with sepsis secondary to pneumonia. On day 4 in the ICU, he is found to be CAM-ICU (confusion assessment method for the ICU) positive. Which of the following medications would be best for the treatment of delirium in this patient? A. Haloperidol. B. Quetiapine. C. Dexmedetomidine. D. Lorazepam.

Answer B: Quetiapine. (Atypical antipsychotics such as quetiapine are recommended to shorten the duration of delirium in critically ill adult patients (Answer B is correct). Haloperidol has not been shown to reduce the duration of delirium in these patients (Answer A is incorrect). Although studies show a lower incidence of delirium in patients receiving dexmedetomidine compared with benzodiazepines, evidence is insufficient to recommend it as a treatment for delirium, the data only suggests that it is a preferred modality of sedation over benzodiazepines (Answer C is incorrect). Benzodiazepines (e.g., lorazepam) increase the incidence of delirium and are not an appropriate treatment modality in patients with delirium unrelated to alcohol or benzodiazepine withdrawal (Answer D is incorrect).)

A patient with stage 5 CKD on hemodialysis (weight 140 kg [308.5 lb]) has an Hgb of 8.9 g/dL, ferritin of 350 ng/mL, and TSAT of 39%. The patient is initiated on darbepoetin 60 mcg subcutaneously every week. Two weeks later, his Hgb is 11.4 g/dL, ferritin is 225 ng/mL, and TSAT is 29%. Which one of the following is best to recommend for this patient? A. Continue darbepoetin 60 mcg subcutaneously every week because Hgb is at goal; recheck Hgb in 1 month. B. Reduce darbepoetin to 40 mcg subcutaneously every week and recheck Hgb in 2 weeks. C. Reduce darbepoetin to 60 mcg subcutaneously every 2 weeks and recheck Hgb in 2 weeks. D. Hold darbepoetin and recheck Hgb in 2 weeks.

Answer B: Reduce darbepoetin to 40 mcg subcutaneously every week and recheck Hgb in 2 weeks. (Although a minimum Hgb target has not been established, Hgb should not exceed 11 g/dL. In addition, the dose should be adjusted to achieve a I-g/dL increase in Hgb in a 2- to 4-week period (Answer B is correct). The current product information guidelines for darbepoetin suggest a 25% reduction in dose if the Hgb concentration increases by more than 1 g/dL in any 2-week period (Answer A is incorrect). The total dose may be adjusted by increasing the dosing interval; however, a 50% reduction in dose is incorrect (Answer C is incorrect). National Kidney Foundation guidelines do not recommend holding the ESA dose when a downward adjustment of the Hgb concentration is needed. Holding a dose may result in the Hgb concentration becoming too low (Answer D is incorrect).)

A 56-year-old postmenopausal woman undergoes dual-energy x-ray absorptiometry (DXA). Her medical history includes HTN and rheumatoid arthritis. Her current drugs are calcium/vitamin D 600 mg/400 units taken twice daily with food and methotrexate 20 mg/week. Results of the DXA are as follows: Region || T-Score || Z-Score ----------------------------------------------- L1-L4 || -2.3 || -1.2 ----------------------------------------------- Femoral Neck || -1.9 || -1.0 ----------------------------------------------- Total Hip || -1.8 || -0.8 Her fracture risk assessment tool (FRAX) score reveals a 10-year risk of 22% for any major fracture and 2.8% for a hip fracture. Her 25-hydroxyvitamin D concentration is 35 ng/nL. Which one of the following is best to recommend for this patient? A. Cholecalciferol. B. Risedronate delayed release. C. Teriparatide. D. Raloxifene.

Answer B: Risedronate delayed release. (The 2013 National Osteoporosis Foundation and the 2010 AACE guidelines recommend using the FRAX to help assess fracture risk and need for pharmacotherapy in patients with low bone density Patients with a high fracture risk (defined as a FRAX 10-year risk of any major osteoporotic fracture of 20% or greater or a hip fracture of 3% or greater) should begin pharmacotherapy to decrease the risk of fracture. According to the AACE 2010 guidelines, bisphosphonates (except ibandronate) and denosumab are considered first-line agents because they decrease the risk of vertebral, nonvertebral, and hip fractures (Answer B is correct). The patient does not have documented low vitamin D concentrations; therefore, the use of cholecalciferol is unwarranted (Answer A is incorrect). Neither teriparatide nor raloxifene protects against hip fracture, and neither is considered a first-line agent (Answer C and Answer D are incorrect).)

A 21-year-old man has mature B-cell acute lymphocytic leukemia. His disease is in its second remission, and he is typed for allogeneic HSCT. The patient received conditioning chemotherapy with busulfan plus cyclophosphamide as his preparative regimen and then underwent a successful transplant. He is now receiving tacrolimus 0.02 mg/kg and methotrexate as GVHD prophylaxis. Which one of the following provides the best monitoring parameters for this patient's immunosuppressive regimen? A. SCr, TG, and albumin. B. SCr, hepatic panel, K, and Mg. C. Electrocardiography and urinalysis. D. WBC, platelet count, and hemoglobin.

Answer B: SCr, hepatic panel, K, and Mg. (The dose-limiting toxicity of tacrolimus is nephrotoxicity, which requires stringent monitoring. Electrolyte disturbances can also occur because of electrolyte wasting; hyperkalemia can develop with acute kidney failure. In addition to nephrotoxicity, methotrexate and tacrolimus can result in hepatotoxicity (Answer B is correct). Lipid panels should be monitored in patients with a history of coronary artery disease or diabetes, but this monitoring is not routine for patients on immunosuppression (Answer A is incorrect). Cardiac monitoring is required for patients receiving cyclophosphamide, but routine monitoring is not required for patients receiving CNI prophylaxis (Answer C is incorrect). Complete blood cell counts are monitored daily during hospitalization for transfusion requirements, but they are not necessarily required for immunosuppression (Answer D is incorrect).)

A 26-year-old sexually active man with no known drug allergies presents to the clinic with a markedly inflamed right eye expressing a thick, mucopurulent, yellowish green discharge. The patient also reports a 3-day history of burning sensation while urinating. On further examination, his cornea is intact, and periauricular adenopathy is noted. Which one of the following is the best therapy for this patient? A. Hospital admission for 3 days of intravenous ceftriaxone 1 g/day. B. Single doses of ceftriaxone 250 mg intramuscularly and azithromycin 1000 mg orally. C. Single dose of azithromycin 1000 mg orally. D. Single dose of ceftriaxone 250 mg intramuscularly.

Answer B: Single doses of ceftriaxone 250 mg intramuscularly and azithromycin 1000 mg orally. (Symptoms of an inflamed eye with yellowish green mucopurulent discharge in a young, sexually active man are most consistent with hyperacute bacterial conjunctivitis secondary to N. gonorrhoeae. Men presenting with hyperacute disease often have concomitant urethritis, as this patient does. Because the coinfection rate with Chlamydia is high, it is recommended to also treat this patient for Chlamydia trachomatis. He should receive single doses of ceftriaxone and azithromycin to treat both pathogens (Answer B is correct). The patient's cornea appears to be intact with no evidence of ulceration, so hospital admission for intravenous ceftriaxone is not needed (Answer A is incorrect). Treating with ceftriaxone alone would not adequately treat Chlamydia (Answer C is incorrect), and treating with azithromycin alone would not adequately treat gonorrhea (Answer D is incorrect). Given that no sensitivity or culture data are reported in this case, both organisms must be covered empirically.)

A 62-year-old man with H TN, T2DM, and recurrent gout attacks (six in the past year) presents to the clinic for follow-up. Today, his BP is 152/92 mm Hg (repeat 154/96 mm Hg), and HR is 74 beats/minute. His antihypertensive regimen consists of carvedilol 25 mg orally twice daily, amlodipine 10 mg orally once daily, and lisinopril 40 mg orally once daily. He has a history of intolerance to allopurinol and probenecid. His laboratory tests today reveal K 4.0 mEq/L, SCr 1.0 mg/dL, and uric acid 7.8 mg/dL. Which one of the following antihypertensive agents would be best to initiate for this patient? A. Hydrochlorothiazide 25 mg daily. B. Spironolactone 12.5 mg daily. C. Valsartan 80 mg daily. D. Clonidine 0.1 mg daily.

Answer B: Spironolactone 12.5 mg daily. (Hydrochlorothiazide is known to increase uric acid and may precipitate gout attacks (this patient has had six attacks in the past year); it is therefore inappropriate for this patient (Answer A is incorrect). Combination therapy with an ACE inhibitor (lisinopril in this case) and valsartan, an ARB, is not routinely recommended in the treatment of HTN (Answer C is incorrect). Clonidine could be an option, but it is not preferred. Furthermore, dosing with clonidine should occur more than once daily because of its shorter half-life (Answer D is incorrect). Numerous trials have shown spironolactone to be a highly effective fourth-line agent in treating resistant HTN. The patient has normal SCr and K concentrations and no contraindication to spironolactone use (Answer B is correct).)

While working as the consulting pharmacist at a local nursing home, you review the patient chart of a 68-year-old woman who is new to the nursing home and who is undergoing a workup for dementia. For the past few months, she has been experiencing loss of memory to recent events, poor recall of childhood events, wandering, and aggression. Her family states that her personality has changed so much that they "don't recognize her." Her drugs include metformin 1000 mg twice daily, glipizide extended release 5 mg/day, enalapril 5 mg/day, amlodipine 5 mg/day, aspirin 81 mg/day, and rosuvastatin 10 mg/day. Her pertinent laboratory values include AIC 6.9%, SCr 0.8 mg/dL, LDL-C 98 mg/dL, HDL-C 40 mg/dL, TG 94 mg/dL, vitamin 112 pg/mL, and 25(OH) vitamin D 35 ng/mL. Which one of the following is best to recommend for this patient? A. Start oral ergocalciferol. B. Start intramuscular cyanocobalamin. C. Discontinue rosuvastatin. D. Increase aspirin to 325 mg/day.

Answer B: Start intramuscular cyanocobalamin. (This patient should be initiated on cyanocobalamin because of her low vitamin level (normal range 200—900 pg/mL), which could be contributing to some of her cognitive changes (Answer B is correct). Recent studies have also shown that doses of oral cyanocobalamin ranging from 1 to 2 g daily produced effects similar to the intramuscular dosage form. Ergocalciferol is not indicated because a 25(OH) vitamin D level greater than 30 ng/mL is considered sufficient by the Endocrine Society guidelines (Answer A is incorrect). Discontinuing rosuvastatin is unlikely to change this patient's mental status (Answer C is incorrect). This patient is currently on a prophylactic aspirin regimen and carries no significant cardiac history or diagnosis of vascular dementia to warrant an increase in aspirin dose, which could increase her risk of bleeding (Answer D is incorrect).)

A 25-year-old woman comes to the clinic because of a new-onset skin condition. She has a history of asthma, gout, and menstrual migraines; she has no known drug allergies. Her home medications include an albuterol metered-dose inhaler (3 years), a fluticasone/salmeterol inhaler (l year), allopurinol (l year), a combination oral contraceptive (5 years), a multivitamin (5 years), calcium (6 months), vitamin D (6 months), sumatriptan (1 year), and nabumetone (2 months). Her first skin lesions occurred 2 weeks ago as macules and vesicles on the palms of her hands and soles of her feet. Now, she has macules and vesicles on her abdomen and upper extremities with the presence of oral mucosal lesions. Minimal skin sloughing is present. Which one of the following best describes her cutaneous lesions? A. Erythema multiforme. B. Stevens-Johnson syndrome (SJS). C. Toxic epidermal necrolysis (TEN). D. Transitional SJS/TEN.

Answer B: Stevens-Johnson syndrome (SJS). (The patient's lesions are most consistent with SJS which are most likely secondary to the recent addition of nabumetone to her drug regimen. Stevens-Johnson syndrome is a systemic version of erythema multiforme and includes severe mucosal erosions but epidermal detachment less than 10% (Answer B is correct). When 1000 to 30% of the skin is lost, SJS and TEN overlap and may be referred to as transitional SJS/TEN (Answer D is incorrect). Toxic epidermal necrolysis is the most severe form, with acute generalized erythema progressing to desquamation of greater than 30% of the skin surface, often within 24 hours (Answer C is incorrect). Erythema multiforme consists of macules, plaques, vesicles, and bullae appearing primarily on the palms and soles and then spreading to other areas, but no skin sloughing (Answer A is incorrect).)

Using a large database, a cohort study is conducted in individuals without health care insurance to assess the rate of adverse drug events resulting in hospitalization. All hospitals in the multicounty area have agreed to contribute to this data set. Which one of the following is the main limitation of this data set? A. Confounding will interfere with interpretation of the study results. B. Study findings may not be generalizable to individuals who have insurance. C. Selection bias will interfere with interpretation of the findings. D. Information bias will interfere with interpretation of the findings.

Answer B: Study findings may not be generalizable to individuals who have insurance. (Patients with health insurance may differ from those who are not insured in numerous ways, making direct comparisons difficult (Answer B is correct). For example, individuals with health insurance have relatively easy access to health care, whereas people who are uninsured generally have more restricted access. People with employer-based health insurance are either employed or part of a household in which someone has a job that offers insurance as a benefit. These jobs may also require some level of training or education, which individuals who are uninsured may or may not have. Although confounding could be present in this study, this is not the main limitation (Answer A is incorrect). In addition, selection bias could interfere with interpretation of the findings, but this is not the main limitation (Answer C is incorrect). Finally, information bias is unlikely in this case (Answer D is incorrect).)

A 27-year-old woman was initiated on a low-dose combined oral contraceptive containing 20 mcg of ethinyl estradiol 2 months ago. She went out of town for the weekend and missed two doses of her contraceptive. Today, she is in the third week of her cycle and wants to know what to do about the missed doses. Which one of the following is the most appropriate recommendation for this patient? A. Take an active tablet as soon as possible (2 tablets on that day) and then continue taking I tablet daily. No additional contraceptive protection is recommended. B. Take an active tablet as soon as possible (2 tablets on that day) and then continue taking I tablet daily. Use condoms or abstain from sex until tablets have been taken for 7 days in a row. Finish the active tablets in the current pack and then start a new pack. C. Discard the current pack, allow bleeding to occur, and then restart a new pack, taking I tablet daily. Use condoms or abstain from sex until the new pill pack has been taken for 7 days in a row. D. Take an active tablet as soon as possible (2 tablets on that day) and then continue taking I tablet daily. Use condoms or abstain from sex until tablets have been taken for 7 days in a row. Use one dose of emergency contraception.

Answer B: Take an active tablet as soon as possible (2 tablets on that day) and then continue taking 1 tablet daily. Use condoms or abstain from sex until tablets have been taken for 7 days in a row. Finish the active tablets in the current pack and start a new pack. (A missed dose of contraceptive tablets is a common problem. In 2013, the Centers for Disease Control and Prevention adapted the World Health Organization (WHO) recommendations for missed doses to a single recommendation, citing a balance of science and simplicity. The recommendation for two or more missed doses consists of taking an active tablet as soon as possible (2 tablets on that day) and then continue taking 1 tablet daily. Use condoms or abstain from sex until tablets have been taken for 7 days in a row. Finish the active tablets in the current pack and then start a new pack (Answer B is correct). A longer hormone-free interval increases the risk of contraceptive failure, necessitating abstinence or the use of backup contraception (Answer A is incorrect). Discarding tablets is recommended only when women have missed inactive (nonhormonal) tablets during the fourth week (Answer C is incorrect). Emergency contraception is usually recommended when doses are missed during the first week of the cycle (Answer D is incorrect).)

In a randomized controlled trial of analgesia after total joint replacement surgery, the median difference between treatment groups in the 0—10 numeric rating scale of pain intensity on postoperative day 2 was 2.2 (95% CI, 1.1—3.5; p=0.01). Which one of the following statements best summarizes whether the p value or the CI is preferred? A. The p value is preferred because it provides an estimate of the magnitude of the observed difference. B. The CI is preferred because it provides an estimate of the magnitude of difference. C. The p value is preferred because it provides the insight needed for clinical decision-making. D. The CI is preferred because it provides the insight needed for clinical decision-making.

Answer B: The CI is preferred because it provides an estimate of the magnitude of difference. (Confidence intervals show the range over which a point estimate is consistent with the data and reveal the random error in an estimate (Answer B is correct). The p value is the probability, conditional on the null hypothesis, of observing at least as large a difference as was observed. The p values are calculated using statistical models that correspond to the type of data collected. In this question, a difference of anywhere from I to 3.5 units on the 0—10 numeric rating scale is consistent with the data. The p value does not provide an estimate of the magnitude of the observed difference, although it is often interpreted this way (Answer A is incorrect). Neither a precise p value nor the CI provides insight sufficient for clinical decision-making (Answer C and Answer D are incorrect).)

In which one of the following scenarios would it be best to apply the Wilcoxon signed rank test? A. The data are continuous and parametric, and groups I and 2 consist of the same individuals. B. The data are continuous but nonparametric, and groups I and 2 consist of the same individuals. C. The data are continuous and parametric, and groups I and 2 consist of different individuals. D. The data are continuous but nonparametric, and groups I and 2 consist of different individuals.

Answer B: The data are continuous but nonparametric, and groups 1 and 2 consist of the same individuals. (The Wilcoxon signed rank test is a nonparametric equivalent to the paired t-test (Answer B is correct). Thus, it is inappropriate for use with both parametric data (Answer A and Answer C are incorrect) and non-paired data (Answer D is incorrect).)

A 70-year-old man recently received a diagnosis of peripheral arterial disease (Fontaine stage IIB); he now returns to the family medicine clinic for follow-up. He has exertional leg pain. His laboratory tests reveal SCr 1.5 mg/dL and AST 45 U/L. Physical examination reveals a resting BP of 110/58 mm Hg and an ankle/brachial index of 0.80 in both legs. Echocardiographic findings include an LVEF of 25%. The medical resident wishes to initiate cilostazol 100 mg orally twice daily. Which one of the following is the most accurate evaluation of this patient's cilostazol regimen? A. The dose is inappropriate on the basis of laboratory findings. B. The drug is inappropriate on the basis of echocardiographic findings. C. The frequency is inappropriate on the basis of physical examination. D. The route is inappropriate on the basis of disease severity.

Answer B: The drug is inappropriate on the basis of echocardiographic findings. (The combination of a low ankle/brachial index (less than 0.90) and exertional leg pain is most consistent with intermittent claudication. Cilostazol is recommended for patients with intermittent claudication, yet and many of its metabolites are inhibitors of phosphodiesterase III. Several drugs with this pharmacologic effect have caused decreased survival compared with placebo in patients with class III—IV congestive HF. Cilostazol is contraindicated in patients with congestive HF of any severity and would not be appropriate in this individual with an LVEF of 25% (Answer B is correct). If the patient were eligible to receive cilostazol, the dose would not need to be modified according to the patient's laboratory findings (Answer A is incorrect). If the patient were eligible to receive cilostazol, it would be dosed twice daily (Answer C is incorrect). Cilostazol is only available orally (Answer D is incorrect).)

An 8-year-old boy is brought to the family medicine clinic with a red, vesicular, itchy rash that developed overnight. His mother reports he had been playing outside at his grandparents' farm the previous afternoon. The rash is limited to his forearms and lower legs. There are no lesions on his face or genital region. The patient denies any shortness of breath or difficulty breathing. Poison ivy is suspected. Which is the best recommendation for this patient? A. Topical diphenhydramine. B. Topical hydrocortisone. C. Methylprednisolone dose pack. D. Bentoquatam 5%.

Answer B: Topical hydrocortisone. (The patient presents with a mild case of poison ivy as evidenced by his lesion count, absence of facial lesions, and absence of shortness of breath. Topical steroids provide relief of itching in most cases of mild poison ivy (Answer B is correct). The use of topical antihistamines is not recommended because it can worsen the rash and increase the intensity of the itching (Answer A is incorrect). Because this patient has only a mild case of poison ivy, oral steroid therapy is not indicated at this time (Answer C is incorrect). Bentoquatam 5% is FDA approved to prevent poison ivy, but not as a treatment (Answer D is incorrect).

A 13-year-old boy is brought to the clinic with facial acne. He has 15 noninflamed comedones on the forehead, nose, and chin. He reports that no other body areas are affected, and no lesions are noted on the neck, shoulders, or back. He does not have evidence of facial scarring. Which one of the following is the most appropriate regimen for this patient? A. Salicylic acid wash once daily in the evening. B. Topical tretinoin applied once daily before bedtime. C. Topical clindamycin solution applied once daily in the morning. D. Oral erythromycin twice daily.

Answer B: Topical tretinoin applied once daily before bedtime. (This patient presents with mild noninflammatory acne. Monotherapy with a topical retinoid, applied daily to affected areas of the face, is the preferred treatment (Answer B is correct). The treatment goal is to prevent new lesions. The use of a topical retinoid in combination with benzoyl peroxide and an antibiotic could also be considered. Salicylic acid wash should only be used as adjunctive therapy (Answer A is incorrect). Topical clindamycin should not be used alone because of a lack of efficacy on comedone formation and the risk of resistance (Answer C is incorrect). Oral antibiotics should be used for moderate inflammatory acne; this patient does not require systemic therapy at this time (Answer D is incorrect).)

A 67-year-old woman was prescribed lansoprazole 30 mg/day for gastroesophageal reflux disease (GERD) and has been adherent to the drug for several weeks, Now, she wants to stop the treatment because she is asymptomatic and wants to limit the number of drugs she has to take. Her medical history includes congestive heart failure and type 2 diabetes mellitus. Her drugs include lisinopril 40 mg/day, carvedilol 12.5 mg twice daily, furosemide 20 mg/day, sitagliptin 50 mg/day, and insulin glargine 30 units/day. Which one of the following is best to recommend for this patient? A. Proton pump inhibitor (PPI) therapy is lifelong and should not be discontinued. B. Try a drug holiday; restart if symptoms recur. C. Discontinue the PPI; initiate a histamine-2 receptor antagonist (H2RA). D. Refer patient to a gastroenterologist.

Answer B: Try a drug holiday; restart if symptoms recur. (Cyclic treatment is a good option for this patient; she may stop PPI therapy and resume at a future time for 4-8 weeks if symptoms recur (Answer B is correct). Lifelong therapy is not recommended in patients with GERD, especially if they are asymptomatic and have nonerosive esophagitis (Answer A is incorrect). Because the patient has not been taking a PPI for an extended period, switching to continuous H2RA is not necessary. In addition, she is consumed with pill burden (Answer C is incorrect). The patient has no indication for gastroenterology referral (no alarm symptoms; her GERD did not fail to respond to PPI therapy) (Answer D is incorrect).)

A 67-year-old man with a history of chronic heart failure is admitted to the coronary care unit after suffering a myocardial infarction. On admission, his family reports that he weighed 70 kg (154.5 lb) at his last cardiologist visit 6 months ago, but his usual weight is 82 kg (181 lb). After his myocardial infarction, the patient developed cardiogenic shock, requiring fluid resuscitation, dobutamine, and norepinephrine. He is receiving continuous renal replacement therapy (CRRT) using citrate anticoagulation. Which one of the following is the best estimate of this patient's daily caloric requirement? A. 1230 kcal/day. B. 1640 kcal/day. C. 2460 kcal/day. D. 3280 kcal/day.

Answer C: 2460 kcal/day. (Because of fluid overload, this patient's usual weight (82 kg [181 lb]) or his most recent preadmission weight (70 kg [154.5 lb]) should be used to calculate his caloric requirements. His estimated caloric requirement is 25—35 kcal/kg/day. Answer C (2460 kcal/day) provides 30 kcal/kg/day, based on the patient's usual body weight (82 kg [181 lb]), or 35 kcal/kg/day, based on his most recent preadmission weight, and is therefore the best answer. The options of 1230 kcal/kg/day and 1640 kcal/kg/day provide less than 25 kcal/kg/day; these amounts are less than recommended for patients such as this one (Answer A and Answer B are incorrect). Answer D (3280 kcal/kg/day) provides more than 35 kcal/kg/day, which would be an excessive daily caloric intake for this patient, leading to complications associated with overfeeding.)

A 62-year-old man who received a liver transplant years ago secondary to hepatitis C virus and hepatocellular cancer has had an elevated SCr during the past few months at follow-up. The hepatologist wants to change him to a less nephrotoxic immunosuppressive regimen. The patient's current drugs are tacrolimus 2 mg twice daily, famotidine 20 mg twice daily, metoprolol 50 mg twice daily, amlodipine 10 mg once daily, zolpidem 10 mg as needed, multivitamin once daily, and supplemental calcium, which he takes when he remembers. The decision is made to initiate sirolimus to decrease calcineurin inhibitor (CNI) exposure. Which one of the following test panels would be best to obtain before initiating sirolimus therapy in this patient? A. Urine protein, thyroid-stimulating factor (TSH), and lipid panel. B. Urine protein, WBC, and lipid panel. C. WBC and TSH. D. TSH and lipid panel.

Answer B: Urine protein, WBC, and lipid panel. (Assessing patients before initiating a new immunosuppressive drug therapy is necessary not only to ensure that the new therapy is appropriate, but also to evaluate tolerability and adherence. Sirolimus is associated with several adverse effects that tend to limit its use, are dose limiting, and lead to a decreased quality of life. Commonly observed toxicities include cytopenia, edema, hyperlipidemia, diarrhea, impaired wound healing, and proteinuria; rarely observed toxicities include bronchial anastomotic dehiscence, venous thromboembolism, and pneumonitis. Screening for potential complications before initiating therapy may help lead to positive outcomes. Checking a lipid panel, urine protein, and WBC before initiating sirolimus therapy is necessary (Answer B is correct). Thyroid function is not known to be affected by the use of sirolimus (Answer A, Answer C, and Answer D are incorrect).)

A 39-year-old Woman (weight 110 kg, height 163 cm) presents to her primary care physician with substernal chest pain. The pain has occurred at around 7:00 a.m. for the past 2 weeks. She ranks the pain as 5/10. The pain does not appear to be associated with an activity and lasts less than 10 minutes. Social history reveals no tobacco or alcohol abuse. Her home drugs include norethindrone 1 mg/Ethinyl estradiol 0.035 mg daily. Vital signs include BP 134/74 mm Hg and HR 86 beats/minute. Laboratory results include TC 160 mg/dL, TG 120 mg/dL, and HDL-C 45 mg/dL; a chemistry panel is within normal limits. A treadmill stress test is performed; however, the patient is unable to complete the examination because of leg cramping. Subsequent coronary angiography reveals no coronary lesions; however, vasospasms are noted in the circumflex artery. Which one of the following is best to recommend for this patient? A. Atenolol 50 mg by mouth once daily B. Verapamil sustained-release mg by mouth once daily. C. Isosorbide mononitrate 30 mg by mouth once daily in the morning. D. Ranolazine 1000 mg by mouth twice daily.

Answer B: Verapamil sustained release 180 mg by mouth once daily. (This patient is experiencing chest pain secondary to coronary vasospasms. β-blockers may worsen coronary vasospasms by unopposed α-receptor activity and should therefore be avoided in this patient population (Answer A is incorrect). Calcium channel blockers (CCBs), including verapamil, are a preferred therapy for Prinzmetal (vasospastic) angina because of their ability to stabilize coronary vasospasms (Answer B is correct). Although nitrates may provide some relief in vasospastic angina by dilating large epicardial vessels, isosorbide mononitrate given once daily will provide a duration of action of only 12-14 hours and will not provide protection against vasospasm during off hours (Answer C is incorrect). The main mechanism of ranolazine involves the cardiac tissue, not the vasculature, and it has no role in angina secondary to vasospasm. In addition, the recommended starting dose would be 500 mg twice daily (Answer D is incorrect).)

A 2-year-old white girl receives a diagnosis of precursor B-cell acute lymphocytic leukemia. Her original complete blood cell count revealed WBC 28 x 10^3 cells/mm^3, Hgb 6.5 mg/dL, and platelet count 55,000 cells/mm^3 All of her blood chemistry test results were within normal limits. The patient's initial lumbar puncture was negative; her DNA index was 1.22. In addition to prednisone and intrathecal therapy, which one of the following regimens is best to initiate for this patient? A. Cyclophosphamide, doxorubicin, and mercaptopurine. B. Vincristine, daunorubicin, and asparaginase. C. Cytarabine, daunorubicin, and asparaginase. D. Cytarabine, cyclophosphamide, and mercaptopurine.

Answer B: Vincristine, daunorubicin, and asparaginase. (A three- or four-drug remission induction regimen containing vincristine, a glucocorticoid, and either asparaginase or an anthracycline (or both) is known to induce remission in up to 99% of patients (Answer B is correct). Cyclophosphamide and mercaptopurine are not used at the beginning of therapy; however, they may be incorporated into therapy later in induction (Answer A is incorrect). Cytarabine is not one of the three or four drugs used in the initial induction treatment of acute lymphoblastic leukemia (Answer C is incorrect). None of the chemotherapeutic agents in Answer D are used at the start of induction (Answer D is incorrect). Patients should also receive intrathecal therapy at the start of treatment.)

A 46-year-old woman presents to the primary clinic with a 2-week history of difficulty sleeping. She is having trouble falling asleep, often lying in bed for up to 2 hours before onset of sleep and awakening several times during the night. She states that she has always been a "light sleeper," but her current difficulties are new. She is a single mother of two teenaged boys and works as an office secretary. She was just told that her work hours may be decreased, and she has limited prescription drug coverage through her employer. Her medical history includes migraine headaches, hypercholesterolemia, and obesity. Her current vital signs are blood pressure 140/86 mm Hg and heart rate 84 beats/minute. Her home drugs include ibuprofen 400 mg orally every 6 hours as needed for headache, simvastatin 40 mg orally daily, and sumatriptan 50 mg orally as needed for headache (she has taken two doses in the past 7 days). She does not currently have headache pain. She states that she has tried to sleep in a calm environment, goes to bed at the same time each night, and has tried over-the-counter remedies with no effect. She does not drink alcohol and smokes (1/2 pack/day). She needs to be awake by 5:30 a.m. daily for her family and work. Which one of the following is best to recommend for this patient? A. Trazodone 50 mg orally at bedtime. B. Zolpidem 5 mg orally at bedtime. C. Flurazepam 30 mg orally at bedtime. D. Ramelteon 8 mg orally at bedtime.

Answer B: Zolpidem 5 mg orally at bedtime. (Zolpidem is a good choice for initiating sleep and is available as a generic, which this patient needs because she has limited prescription coverage. She has no substance-abuse history other than tobacco use, so abuse of zolpidem is not a current concern (Answer B is correct). Trazodone is also available generically and is commonly used for sedation; however, it may cause daytime sedation, which could be problematic for this patient because she has children (Answer A is incorrect). Flurazepam is not commonly used and carries a risk of daytime sedation (Answer C is incorrect). Ramelteon would aid in falling asleep, and it is not a controlled substance; however, it is available only as a brand-name product and is more expensive (Answer D is incorrect).)

A medical resident has requested information on a recent news story regarding the depletion of magnesium by proton pump inhibitors. She requests more information about the clinical presentation as well as the incidence of this ADR in various patient populations. Which one of the following would be the best source to find this information? A. www.clinicaltrials.gov B. www.fda.gov C. www.mayoclinic.com D. www.clinicalevidence.com

Answer B: www.fda.gov (The FDA Web site is correct because it has the most current ADR information that is either reported by manufacturers or detected through the pharmacovigilance program MedWatch (Answer B is correct). The site also provides e-mail alerts to clinicians when new ADRs are reported. The Clinical Trials Web page is not the best answer because this site is for registered clinical trials, and no information on this ADR would be available unless a clinical trial were conducted on the subject (Answer A is incorrect). The ADR reports are not indexed in clinicaltrials.gov. The Mayo Clinic Web site is meant for health care consumers versus information for the health care professional, and the latest ADRs are not necessarily discussed on the site (Answer C is incorrect). The Clinical Evidence Web site provides guidelines and does not have the most current safety data (Answer D is incorrect).)

A 60-year-old man is brought to the emergency department by ambulance for public intoxication. The resident orders an intravenous fluid of dextrose 5% in water to run continuously at 100 mL/hour to begin rehydrating the patient. Which one of the following is best to recommend for this patient? A. 400 mg of folic acid orally. B. 1 mg of lorazepam intravenously. C. 100 mg of thiamine intramuscularly. D. 10 mg of haloperidol intravenously.

Answer C: 100 mg of thiamine intramuscularly. (Electrolyte imbalances and nutritional deficiencies are common in patients with alcoholism. Administering dextrose intravenously to this patient before administering thiamine could precipitate Wernicke triad. Therefore, administering thiamine is the best initial step (Answer C is correct). Folic acid, although often supplemented in patients with alcoholism, would not help prevent the potential irreversible effects of Wernicke-Korsakoff syndrome in this patient (Answer A is incorrect). Although benzodiazepines are used for the prevention of withdrawal seizures, it is imperative to administer the thiamine dose before the rehydration with a dextrose-based solution (Answer B is incorrect). There is no indication for haloperidol administration in this patient (Answer D is incorrect).)

A 58-year-old man (height 172 cm, weight 68 kg) presents to the ED with a new left femoral DVT. He denies any recent surgery, immobility, prior VT E, smoking, inherited thrombophilias, or other known VT E risk factors. His medical history includes HTN and osteoarthritis. He takes enalapril 10 mg twice daily and acetaminophen 1000 mg twice daily. Warfarin is initiated at 5 mg orally every evening, and enoxaparin is bridged at I mg/kg subcutaneously every 12 hours until a therapeutic INR is attained. Which one of the following would be best to recommend regarding the duration of anticoagulation therapy for this patient? A. 6 weeks; then discontinue anticoagulation. B. 3 months; longer if a hypercoagulable workup is positive. C. 3 months minimum; then reevaluate risk-benefit ratio and consider long-term anticoagulation if no contraindications. D. 6-12 months; then discontinue anticoagulation.

Answer C: 3 months minimum; then reevaluate risk-benefit ratio and consider long-term anticoagulation if no contraindications. (Current recommendations regarding idiopathic DVT include 3 months of treatment, with reevaluation after 3 months, and consideration of long-term anticoagulation if no contraindications are present (Answer C is correct). A therapy duration of 6 weeks is not long enough and could result in an increased risk of VT E recurrence (Answer A is incorrect). Current recommendations suggest that a hypercoagulable workup is unnecessary; results from these tests do not help predict the risk of VT E recurrence (Answer B is incorrect). Discontinuing anticoagulation after 6-12 months provides a longer initial duration than is recommended by the American College of Chest Physicians for unprovoked first thromboembolic event (Answer D is incorrect).)

A study was designed to investigate the benefits of a new antiplatelet agent in the prevention of myocardial infarction in patients with a history of long-standing coronary artery disease. The rate of myocardial infarction in the treatment group was 12%, and the rate in the control group was 18%. Which one of the following best describes the relative risk reduction (RRR) derived from the treatment group? A. -6% B. 6% C. 33% D. 67%

Answer C: 33% (The RRR is calculated as the control event rate (CER) minus the experimental event rate (EER) divided by the CER. In this case, 0.18 — 0.12/0.18 =0.33, or 33% (Answer C is correct). The option —6% is EER minus CER (Answer A is incorrect). The option 6% represents the absolute risk reduction (ARR), which is calculated as CER minus EER (Answer B is incorrect). The option 67% represents the relative risk, which is calculated as EER divided by CER (Answer D is incorrect).)

A 34-year-old man (weight 86 kg [189.5 lb]) with type 2 diabetes mellitus (T2DM) and an AIC of 10.2% is being converted from oral diabetes drug therapy to basal/bolus insulin with insulin glargine and insulin aspart. His endocrinologist wants to start total daily insulin (TDI) at 0.5 unit/kg/day. Which one of the following is the best initial dose estimate of rapid-acting insulin aspart before breakfast for this patient? A. 2 units. B. 4 units. C. 7 units. D. 14 units.

Answer C: 7 units. (This patient's TDI requirements equal 43 units (86 kg x 0.5 unit/kg/day). Half of this TDI is initially used for basal insulin requirements and half for bolus insulin requirements before meals. The 21 units for bolus requirements should initially be divided equally between three meals, which equals 7 units per meal (Answer C is correct). The other three options provide either too much or too little estimated insulin at each meal. A bolus dose of either 2 units or 4 units is unlikely to provide enough insulin at mealtime and could lead to hyperglycemia (Answer A and Answer B are incorrect). A bolus dose of 14 units has the potential to cause hypoglycemia because it could provide too much insulin (Answer D is incorrect).)

An 82-year-old woman (height 6'2", weight 46 kg [101 lb]) who resides in a nursing home has advanced AD. She is no longer ambulatory and requires assistance with feedings. She also has a stage 2 pressure ulcer around her sacral area. Which one of the following targeted daily protein intakes will best help improve the healing of this patient's pressure ulcer? A. 30 g. B. 50 g. C. 70 g. D. 100 g.

Answer C: 70 g. (Individuals receiving a disease-specific supplement that increases protein to about 20% of calories have experienced better wound healing than individuals who received other types of high-calorie supplementation. The recommended amount of protein to aid with this patient's wound healing is around 1.5 g/kg/day (20% of total calories), which is 70 g (Answer C is correct). Daily protein intakes of 30 g, 50 g, or 100 g are inappropriate for this patient because these amounts would provide her with 8%, 14%, or 29% of total daily calories as protein (Answer A, Answer B, and Answer D are incorrect).)

A 63-year-old man with androgen-independent prostate cancer has significant pain from bone metastases and requires opioids on a daily basis. He has been taking sustained-release morphine 130 mg orally every 12 hours and immediate-release morphine 20—30 mg every 4 hours for breakthrough pain. The patient now has trouble swallowing tablets and requests an alternative opioid for his pain management. The physician requests assistance in changing him to a fentanyl patch maintenance dose equivalent to his current long-acting morphine plus a buccal dosing form for breakthrough pain. Which one of the following fentanyl regimens is best to recommend for this patient? A. A 100-mcg/hour patch plus buccal lozenges 200 mcg for breakthrough pain. B. A 75-mcg/hour patch plus 200 mcg as either effervescent tablets or buccal lozenges for breakthrough pain. C. A 75-mcg/hour patch plus effervescent tablets 100 mcg for breakthrough pain. D. A 50-mcg/hour patch plus 200 mcg as either effervescent tablets or buccal lozenges for breakthrough pain.

Answer C: A 75-mcg/hour patch plus effervescent tablets 100 mcg for breakthrough pain. (A 75-mcg/hour fentanyl patch is equivalent to scheduled oral morphine 260 mg/day, and fentanyl 100-mcg effervescent tablets are an acceptable starting dose for this patient's level of need because his current morphine dose of 20—30 mg is relatively modest, and the equivalent fentanyl oral dose is uncertain. The patient should be started at the lower dose and titrated to good analgesic effect with acceptable adverse effects (Answer C is correct). The dose with a 50-mcg/hour fentanyl patch is too low (Answer D is incorrect), whereas the dose with a 100-mcg/hour fentanyl patch is too high (Answer A is incorrect), and these doses are likely to provide insufficient analgesia or excessive adverse effects, respectively. Fentanyl effervescent tablets and buccal lozenges are not bioequivalent; the former can be partly consumed (Answer B is incorrect), whereas the latter provides the complete dose, and both require titration to find the correct dose once the starting dose is ascertained (Answer D is incorrect).)

A 20-year-old woman (height 65 inches, weight 99 kg [219 lb]) presents to your clinic for an initial visit. Her BMI (body mass index) has always been high, but she has gained almost 40 lb since starting college 3 years ago. She has a diet rich in convenience and snack foods. She drinks several alcoholic beverages on most weekends and does not smoke. She played soccer competitively before college but no longer engages in regular physical activity. Her father is overweight and has T2DM and HTN. The patient denies polyuria, polydipsia, blurred vision, changes in her skin or hair, changes in her menses, or cold intolerance. Her blood pressure today is 148/92 mm Hg. Which one of the following would best manage obesity in this patient? A. Phentermine 15 mg three times/day with meals plus a low-carbohydrate diet. B. Diethylpropion controlled release 75 mg every morning plus a calorie-restricted diet with a calorie deficit of 500 kcal/day, less than 10% calories from saturated fat, and limited refined sugars. C. A diet with a calorie deficit of 500 kcal/day, less than 30% calories from fat, and limited refined sugars, combined with a progressive exercise program. D. An exercise prescription for 60 minutes of soccer five times/week with a plan to increase to every day.

Answer C: A diet with a calorie deficit of 500 kcal/day, less than 30% calories from fat, and limited refined sugars, combined with a progressive exercise program. (In general, lifestyle changes should be tried before drug therapy is initiated for weight loss (Answer A and Answer B are incorrect). In addition, this patient's poorly controlled blood pressure makes these drug therapies poor choices. A multifaceted approach is the most effective means of treating obesity (Answer C is correct). An exercise prescription may be reasonable, but relying completely on exercise for weight reduction in a patient who finds little time for regular physical activity is unrealistic (Answer D is incorrect).)

A 54-year-old woman has stage 5 CKD secondary to diabetic nephropathy and has been on hemodialysis for the past 4 years. Her most recent laboratory values are calcium 9.0 mg/dL, phosphorus 5.1 mg/dL, albumin 2.8 g/dL, and iPTH 882 pg/mL. She is currently taking calcium acetate 1334 mg by mouth three times/day with meals and doxercalciferol 4 mcg intravenous push three times/week at the end of hemodialysis. She has never been prescribed aluminum-containing phosphate binders. Which one of the following would best manage this patient's CKD mineral and bone disorder? A. Discontinue calcium acetate and add sevelamer 403 mg with meals. B. Discontinue calcium acetate and add lanthanum 500 mg with meals. C. Add cinacalcet 30 mg by mouth once a day. D. Increase doxercalciferol to 6 mcg intravenously three times/week.

Answer C: Add cinacalcet 30 mg by mouth once a day. (This patient has secondary hyperparathyroidism and a high corrected serum calcium concentration (9.96 mg/dL). She is at risk of osteitis fibrosa cystica and extraskeletal calcification. Her phosphorus concentration is controlled with calcium acetate therapy; however, despite the use of doxercalciferol, her iPTH level is still high (greater than 300 pg/mL for stage 5 CKD). Changing calcium acetate to sevelamer or lanthanum may be beneficial to prevent extra calcium intake but will not improve the elevated iPTH level (Answer A and Answer B are incorrect). Discontinuing calcium acetate would prevent further hypercalcemia, but the phosphorus level could rise if not controlled. Increasing the doxercalciferol dose would better control the iPTH level, but it might further increase the calcium concentration by increasing intestinal calcium absorption (Answer D is incorrect). Therefore, adding cinacalcet is the best option because it will decrease the iPTH level (in addition to the effect from doxercalciferol) and may also lower the calcium concentration because of the modulation of the calcium-sensing receptor on the parathyroid gland (Answer C is incorrect).)

A 23-year-old man receives a diagnosis of schizophrenia and is initiated on risperidone 3 mg/day. In the following weeks, he stops taking his medications and decompensates. Now, he is again admitted to the hospital because he reports that voices are telling him that he should hang himself. He is reinitiated on risperidone 3 mg daily, with the addition of lorazepam I mg every 4—6 hours as needed for anxiety. The patient responds to therapy, and his risperidone dose is subsequently titrated to 6 mg/day with resolution of his psychotic symptoms. Today, the nursing staff report that the patient has become more restless and unable to sit still for long periods. He has been observed pacing around the unit with a sullen expression. During patient interviews, he often stands up from his chair and taps his feet. Which one of the following is best to recommend for this patient? A. Increase the risperidone dose. B. Add benztropine I mg twice daily. C. Add propranolol 10 mg twice daily. D. Discontinue risperidone; initiate haloperidol.

Answer C: Add propranolol 10 mg twice daily. (This patient has symptoms of akathisia because of the increased risperidone dose. Propranolol is the treatment of choice for akathisia if the antipsychotic agent is to be continued (Answer C is correct). Risperidone is known to cause akathisia, especially at doses above 6 mg/day. This patient's dose was already increased to 6 mg/day, so a further increase would likely worsen this adverse effect (Answer A is incorrect). Although benztropine may help alleviate other forms of extrapyramidal symptoms, it can worsen akathisia; therefore, it is inappropriate (Answer B is incorrect). Finally, although changing to a different antipsychotic is an acceptable option to treat akathisia, haloperidol is an inappropriate choice because it is a potent conventional agent known to cause akathisia and other types of extrapyramidal symptoms (Answer D is incorrect).)

A 63-year-old man is admitted to the hospital with active HZ, as evidenced by erythematous eruptions and ulcerative lesions of the skin. He is initiated on valacyclovir 1 g three times/day and discharged on a 7-day course. He follows up with his primary care physician 14 days after discharge and requests vaccination against future HZ episodes. The patient's lesions are resolved, and he notes no symptoms of neuralgia. Which one of the following HZ vaccination plans is best for this patient? A. Administer the vaccine at least 14 days after antiviral therapy is completed. B. Vaccination is unnecessary; the patient has already developed active immunity. C. Administer the vaccine to the patient at this follow-up visit. D. Administer the vaccine to the patient in 2 years, when he is 65 years of age.

Answer C: Administer the vaccine to the patient at this follow-up visit. (For an active HZ infection, vaccination should be deferred until at least 24 hours after the treatment therapy is completed, the acute stage of the illness is over, and symptoms have abated. This patient presents to his physician 7 days after completing his antiviral regimen with no complaints of neuralgia, and there are no noted lesions; therefore, he may be given the HZ vaccine at this time (Answer C is correct, and Answer A is incorrect). An HZ vaccination is FDA approved for individuals 50 years and older; however, the ACIP recommends an HZ vaccine for individuals 60 years or older, regardless of their history of HZ (Answer B and Answer D are incorrect).)

You work in an ambulatory care clinic and are responsible for educating prescribers on the revised preventive guidelines for meningitis. Which one of the following statements about the use of the meningococcal vaccine is best to include in the education campaign? A. The vaccine should be offered to all children. B. High-risk children younger than 2 years cannot receive the conjugate vaccine. C. All adolescents 11—13 years of age should receive the vaccine. D. Children undergoing a cochlear implant procedure should receive a dose before surgery.

Answer C: All adolescents 11—13 years of age should receive the vaccine. (The updated ACIP guidelines call for vaccination of all adolescents between 11 and 13 years of age (Answer C is correct). The meningococcal conjugate vaccine (MCV4; Menactra) is labeled for use in individuals 9 months to 55 years of age, and MCV4 (Menveo) is labeled for those 2 months to 55 years. The ACIP does not recommend routine vaccination in all children, but it does recommend vaccination for high-risk children with complement deficiencies, asplenia, travel to certain areas, and community outbreaks (Answer A is incorrect). New recommendations have included the use of MCV4 in high-risk infants older than 9 months (Answer B is incorrect). Children undergoing a cochlear implant are at increased risk of pneumococcal disease, not meningococcal disease, and do not require vaccination (Answer D is incorrect).)

Currently, GLP-I (glucagon-like peptide-I) analogs and incretin mimetics have Risk Evaluation and Mitigation Strategies (REMS) mandated by the FDA. Which of the following best describes the primary purpose of REMS? A. To restrict access because these medications may be associated with harmful effects. B. To create a patient registry for these medications. C. To facilitate collaboration between different disciplines involved in patient care. D. To ensure that the benefits of these agents outweigh their risks.

Answer D: To ensure that the benefits of these agents outweigh their risks. (The intent of REMS is not to restrict access to medications (Answer A is incorrect). Some REMS programs do require patient registries or other elements of safe use such as interdisciplinary collaboration, but many REMS do not require these elements (Answer B and Answer C are incorrect). The primary goal of REMS is to ensure that drug benefits outweigh risks (Answer D is correct).)

A 64-year-old postmenopausal woman with a recent diagnosis of stage IIA breast cancer has a medical history significant for type 2 diabetes mellitus, deep venous thrombosis (DVT), and osteopenia (T-score of -1). Which one of the following would be the best endocrine option for this patient? A. Tamoxifen for 5 years plus monthly zoledronic acid injection. B. Tamoxifen for 2 years, followed by letrozole for an additional 3 years. C. Anastrozole for 5 years plus monthly ibandronate. D. Goserelin for 2 years, followed by anastrozole for 3 years.

Answer C: Anastrozole for 5 years plus monthly ibandronate. (Anastrozole has significantly lower risk of thromboembolic events compared with tamoxifen in this patient, who has stage II breast cancer and a history of osteopenia (Answer C is correct). Furthermore, her osteopenia can be managed by initiating a bisphosphonate when anastrozole therapy is begun. Data show that this strategy is effective in minimizing aromatase inhibitor—associated bone loss. Tamoxifen can cause thromboembolic events in women (especially in those older than 50 years), and it should be avoided in this patient with a history of DVT. Zoledronic acid is indicated for osteoporosis but is injected annually, not monthly (Answer A is incorrect). Tamoxifen for 2 years, followed by letrozole for an additional 3 years, is incorrect because it places the patient at risk of thromboembolic events, despite the change to an aromatase inhibitor (Answer B is incorrect). Goserelin is indicated for premenopausal patients only (Answer D is incorrect).)

A 35-year-old woman undergoes routine Papanicolaou (Pap) smear testing for cervical cancer prevention. Results show atypical cells of unknown significance. Her gynecologist recommends a repeat Pap smear in 6 months, together with human papillomavirus (HPV) testing. Six months later, a repeat examination reveals a normal Pap smear and a negative HPV test. Which one of the following cervical cancer screening recommendations is most appropriate for the patient at this time? A. Pap smear plus HPV DNA testing every 6 months until three negative tests. B. Pap smear tests annually for life. No further HPV DNA testing needed. C. Annual Pap smears until three negative tests; then test every other year. HPV DNA test every 3 years. D. Pap smear and HPV DNA testing every 3 years.

Answer C: Annual Pap smears until three negative tests; then test every other year. HPV DNA test every 3 years. (Undergoing annual Pap smears and HPV DNA testing every 6 months is unnecessary, costly, and not recommended by current guidelines (Answer A is incorrect). The patient can undergo annual Pap smears, and after she has had three negative tests, testing can be conducted every 2-3 years instead of annually Human papillomavirus is an indolent virus, and most patients will clear it through their own immune system. In some patients, however, the virus will remain indolent and can cause the development of cervical dysplasia (or abnormal cells) that, if left untreated, can progress to cervical cancer. Testing for HPV should be conducted together with Pap smear testing every 3 years if the patient has three negative tests (Answer C is correct; Answer B is incorrect). Pap smear plus HPV DNA testing every 3 years is not appropriate at this time; a patient cannot start with this interval of testing until she has had three negative Pap smears (Answer D is incorrect).)

A 75-year-old man presents to the clinic for examination after a recent hospitalization for HF exacerbation. He was discharged 4 weeks ago with the following drugs: enalapril 10 mg twice daily, furosemide 40 mg twice daily, spironolactone 25 mg daily, and isosorbide dinitrate 20 mg three times daily. While hospitalized, his BP was 143/95 mm Hg, and his HR was 91 beats/minute. His medical history is significant for an MI (24 years ago), left ventricular dysfunction (LVD) (LVEF less than 35%), and carotid artery disease (CAD). The patient is adherent to his drug regimen and tries to remain active by walking with his wife. He does not any self-monitoring at home. Today in the clinic, vital signs and laboratory values include BP 134/84 mm Hg, HR 89 beats/minute, Na 147 mEq/L, K 4.6 mEg/L, SCr 0.9 mg/dL, and glucose 114 mg/dL. Which one of the following β-blockers would be best to recommend for this patient? A. Atenolol 25 mg daily. B. Metoprolol succinate 100 mg twice daily. C. Carvedilol 3.125 mg twice daily. D. Bisoprolol 5 mg daily.

Answer C: Carvedilol 3.125 mg twice daily. (Carvedilol has extensive clinical outcomes data for CAD and LVD, and this starting dose is appropriate for the patient (Answer C is correct). Although atenolol is the most widely studied agent for HTN treatment, it is not well established for patients with a history of HF (Answer A is incorrect). In addition, according to the 2007 Cochrane review, atenolol is not a favored β-blocker, particularly based on once-daily dosing. The suggested starting dose of metoprolol succinate is too high for this patient and might lead to an exacerbation (Answer B is incorrect). A better starting dose is 25—50 mg daily, in divided doses if necessary. Bisoprolol could be an option, but the starting dose of 5 mg daily might not be tolerated (Answer D is incorrect). A better starting dose for this patient is 1.25 mg daily.)

A 58-year-old man is admitted to the hospital for pneumonia. His medical history is significant for total knee replacement 21/2 weeks ago, which required a 4-day hospital stay. Which one of the following is the best empiric regimen for management of this patient's pneumonia? A. Levofloxacin. B. Ceftriaxone and azithromycin. C. Cefepime plus ciprofloxacin plus linezolid. D. Levofloxacin plus vancomycin.

Answer C: Cefepime plus ciprofloxacin plus linezolid. (Because of this patient's recent hospitalization, his pneumonia is considered health care—associated pneumonia. Health care—associated pneumonia is most appropriately empirically treated with an antipseudomonal ß-lactam plus a fluoroquinolone or aminoglycoside plus vancomycin or linezolid (depending on risk factors). Levofloxacin or ceftriaxone and azithromycin would be more appropriate for a CAP (Answer A and Answer B are incorrect). Levofloxacin plus vancomycin does not provide sufficient antipseudomonal coverage (Answer D is incorrect). Therefore, cefepime plus ciprofloxacin plus linezolid is the best option (Answer C is correct).)

A meta-analysis is undertaken to study the effects of an over-the-counter herbal preparation on blood pressure. The meta-analysis includes seven different randomized trials with sample sizes ranging from 36 patients to 428 patients. The results show a decrease in systolic blood pressure of 0.3 mm Hg (95% CI, 0—0.6 mm Hg) and a decrease in diastolic blood pressure of 0.4 mm Hg (95% CI, 0.1—0.7 mm Hg). In assessing the methods of this meta-analysis, which tool would be most useful to detect publication bias? A. Forest plot. B. Survival curve. C. Funnel plot. D. I^2 calculation.

Answer C: Funnel plot. (A funnel plot is a graphic tool used to assess the possibility of publication bias (Answer C is correct). The absence of the left side of the funnel plot suggests that smaller, negative studies have not been published, even though they were conducted. A Forest plot is used to display the overall meta-analysis results graphically, whereas a survival curve displays the time to an event (Answer A and Answer B are incorrect). The I^2 is a method of detecting heterogeneity between studies in a meta-analysis and does not address publication bias (Answer D is incorrect).)

A 35-year-old man presents to the hospital with a 2-day history of sudden left knee pain. He has been experiencing fevers, pain, swelling, and limited mobility in his knee. His medical history includes gout, hypertension, and osteoarthritis. The patient works as a carpenter and assumes that he injured his knee while at work. After an arthroscopy and drainage of his left knee, the synovial fluid analysis shows a WBC of 55 x 10^3 cells/mm3, neutrophils 84%, and no crystals. A Gram stain is pending. Which one of the following is the best empiric treatment for this patient? A. Ceftriaxone. B. Colchicine. C. Ceftriaxone plus vancomycin. D. Vancomycin.

Answer C: Ceftriaxone plus vancomycin. (Empiric therapy for patients presenting with monoarticular septic arthritis should include therapy for both Staphylococcus aureus and Neisseria gonorrhoeae. Ceftriaxone and cefotaxime are drugs of choice for N. gonorrhoeae. Because methicillin-resistant S. aureus (MRSA) rates are high in the community, vancomycin should be added until MRSA is ruled out. Therefore, ceftriaxone plus vancomycin is the best choice (Answer C is correct). Ceftriaxone alone is inadequate to treat MRSA (Answer A is incorrect). Therapy for this patient should include treatment of N. gonorrhoeae, even though his sexual history is unknown, so vancomycin alone is not the best choice (Answer D is incorrect). Although he has a history of gout, the synovial fluid analysis indicates that infection is more likely given a WBC of 55 x 10^3 cells/mm^3 with 84% neutrophils and no crystals, so colchicine treatment for gout is not needed (Answer B is incorrect). Once a Gram stain and culture are available, therapy can be tailored.)

A 38-year-old man presents to the clinic with the chief concerns of dysuria and purulent penile discharge. A culture confirms the presence of N. gonorrhoeae with the following MICs: azithromycin 2 mcg/mL, cefixime 2 mcg/mL, ceftriaxone 0.25 mcg/mL, and ciprofloxacin 0.25 mcg/mL. Which one of the following is the most appropriate treatment option for this patient? A. Azithromycin. B. Cefixime. C. Ceftriaxone. D. Ciprofloxacin.

Answer C: Ceftriaxone. (Single doses of cefixime or ceftriaxone are the recommended regimens for treatment of uncomplicated gonococcal infections. In this case, ceftriaxone is the preferred choice because a cefixime MIC of 0.5 mcg/mL or more suggests decreased susceptibility (Answer C is correct; Answer B is incorrect). Fluoroquinolones are no longer recommended for the treatment of gonococcal infections (Answer D is incorrect). In addition, this isolate showed intermediate resistance to ciprofloxacin (MIC 0.125—0.5 mcg/mL). Although azithromycin may be effective for the treatment of uncomplicated gonorrhea infections, use of this therapy is not recommended because of concerns about rising resistance with overuse (Answer A is incorrect). An azithromycin MIC of 2 mcg/mL, as in this case, is suggestive of decreased susceptibility.)

A 72-year-old man presents with uncontrolled symptoms of BPH and new complaints of ejaculatory dysfunction. He has been taking tamsulosin 0.4 mg/day for I month and finasteride 5 mg/day for 6 months. Which one of the following is best to recommend for this patient? A. Increase tamsulosin to 0.8 mg/day. B. Increase finasteride to 10 mg/day. C. Change tamsulosin to alfuzosin. D. Change finasteride to dutasteride.

Answer C: Change tamsulosin to alfuzosin. (The patient needs further improvement of his BPH and ejaculatory symptoms, and changing to alfuzosin is likely improve his BPH symptoms and may have less potential to cause ejaculatory dysfunction as an adverse effect (Answer C is correct). Increasing the tamsulosin dose could improve his BPH symptoms but not his ejaculatory dysfunction because tamsulosin may be the cause of his ejaculatory dysfunction (Answer A is incorrect). Increasing the finasteride dose is not indicated for any situation (Answer B is incorrect). Changing finasteride to dutasteride would probably not affect his BPH symptoms or ejaculatory dysfunction because the mechanism of action and adverse effects of these agents are very similar. Both finasteride and dutasteride can cause ejaculatory disorders (Answer D is incorrect).)

A 36-year-old woman has received a diagnosis of premenstrual dysphoric disorder (PMDD) characterized by severe mood-related symptoms. The patient has a history of migraines, diet-controlled diabetes, and hypertension. Her current drugs include sumatriptan and hydrochlorothiazide. She smokes (I pack/day) and drinks coffee (5—6 cups/day). Which one of the following drug regimens is best for the initial management of this patient's symptoms? A. Bupropion 150 mg orally two times/day continuously throughout the menstrual cycle. B. Paroxetine 20 mg/day orally dosed intermittently during the luteal phase. C. Citalopram 20 mg/day orally dosed continuously throughout the menstrual cycle. D. Fluoxetine 60 mg/day orally dosed intermittently during the luteal phase.

Answer C: Citalopram 20 mg/day orally dosed continuously throughout the menstrual cycle. (Antidepressants are considered first-line therapy for patients suffering from PMDD because serotonin dysregulation is postulated as a cause of PMDD. Therefore, antidepressant drugs with serotonergic activity are preferred to antidepressants targeting other neurotransmitters, such as norepinephrine and dopamine. The selective serotonin reuptake inhibitors (SSRIs) can usually be dosed continuously or intermittently in women with PMDD. However, paroxetine has a short half-life, and only the controlled-release formulation should be used intermittently (Answer B is incorrect). Fluoxetine is effective for PMDD; however, the dose of 60 mg/day is not more effective than 20 mg/day and is associated with more adverse effects (Answer D is incorrect). Bupropion, a dopaminergic antidepressant, is ineffective for the treatment of PMDD symptoms, making it an inappropriate choice for the treatment of this patient's symptoms (Answer A is incorrect). Citalopram is effective at a dosage of 20 mg/day; therefore, it would be the preferred agent for this patient (Answer C is correct).)

A family medicine physician seeks your advice on strategies to decrease the development of antibiotic resistance in patients treated for acne vulgaris. Which of the following therapeutic strategies should be recommended to your colleague as the most likely to reduce the development of antibiotic resistance in his clinic population? A. Take oral antibiotics every other day for the therapeutic course. B. Alternate oral antibiotic classes every 4 weeks for the therapeutic course. C. Combine oral antibiotics with topical benzoyl peroxide. D. Combine oral antibiotics with adapalene.

Answer C: Combine oral antibiotics with topical benzoyl peroxide. (The use of benzoyl peroxide with oral antibiotics helps to decrease the incidence of antibiotic resistance when treating patients for acne (Answer C is correct). The oral antibiotics can be used with adapalene for treatment, but it will not change the likelihood of resistance (Answer D is incorrect). Altering the dosing schedule or using sequences of oral antibiotic classes is not encouraged and will not decrease resistance patterns (Answer A and Answer B are incorrect).)

A 49-year-old man was prescribed allopurinol 100 mg/day and colchicine 0.6 mg twice daily a few days ago because of three episodes of gout in the past year. The patient's recent laboratory values include a serum uric acid concentration of 9.3 mg/dL, SCr 1.1 mg/dL, AST 38 U/L, and ALT 32 U/L. He presents to your clinic with redness, swelling, and intense pain in his right foot for about 12 hours. He tells you that he never filled his prescription for colchicine at the pharmacy. He states: "I didn't understand why I needed two drugs, so I only filled allopurinol. Now I wonder if I should even take allopurinol because it isn't working." Which one of the following is the best intervention for this patient? A. Discontinue allopurinol and start naproxen. B. Discontinue allopurinol and start colchicine. C. Continue allopurinol and start indomethacin. D. Continue allopurinol and start a corticosteroid.

Answer C: Continue allopurinol and start indomethacin. (Although allopurinol may cause a gout flare when initiated of the mobilization of urate patients should be educated to continue taking allopurinol during acute attacks as long as appropriate anti-inflammatory medications are initiated (Answer A and Answer B are incorrect) Allopurinol should be continued but an NSAID (indomethacin) should added to treat the pain and inflammation of the acute attack (Answer C is correct). Normally. urate-lowering therapy is not initiated during a short-term attack; however, because this patient has already started taking allopurinol, it should be Although the corticosteroid could treat the acute attack, it is not tor preventing attacks by initiating allopurinol (Answer D is incorrect).)

A 53-year-old woman with chronic hepatitis B virus (HBV) infection comes to the hepatology clinic for follow-up. Results of laboratory tests completed 3 months ago were HBsAg positive, HBeAg negative, HBV DNA negative, ALT 23 IU/L, and SCr 0.8 mg/dL. Results of laboratory tests performed 2 weeks before her current appointment were HBsAg positive, HBeAg negative, HBV DNA 21,000 IU/mL, and ALT 260 IU/L. The patient's current antiviral treatment, telbivudine 600 mg daily, was started 2 years ago. Her hepatologist suspects genotypic resistance to telbivudine. While awaiting the results of genotypic resistance testing, which one of the following is best to recommend for this patient? A. Continue telbivudine and add lamivudine 100 mg daily. B. Discontinue telbivudine and start entecavir 0.5 mg daily. C. Continue telbivudine and add tenofovir disoproxil fumarate 300 mg daily. D. Continue telbivudine and add adefovir dipivoxil 30 mg daily.

Answer C: Continue telbivudine and add tenofovir disoproxil fumarate 300 mg daily. (This patient has been treated with telbivudine 600 mg daily for 2 years for chronic HBV infection. In the past 3 months, her HBV DNA has increased from negative to 21,000 IU/dL, and her ALT concentration has increased from 23 IU/L to 260 IU/L. Telbivudine resistance should be suspected as the cause of her recent biochemical and virologic breakthrough. Because genotypic mutations conferring telbivudine resistance are similar to those with lamivudine, adding lamivudine to her current therapy is unlikely to be effective (Answer A is incorrect). Because telbivudine resistance predisposes to entecavir resistance, changing to entecavir is not an appropriate treatment option (Answer B incorrect). In patients with suspected telbivudine resistance, tenofovir is an acceptable therapy option to manage resistance while awaiting the results of genotype testing (Answer C is correct). Although adefovir is a potential option for treating patients with telbivudine-resistant HBV infection, the dosage listed exceeds the current recommended dosage (10 mg/day) and could cause nephrotoxicity (Answer D is incorrect).)

A 78-year-old nursing home resident is receiving phenytoin chewable tablets 300 mg once daily for seizures. Her recent total phenytoin serum concentration, measured about 4 hours after a dose, was 16 mg/L. Two months earlier, her phenytoin serum concentration was 8 mg/L when measured about 3 hours before her same 300-mg dose. The physician is concerned about the most recent concentration. Her medication administration record notes no issue of missed doses or problems related to adherence in the past 3 months. The patient has had no documented seizures during the past year, and no adverse effects were noted by patient or clinical care staff. Which one of the following options would be best for this patient? A. Audit patient adherence. B. Decrease the phenytoin dose to 250 mg/day. C. Continue the current phenytoin dose. D. Change the phenytoin dose to 150 mg twice daily.

Answer C: Continue the current phenytoin dose. (Because of the Michaelis-Menten pharmacokinetics of phenytoin, the half-life is variable in relation to the serum concentration. Previous work has determined the half-life of phenytoin to be about 6—24 hours. However, these studies usually used lower serum concentrations of phenytoin. A recent study at doses and serum concentrations in the range this patient is receiving showed the half-life of phenytoin to be an average of 40 hours. With this long half-life, dosing more than once daily is unnecessary (Answer D is incorrect). Studies of nursing home patients show that phenytoin concentrations can fluctuate greatly, even when drawn at similar times around a dose and with consistent adherence (Answer C is correct). It is unlikely that patient adherence is playing a role in the fluctuation observed because medication administration is observed and recorded in her current setting (Answer A is incorrect). The patient is not showing clinical signs of toxicity, and this amount of fluctuation is common in this patient population, so decreasing the dose is not necessary (Answer B is incorrect).)

A 54-year-old man with a history of coronary artery disease, COPD, hypertension, and hyperlipemia presents to the clinic for a follow-up. His current home drugs include metoprolol, lisinopril, simvastatin, albuterol/ipratropium nebulizers, and aspirin. The consulting primary care physician is concerned about the use of β-blockers in this patient because of the possibility of bronchospasm. Which one of the following best addresses the physician's concern? A. Discontinue metoprolol: replace With carvedilol. B. Discontinue metoprolol and replace with a calcium channel blocker. C. Continue metoprolol and reassure the primary care physician D Increase the use of inhaled albuterol to counter the β-blockade from metoprolol.

Answer C: Continue to metoprolol and reassure the primary care physician (Studies show substantial benefits of cardioselective β-blockers in patients with COPD for the treatment of cardiac disease (Answer C is correct). However, non-cardioselective β-blockers such as carvedilol are best avoided in COPD, or if initiated, respiratory status should be monitored closely (Answer A is incorrect). Of note, β-blockers, even cardioselective agents are more likely to result in bronchoconstriction in patients with asthma than in patients with COPD. Many physicians have been taught globally that these agents should not be used in asthma, and this information is often extrapolated to COPD, leading to the underuse of a drug class that has clear mortality benefits in cardiac disease. Discontinuing metoprolol without evidence of significant worsening of respiratory status is likely to increase the patient's mortality risks from his coronary artery disease (Answer A and B are incorrect). It is possible that higher doses of β-agonists may be needed in the presence of non-selective β-blockers such as carvedilol, but this need is less likely with a cardioselective agent such as metoprolol in a patient with COPD (Answer D is incorrect).)

A patient who started once-weekly exenatide long-acting release in addition to metformin is thought to have developed thyroid carcinoma. Which one of the following is most appropriate for this patient? A. Change exenatide to liraglutide because liraglutide has a lower risk of thyroid carcinoma. B. Continue current therapy and refer the patient to an oncologist for further evaluation. C. Discontinue exenatide and notify the exenatide manufacturer and the U.S. Food and Drug Administration (FDA) of a potential adverse event. D. Obtain a serum calcitonin concentration and refer the patient to an endocrinologist.

Answer C: Discontinue exenatide and notify the exenatide manufacturer and the FDA of a potential adverse event. (Although thyroid carcinoma has not been reported in clinical trials, it has been noted in animal studies with both liraglutide and once-weekly exenatide. Exenatide is contraindicated in patients with a personal or family history of medullary thyroid carcinoma; therefore, discontinuing the drug and notifying the manufacturer and the FDA is the most appropriate option listed (Answer C is correct). Given the potential risk with liraglutide and the lack of clinical information comparing the risk of these two agents, a change from exenatide to liraglutide is not appropriate (Answer A is incorrect). Given the significance of the possible adverse event, continuing the current therapy is not desirable (Answer B is incorrect). The utility of serum calcitonin concentrations is not known at this time (Answer D is incorrect).)

A 9-year-old boy with attention-deficit/hyperactivity disorder (ADHD), combined type, has been taking methylphenidate immediate release (IR) 10 mg orally in the morning and after school. His appetite is unchanged; however, he notes difficulty falling asleep at night. His teachers report that he is disruptive during his afternoon classes. Which one of the following is best to recommend for this patient? A. Increase methylphenidate IR to 15 mg in the morning and after school. B. Switch to dextroamphetamine 10 mg orally twice daily. C. Discontinue methylphenidate IR; start mixed amphetamine salts extended release 30 mg/day. D. Discontinue methylphenidate IR; start methylphenidate oral osmotic 36 mg orally daily.

Answer C: Discontinue methylphenidate IR; start mixed amphetamine salts extended release 30 mg/day. (Methylphenidate IR is only partly effective for this patient and is not lasting throughout the school day. An increase in dose will not lengthen the time of symptom response (Answer A is incorrect). Use of dextroamphetamine also requires twice-daily dosing (Answer B is incorrect). Switching to methylphenidate oral osmotic (Concerta) may exacerbate the insomnia this patient is experiencing because of the 12- to 18-hour duration of action (Answer D is incorrect). Use of amphetamine, dextroamphetamine mixed salts extended release (Adderall XR) will allow a once-daily dose that will last through the day (Answer C is correct).)

A 3-month-old infant (born at 29 weeks' gestation) has been hospitalized since birth. She has been receiving omeprazole 1.5 mg/kg orally every morning and metoclopramide 0.2 mg/kg orally every 6 hours for the management of gastroesophageal reflux disease (GERD). She is currently asymptomatic and will be discharged in 2 days. Which one of the following is the best regimen to recommend for this patient? A. Continue the patient's current therapy. B. Continue omeprazole at the current dosage and decrease the metoclopramide dosage to 0.1 mg/kg orally every 6 hours. C. Discontinue metoclopramide now and follow the patient on omeprazole alone before discharge. D. Discontinue metoclopramide and increase the omeprazole dosage to 1.5 mg/kg given orally two times/day.

Answer C: Discontinue metoclopramide now and follow the patient on omeprazole alone before discharge. (The NASPGHN/ESPCHN guidelines do not recommend metoclopramide for the management of GERD in pediatric patients because there is a lack of clinical evidence supporting its use. In addition, neonates are at the highest risk of adverse effects with metoclopramide (Answer A and Answer B are incorrect). The current NASPGHAN/ESPGHAN guidelines recommend PPI therapy dosed once daily in the morning before the first meal of the day for the management of GERD in children (Answer C is correct). The guidelines provide little support for the use of twice-daily PPIs, and a dosage increase would be indicated only if the patient were experiencing increased or recurrent symptoms (Answer D is incorrect).)

A 72-year-old woman with a medical history of hypertension (HTN) (10 years) and type 2 diabetes mellitus (T2DM) (12 years) recently received a diagnosis of moderate Alzheimer disease (AD). Her drugs include aspirin 81 mg/day, lisinopril 40 mg/day, glipizide extended release 10 mg/day, and metformin 1000 mg/day. She lives with her daughter, who is her primary caretaker. The patient denies any alcohol, tobacco, or illicit drug use. When she received the AD diagnosis last week, her physician initiated rivastigmine 1.5 mg twice daily for moderate disease. Since then, the patient reports being awakened by "very real dreams almost every night." Her daughter has not witnessed any changes in behavior, but she brings her mother for a follow-up visit with the physician to address this new symptom. The physician associates these vivid dreams with the rivastigmine therapy and calls for a therapeutic recommendation. Which of the following is most appropriate for this patient? A. Discontinue rivastigmine and initiate donepezil 5 mg/day. B. Continue rivastigmine and initiate paroxetine 20 mg/day. C. Discontinue rivastigmine and initiate memantine 5 mg/day. D. Continue rivastigmine and initiate haloperidol 0.5 mg/day.

Answer C: Discontinue rivastigmine and initiate memantine 5 mg/day. (Rivastigmine can cause vivid dreams. Because of a temporal relationship between the patient's concern and the initiation of rivastigmine, this agent should be discontinued, and memantine should be initiated for the treatment of moderate AD (Answer C is correct). All acetylcholinesterase inhibitors can cause vivid dreams; therefore, donepezil is not an appropriate option (Answer A is incorrect). Rivastigmine should be discontinued, and neither paroxetine nor haloperidol is appropriate at this time because the patient's concern is likely a drug-related adverse effect (Answer B and Answer D are incorrect).)

A 72-year-old man (height 66 inches, weight 74 kg) had an implantable cardioverter defibrillator placed 5 years ago secondary to hemodynamically unstable sustained ventricular tachycardia (VT). Now he presents to the ED stating that his ICD has been firing "a lot " during the 24 hours. Interrogation of the ICD reveals that he has been experiencing recurrent episodes of VT. He has a history of HTN and HF (Left ventricular ejection fraction [LVEF] 35%). His drugs include carvedilol 25 mg twice daily orally, lisinopril 40 mg daily orally, furosemide 40 mg daily orally, and eplerenone 25 mg daily orally. His vital signs include BP 115/75 mm Hg and HR 60 beats/minute. His physical examination reveals no evidence of jugular venous distention (JVD), pedal edema, murmurs, rubs, or gallops. Laboratory values include K 4.3 mEq/L, Mg 2.1 mEq/L, and SCr 1.2 mg/dL. Which one of the following would best reduce the frequency of ICD shocks in this patient? A. Initiate dofetilide. B. Increase carvedilol to 50 mg twice daily. C. Initiate amiodarone. D. Initiate sotalol.

Answer C: Initiate amiodarone. (When the ICD delivers shocks, it can lead to pain and anxiety for the patient, as well as draining the device's battery. Therefore, concomitant antiarrhythmic therapy can be used in these patients to suppress episodes of both ventricular and supraventricular arrhythmias and subsequently reduce the frequency of the ICD discharge. According to the results the optimal Pharmacological Therapy in Cardioverter Defibrillator Patients trial, patients in the amiodaron plus β-blocker group experienced significantly fewer ICD shocks than patients in either the β-blocker or sotalol group. This trial was conducted primarily in patients who received an ICD for secondary prevention of sudden cardiac death; therefore, the results of this trial can be extrapolated to this patient. The patient is already receiving a β-blocker, but it has been ineffective at preventing the recurrent VT. His HR is already 60 beats/minute; therefore, increasing the carvedilol dose is unwarranted (Answer B is incorrect). According to the results of this trial, the addition of oral amiodarone therapy should be more effective than β-blocker monotherapy in reducing the frequency of ICD shocks (Answer C is correct). In this trial, sotalol did not significantly reduce the risk of ICD shocks compared with the β-blocker monotherapy (Answer D is incorrect). In addition. sotalol should not be used in this patient because he has LV systolic dysfunction. Although dofetilide can be safely used in patients with an LVEF less than 40%, its use is limited to atrial arrhythmias, not ventricular arrhythmias (Answer A is incorrect).)

An 11-year-old girl admitted to the hospital for evaluation of cysts in her liver and kidney is given a diagnosis of autosomal dominant polycystic kidney disease (ADPKD). Her blood pressure readings have consistently been above the 99th percentile for age and sex. Her electrolytes are within normal limits, and she does not present with current acute renal insufficiency. Her current home drugs are acetaminophen, multivitamin, loratadine, and fluticasone inhaler. Which one of the following is best to recommend for treating this patient's blood pressure? A. Atenolol. B. Chlorothiazide. C. Enalapril. D. Verapamil.

Answer C: Enalapril. (This patient's blood pressure is consistently above the 99th percentile, suggesting hypertension; this is likely associated with her new diagnosis of ADPKD and warrants treatment to help prevent renal and cardiac complications of uncontrolled hypertension. For patients with no contraindications, an angiotensin-converting enzyme inhibitor (ACEI) such as enalapril should be the initial antihypertensive agent because inhibitors (ACEIs) prevent decreases in renal function as well as cardiac hypertrophy in pediatric patients with polycystic kidney disease (Answer C is correct). This patient has no issues with edema and is not in acute renal insufficiency; thus, a diuretic such as chlorothiazide is less likely to be effective (Answer B is incorrect). ß-Blockers and calcium channel blockers can be used to reduce blood pressure in patients with ADPKD; however, they are reserved for patients with significant SCr elevations, concomitant cardiac disease, or intolerance of both ACEIs and angiotensin receptor antagonists. This patient does not meet these criteria (Answer A and Answer D are incorrect).)

A 41-year-old Hispanic man with a history of alcoholic cirrhosis is admitted to the hospital with abdominal pain, nausea, hematemesis, variceal hemorrhage, and altered mental status. He has had previous episodes of hepatic encephalopathy (HE), ascites, and portal hypertension. The patient's oral home drugs include propranolol 40 mg twice daily, lactulose 30 mL three times/day, furosemide 40 mg once daily, spironolactone 50 mg once daily, and rifaximin 550 mg twice daily. Laboratory values include Hgb 6.4 g/dL, Hct 18.8%, platelet count 60,000/mm3, BUN 26 mg/dL, SCr 1.6 mg/dL, albumin 2.4 g/dL, total bilirubin 1.5 mg/dL, and INR 1.6. His vital signs include blood pressure (BP) 105/70 mm Hg and heart rate (HR) 103 beats/minute. Which one of the following is best to recommend for this patient? A. Endoscopic sclerotherapy and terlipressin continuous infusion. B. Combination of vasopressin and nitroglycerin continuous infusions. C. Endoscopic variceal ligation and octreotide continuous infusion. D. Endoscopic sclerotherapy and vasopressin continuous infusion.

Answer C: Endoscopic variceal ligation and octreotide continuous infusion. (The combination of pharmacologic therapy and endoscopic therapy is the most effective approach in the treatment of acute variceal hemorrhage. Use of vasopressin in the treatment of variceal hemorrhage is limited by its many adverse effects such as hypertension, cardiac ischemia, and arrhythmias (Answer B is incorrect). Although hypertension and angina can be managed by intravenous administration of nitroglycerin, drug therapy should optimally be combined with endoscopic therapy for the best clinical outcome. Endoscopic sclerotherapy can be combined with terlipressin as an option for the management of acute variceal hemorrhage; however, terlipressin is not currently available in the United States, and the initial dose of terlipressin is 2 mg intravenously every 4 hours (Answer A is incorrect). Both endoscopic sclerotherapy and ligation are acceptable options, but endoscopic variceal ligation is preferred to sclerotherapy because of the proven superiority of band ligation over sclerotherapy in several trials (Answer D is incorrect). Octreotide is preferred to vasopressin because of vasopressin's potent vasoconstrictive properties, including peripheral ischemia, arrhythmias, and hypertension (Answer C is correct).)

A 79-year-old woman who resides in a long-term care facility is treated for pneumonia with levofloxacin 750 mg daily. After 4 days of treatment, she is afebrile, and her respiratory symptoms have significantly improved. However, she is more confused, and the physical therapy staff report problems with coordination and tremor. Which one of the following is the best intervention for this patient? A. Order a repeat complete blood cell count (CBC) and chest radiography. B. Discontinue levofloxacin and initiate ceftriaxone. C. Evaluate her clearance and adjust the levofloxacin dosage accordingly. D. Order a new set of blood cultures and serum chemistries.

Answer C: Evaluate her renal function and adjust the levofloxacin dosage accordingly. (This patient appears to be experiencing resolution of her pneumonia but may be experiencing adverse effects from her antibiotic drug. Tremor and other CNS adverse effects have been reported with fluoroquinolones. She is receiving a standard adult dosage of levofloxacin. Without an assessment of her renal function before initiation of therapy, she was vulnerable to receiving an excessive dosage (Answer C is correct). She does not require a change to an alternative antibiotic drug if the symptoms resolve on dosage adjustment (Answer B is incorrect). The patient's response to the drug therapy suggests that neither repeating the CBC nor ordering chest radiography is necessary (Answer A is incorrect). She does not meet the criteria for suspected bacteremia; therefore, blood cultures and serum chemistries are not indicated (Answer D is incorrect).)

A 57-year-old woman received a heart transplant 2 years ago. Today, she is seen at follow-up after a 6-day hospitalization for a histoplasmosis infection treated with itraconazole. She has a medical history significant for hypothyroidism, gout, and chronic back pain. Her drugs include tacrolimus 3 mg twice daily, mycophenolate mofetil 1000 mg twice daily, prednisone 10 mg once daily, levothyroxine 75 mcg once daily, and oxycodone 10 mg every 8 hours as needed for pain. She has no drug allergies and denies use of over-the-counter or supplemental products. Although it is 3 days after hospital discharge, she has not yet filled her outpatient prescription for itraconazole 200 mg two times/day for the histoplasmosis infection. All of her other drugs remain the same, except for mycophenolate mofeti), which was reduced to 500 mg twice daily. Her daily tacrolimus trough levels (nanograms per milliliter) while in the hospital were as follows: 5.2 on day I; 6.3 on day 2; 9.1 on day 3; 22.6 on day 4; 20.6 on day 5; and 25.2 on day 6. Her tacrolimus level today is 13.8 ng/mL. Which one of the following is best to recommend regarding this patient's outpatient itraconazole therapy? A. Do not fill itraconazole; initiate nystatin swish and swallow three times daily. B. Fill itraconazole immediately; no other changes are necessary. C. Fill itraconazole immediately, increase tacrolimus monitoring frequency; reduce tacrolimus to 1.5 mg twice daily. D. Fill itraconazole immediately; reduce tacrolimus to 2 mg twice daily.

Answer C: Fill itraconazole immediately, increase tacrolimus monitoring frequency; reduce tacrolimus to 1.5 mg twice daily. (Tacrolimus is a major substrate, and itraconazole is an inhibitor of the CYP3A4 isoenzyme. Itraconazole inhibition results in decreased metabolism of tacrolimus and ultimately an increase in tacrolimus exposure, resulting in increased tacrolimus levels, Complicating this situation is that this patient was initiated on itraconazole while in the hospital; then treatment was interrupted and not administered for 2 days as an outpatient because she did not fill her prescription immediately on discharge. The tacrolimus levels increased after itraconazole initiation and then started to decline after discharge, as evidenced by the daily trough levels. In this case, inpatient records allow observation of the impact of the interaction between tacrolimus and itraconazole. From this information, proactively reducing her tacrolimus dose as she resumes itraconazole therapy can prevent additional supratherapeutic tacrolimus levels. Tacrolimus dose initiation and dose adjustments require frequent laboratory monitoring to ensure target trough levels are achieved. Itraconazole is recommended therapy for histoplasmosis, and the prescription must be filled; both increasing the tacrolimus monitoring frequency and decreasing the tacrolimus dose are also correct (Answer C is correct). Treatment with nystatin swish and swallow is not recommended (Answer A is incorrect). Decreasing the tacrolimus dose proactively is recommended (Answer B is incorrect) The monitoring frequency should also be increased (Answer D is incorrect).)

A 38-year-old woman with gout and hypertension is being discharged from the emergency department after an asthma attack. Which one of the following drugs in her discharge orders would be most relevant to improving the HEDIS measure "Use of Appropriate Medications for People with Asthma" (ASM)? A. Albuterol. B. Prednisone. C. Fluticasone. D. Salmeterol.

Answer C: Fluticasone. (The HEDIS measure ASM would be affected by the discharge order of fluticasone (Answer C is correct). This measure evaluates the percentage of members with persistent asthma who were dispensed at least one prescription for a preferred therapy such as inhaled corticosteroids (ICSs), ICS combinations, or leukotriene modifiers during the measurement year. Prednisone is not included in the ASM HEDIS measure (Answer B is incorrect). However, albuterol and salmeterol are part of the ASM measure to identify patients with persistent asthma (Answer A and Answer D are incorrect). Albuterol is not listed as a preferred asthma medication, and salmeterol is listed only when it is used in combination with fluticasone.)

A 36-year-old man (71 inches, weight 77 kg [170 lb]) with a history of intravenous drug use and no known drug allergies is admitted to the hospital with a I-week history of fever and weakness and a physical examination positive for petechiae on his distal extremities. A transesophageal echocardiogram is positive for a 0.8-cm vegetation on his tricuspid valve, and blood cultures are positive for S. aureus. Susceptibilities for the organism are as follows: ----------------------------------------------------------------------- Oxacillin 1 S, Clindamycin < 0.5 S, Vancomycin 1 S, Penicillin > 8 R, Minocycline < 4 S, Mupirocin < 256 S, Levofloxacin < 1 S, TMP/SMX < 0.5/9.5 S, Rifampin < 0.5 S R = resistant; S = susceptible; TMP/SMX = trimethoprim/sulfamethoxamle. ---------------------------------------------------------------------- The patient's SCr is 0.8 mg/dL. He receives a diagnosis of uncomplicated endocarditis. Which one of the following is the best antimicrobial regimen for this patient? A. Vancomycin 1200 mg intravenously every 12 hours for 6 weeks. B. Oxacillin 2 g intravenously every 4 hours and gentamicin 75 mg intravenously every 8 hours for 6 weeks. C. Gentamicin 75 mg intravenously every 8 hours for 5 days and nafcillin 2 g every 4 hours for 2 weeks. D. Gentamicin 75 mg intravenously every 8 hours for 2 weeks, nafcillin 2 g intravenously every 4 hours for 6 weeks, and rifampin 300 mg orally every 8 hours for 6 weeks.

Answer C: Gentamicin 75 mg intravenously every 8 hours for 5 days and nafcillin 2 g every 4 hours for 2 weeks. (Because the patient has no allergies and the organism is oxacillin-susceptible, vancomycin is not considered optimal therapy (Answer A is incorrect). Studies have shown that short-duration therapy (2 weeks) with or without synergistic aminoglycoside therapy is effective for uncomplicated right-sided staphylococcal endocarditis. This case is considered uncomplicated because the patient does not have accompanying kidney failure, signs of extrapulmonary infection, emboli aortic or mitral valve involvement, or signs of meningitis; therefore, 6 weeks of therapy is unnecessary (Answer B is incorrect). The regimen in Answer D would be the correct regimen for prosthetic valve staphylococcal endocarditis (Answer D is incorrect). Because this patient has a native valve, gentamicin for 5 days and nafcillin therapy for 2 weeks is the appropriate regimen (Answer C is correct).)

A man (weight 60 kg) presents to the emergency department (ED) with a 6-hour history of chest pressure (10/10 on a pain scale). Echocardiography shows ST depression in inferior leads. His medical history is significant for hypertension (HTN), diabetes, and chronic kidney disease. Pertinent laboratory results are troponin I 5.8 ng/L, SCr 4.5 mg/dL, and estimated CrCl 15 mL/minute/1.73m^2. The patient has been given aspirin 325 mg x I; a nitroglycerin drip is initiated at 5 mcg/minute and will be titrated to chest pain relief and blood pressure (BP). The cardiology team plans to take this patient to the catheterization laboratory in the morning. Which one of the following anticoagulation strategies is most appropriate to initiate in this patient? A. Enoxaparin 60 mg subcutaneously every 12 hours. B. Fondaparinux 2.5 mg subcutaneously daily. C. Heparin 3600 units intravenous bolus, followed by 720 units/hour continuous infusion. D. Bivalirudin 45 mg bolus, followed by 105 mg/hour infusion.

Answer C: Heparin 3600 units intravenous bolus, followed by 720 units/hour continuous infusion. (Guidelines for unstable angina/non-ST-segment elevation myocardial infarction (UA/NSTEMI) recommend the use of one anticoagulant during an acute event. Enoxaparin, fondaparinux, unfractionated heparin (UFH), and bivalirudin are all recommended as class I agents for the medical management of UA/NSTEMI. However, fondaparinux falls out of favor when a patient is to be managed invasively (Answer B is incorrect). Increased risk of catheter-related thrombosis has been associated with fondaparinux use in the catherization laboratory, and additional heparin would have to be given to this patient to ensure adequate anticoagulation. Of the remaining options, UFH is preferred because of its rapid clearance (Answer C is correct). Both enoxaparin and bivalirudin are appropriate options but would need to be dose-adjusted given this patient's estimated CrCl of 15 mL/minute (Answer A and Answer D are incorrect).)

An infant is brought to the clinic for his I-year checkup. His older brother is undergoing chemotherapy for leukemia. Which one of the following vaccine combinations should be recommended today for this infant? A. Live, attenuated influenza vaccine (LAIV4) and meningococcal conjugate vaccine (MCV4). B. Measles, mumps, and rubella (MMR) and LAIV4. C. Haemophilus influenzae type B (Hib) and MMR. D. MCV4 and Hib.

Answer C: Hib and MMR. (Living in a household with an immunocompromised person is not a valid contraindication for immunization, regardless of whether the vaccine is live, attenuated (MMR) or inactivated (Hib). This infant should receive all vaccines when he is scheduled for them. At 12 months, he should receive his first dose of MMR as well as his booster dose of Hib, which can be administered any time from 12 to 15 months of age (Answer C is correct). The patient would not receive the MCV4 vaccine now because he is not considered at high risk; instead, he would receive the MCV4 at 11—13 years of age (Answer A and Answer D are incorrect). In addition, the inactivated influenza vaccine (II V 3) is the appropriate choice for this patient because the live, attenuated version (LAIV4) is not approved for use in patients younger than 2 years (Answer A and Answer B are incorrect).)

A 12-month-old infant is evaluated in your clinic today. His parents want him to receive his 12-month vaccines. You note in the chart that 1 week ago, he was in the clinic to receive the varicella vaccine because of a chickenpox outbreak at his day care facility. The patient is up to date with all previously scheduled immunizations. Which one of the following vaccine combinations is best for this patient to receive today? A. Inactivated polio vaccine (IPV) and MMR. B. MMR and Hib. C. Hib and PCV13. D. PCV13 and DTaP.

Answer C: Hib and PCV13. (At 12 months of age, this infant is scheduled for the following vaccines: Hib, PCV13, IPV, II V 3, MMR, and hepatitis A. However, because he received the varicella vaccine last week, he cannot receive the MMR vaccine for at least 3 more weeks because 28 days' separation between live vaccines is necessary to form an immune response to the second vaccine (Answer A and Answer B are incorrect). Inactivated vaccines may be given without regard to spacing with live vaccines. The DTaP vaccine is not indicated at the 12-month visit. He is due for a booster dose at 15 months of age (Answer D is incorrect). Therefore, the most appropriate vaccines for this patient to receive today are Hib and PCV13 (Answer C is correct).)

A 32-year-old man with a history of schizophrenia presents to your clinic. He has been stable on clozapine for 2 years and has his laboratory values monitored every 4 weeks. He recently moved and now needs a new practitioner to assist him with clozapine monitoring. The patient's WBC is 3.6 x 10^3 cells/mm^3 and absolute neutrophil count (ANC) is 1200 cells/mm^3. Which one of the following would be the best next step for this patient? A. Continue clozapine and increase laboratory monitoring to every day. B. Monitor every 4 weeks and ask the patient to call the office if he begins to feel weak. C. Hold clozapine and rechallenge with clozapine when the ANC is greater than 2000 cells/mm D. Discontinue clozapine and do not rechallenge.

Answer C: Hold clozapine and rechallenge with clozapine when the ANC is greater than 2000 cells/mm^3. (Clozapine has strict monitoring guidelines. The ANC of 1200 cells/mm^3 meets the criteria for moderate granulocytopenia. Therefore, this patient must have an interruption in clozapine therapy with close monitoring daily until the ANC is greater than 1500 cells/mm^3 and then twice weekly until the ANC is greater than 2000 cells/mm^3 (Answer C is correct). Clozapine may be reintroduced only when the total WBC is greater than 3.5 x 10^3 cells/mm3 and the ANC is greater than 2000 cells/mm3 (Answer A, Answer B, and Answer D are incorrect).)

A 54-year-old man is referred to a CV clinic for management of his BP. He reports no recent angina or breathing exacerbations. His medical history includes HTN, hyperlipidemia, stable angina, and chronic obstructive pulmonary disease. His drug regimen includes aspirin 81 mg daily, nitroglycerin 0.4 mg sublingually as needed for chest pain, diltiazem extended release 360 mg daily, isosorbide mononitrate 30 mg daily, ranolazine 500 mg twice daily, lisinopril 20 mg daily, simvastatin 20 mg daily, albuterol/ipratropium two inhalations four times daily, and salmeterol/fluticasone 50/250 one inhalation twice daily. Vital signs include BP 152/96 mm Hg and HR 58 beats/minute. Laboratory results include SCr 1.1 mg/dL (baseline), serum potassium 4.0 mEq/L, TC 180 mg/dL, TG 140 mg/dL, HDL-C 55 mg/dL, and AIC 7.1%. An ECG reveals a normal sinus rhythm. Which one of the following drug changes would best improve this patient's BP control? A. Initiate carvedilol 12.5 mg twice daily. B. Initiate telmisartan 40 mg daily. C. Increase lisinopril to 40 mg daily. D. Increase isosorbide mononitrate to 60 mg daily.

Answer C: Increase lisinopril to 40 mg daily. (β-Blockers are indicated in patients with stable angina; however, the patient's HR is at goal of less than 60 beats/minute, and the addition of carvedilol at 12.5 mg twice daily would be too high an initial starting dose and would risk excessively lowering the patient's HR (Answer A is incorrect). The combination of an angiotensin receptor blocker (ARB), such as telmisartan, and an angiotensin-converting enzyme (ACE) inhibitor has not shown a reduction in CV outcomes (Answer B is incorrect). The ON TARGET trial showed no difference in CV death, MI, or hospitalization for HF between monotherapy and combination therapy, yet an increase in adverse effects is significant when an ACE inhibitor and an ARB are used concomitantly. An increase in lisinopril from 20 mg to 40 mg could safely provide additional BP reduction (Answer C is correct). The initiation and titration of nitrate therapy is reserved for patients with angina. An increase in isosorbide mononitrate might provide a very modest reduction in BP; however, the patient reports no symptoms of angina (Answer D is incorrect).)

A 32-year-old man receives a diagnosis of immunoglobulin A (IgA) nephropathy and has an initial trial of losartan 50 mg daily; he is initiated on prednisone 70 mg orally once daily. His laboratory values include SCr 1.1 mg/dL, BUN 19 mg/dL, and proteinuria 1.5 g/day. Two months later, his SCr is 1.0 mg/dL, and his proteinuria has decreased to 1.1 g/day. His blood pressure (BP) is 148/86 mm Hg, and his fasting blood glucose is 138 mg/dL from capillary blood, both of which are elevated from baseline. His LDL-C and HDL-C are 92 mg/dL and 58 mg/dL, respectively. Which one of the following is best to recommend for this patient? A. Continue losartan; taper oral prednisone over 6 weeks; then continue at 10 mg/day. B. Continue losartan, discontinue prednisone, and initiate oral omega-3 fatty acids 12 g/day. C. Increase the oral losartan dose to 100 mg/day and continue prednisone. D. Discontinue losartan and initiate oral enalapril 10 mg/day.

Answer C: Increase oral losartan to 100 mg/day and continue prednisone. (Corticosteroids can be considered for patients with persistent proteinuria greater than I g/day, despite optimal ACE inhibitor or angiotensin receptor blocker (ARB) therapy. Patients who receive high doses of corticosteroids for a shorter time (less than 6—12 months) have better outcomes than those who receive lower doses for longer periods. Adverse events can be managed until the induction phase is completed. This patient is responding with a slight decrease in proteinuria, and prednisone should be continued until a complete remission is attained. He would likely benefit from an increased dosage of losartan for both antiproteinuric and BP-lowering effects. It is unclear whether greater BP reduction should be done for patients with more severe proteinuria, but the recommended BP goal is less than 125/75 mm Hg for patients with proteinuria greater than 1 g/day (Answer C is correct; Answer A and Answer B are incorrect). No evidence supports that ACE inhibitors are superior to ARBs in treating proteinuria (Answer D is incorrect).)

A 75-year-old man with a 55-year history of type 1 diabetes mellitus (TIDM) is currently taking regular insulin and neutral protamine Hagedorn (NPH) insulin twice daily (before breakfast and supper) for a TDI dose of 30 units. He tests his blood glucose just before meals and at bedtime. The patient has had no substantial changes in diet, exercise, stress, or illness during this time. His daily blood glucose values were recorded as follows: Day || 7:00 a.m (mg/dL) || Noon (ng/dL) || 5:00 p.m. (mg/dL) || 10:00 p.m. (mg/dL) --------------------------------------------------- Wednesday || 97 || 115 || 157 || 116 --------------------------------------------------- Thursday || 100 || 109 || 149 || 118 --------------------------------------------------- Friday || 95 || 110 || 153 || 110 --------------------------------------------------- Saturday || 105 || 111 || 160 || 115 --------------------------------------------------- Sunday || 107 || 106 || 154 || 111 Which one of the following is best to recommend for this patient? A. Add a noontime regular insulin dose. B. Increase the morning regular insulin. C. Increase the morning NPH insulin. D. Increase the evening regular insulin.

Answer C: Increase the morning NPH insulin. (The patient's presupper blood glucose values are too high, which represents poor postlunch glucose control. These blood glucose values are most affected by the morning NPH dose, which should be increased (Answer C is correct). Increasing the evening regular insulin would help cover these high readings but would not prevent future high readings (Answer D is incorrect). Adding a prelunch regular insulin dose would decrease the presupper blood glucose but at the cost of an additional injection (Answer A is incorrect). Increasing the dose of the morning regular insulin would not address the 5:00 p.m. hyperglycemia because the duration of effect of regular insulin is only 4 hours. In addition, an increase in dose could precipitate midmorning or noontime hypoglycemia (Answer B is incorrect).)

A 62-year-old man with a history of depression presents to the clinic for follow-up. Vital signs today include blood pressure 130/80 mm Hg and heart rate 72 beats/minute. He is taking venlafaxine extended release (XR). The dosage was titrated to 150 mg/day for treatment of a major depressive episode 10 months ago, and he has continued on this regimen. The patient believes his depressive symptoms resolved during the first 4 months of treatment, but since then, he has noted a gradual decline in his mood and energy level. He has become more vocal about other symptoms and expresses that he has been feeling constant, uncontrollable worry for the past 6 months since he and his wife separated. He reports feelings of restlessness, muscle tension, difficulty concentrating at work, and increased irritability. Which one of the following is best to recommend for this patient? A. Add buspirone 7.5 mg two times/day. B. Discontinue venlafaxine XR and initiate tiagabine 15 mg/day. C. Increase venlafaxine XR to 225 mg/day. D. Increase venlafaxine XR to 225 mg/day and add tiagabine 15 mg/day.

Answer C: Increase venlafaxine XR to 225 mg/day. (This patient has symptoms of depression and generalized anxiety disorder (GAD). He has a history of partial response to venlafaxine; hence, he might respond to an increase in the venlafaxine dose. Venlafaxine can decrease symptoms of anxiety; therefore, increasing venlafaxine XR to 225 mg/day is the best response (Answer C is correct). Although buspirone has efficacy in the treatment of GAD, combining buspirone with his current dose of venlafaxine is an inappropriate use of pharmacotherapy because it would unlikely affect his depressive symptoms (Answer A is incorrect). If a decision were made to change treatment, other options with better evidence of efficacy should be tried before treatments with little controlled evidence. Therefore, initiating tiagabine, which is not an approved therapy for GAD, is not the best choice (Answer B is incorrect). One change in therapy should be initiated at a time to determine which change has a benefit for treatment outcome (Answer D is incorrect).)

A 66-year-old woman presents to her primary care provider with fever, cough, nausea, weight loss, and fatigue for the past 6 weeks; these symptoms have not responded to several courses of antibiotics. Chest radiography reveals pneumonia with bilateral hilar lymphadenopathy. Her medical history is significant for rheumatoid arthritis, gastroesophageal reflux disease, and osteoporosis. Her home drugs include methotrexate, folic acid, infliximab, omeprazole, alendronate, and vitamin D. Her social history is significant for a recent move from Michigan to Missouri to be closer to her grandchildren. She is working to renovate an old farmhouse. Her provider orders a urinary histoplasmosis antigen, which returns positive. Which one of the following is best to initiate in this patient? A. Voriconazole 400 mg orally twice daily for 3 days and then 200 mg orally twice daily. B. Itraconazole capsule 300 mg orally three times daily for 3 days and then 200 mg orally twice daily. C. Itraconazole solution 300 mg orally three times daily for 3 days and then 200 mg orally twice daily. D. It is not necessary to initiate treatment for this mild, self-limiting disease.

Answer C: Itraconazole solution 300 mg orally three times daily for 3 days and then 200 mg orally twice daily. (Because the patient does not require hospitalization and her histoplasmosis appears not to be extrapulmonary, she is classified as having mild to moderate pulmonary histoplasmosis. Typically, mild to moderate histoplasmosis requires no treatment, and it is generally considered self-limiting. However, if symptoms continue for more than 4 weeks or if the patient is immunocompromised, treatment should be considered. This patient is considered immunocompromised, given her treatment with infliximab for rheumatoid arthritis, and has had symptoms for 6 weeks; therefore, she should be treated (Answer D is incorrect). On an outpatient basis, the treatment of choice for mild to moderate pulmonary histoplasmosis is itraconazole. Because she is taking omeprazole, she should be placed on itraconazole solution rather than capsules because capsules require an acidic environment for optimal absorption (Answer C is correct; Answer B is incorrect). Itraconazole is preferred therapy to voriconazole (Answer A is incorrect).)

A study was conducted among patients who received hospice care at the end of life. Researchers examined the relationship between a set of clinical and demographic characteristics and pain. Pain was assessed when the patient began receiving hospice care and then every several days until the individual died or was discharged. The outcome of interest was whether the person had severe pain at the last observation. Which one of the following types of analysis would be best to use in this situation? A. Linear regression. B. Survival analysis. C. Logistic regression. D. Ordered logistic regression.

Answer C: Logistic regression. (The outcome of interest, regardless of whether the patient had severe pain at the last observation, is a categorical dependent variable. Therefore, logistic regression is the best choice (Answer C is correct). Linear regression is used when the outcome is measured on a continuous scale (Answer A is incorrect). Survival analysis is used when the outcome of interest is time to an event and when the analyst wants to account for censoring (Answer B is incorrect). Ordered logistic regression is a generalization of logistic regression that allows the outcome to have more than two ordered categories (Answer D is incorrect).)

Ten months ago, a 47-year-old woman underwent a kidney-pancreas transplant. Today, she returns to the posttransplant clinic with severe, persistent diarrhea (more than six stools a day). Her previous diarrhea workups have been negative, and a second stool sample obtained last week revealed no infectious source. Her current laboratory results include SCr 1.2 mg/mL (baseline 1.0 ng/mL). All other laboratory values are within normal limits. Her current drugs include cyclosporine 100 mg twice daily, mycophenolate mofetil 1000 mg twice daily, prednisone 7.5 mg once daily, citalopram 10 mg once daily, loratadine 10 mg once daily, and zolpidem 10 mg once daily as needed for insomnia. The medical team is convinced the patient's diarrhea is caused by mycophenolate mofetil. Which of the following recommendations for mycophenolate mofetil would best address this patient's diarrhea? A. Discontinue the drug and initiate mycophenolate sodium. B. Discontinue the drug and increase the cyclosporine dose. C. Maintain the total daily dose, but increase the frequency of administration to four times daily. D. Maintain the total daily dose, but decrease the frequency of administration to once daily.

Answer C: Maintain the total daily dose, but increase the frequency of administration to four times daily. (Mycophenolate mofetil is a prodrug for mycophenolic acid (MPA); MPA inhibits inosine monophosphate dehydrogenase and subsequent de novo purine synthesis, ultimately inhibiting T lymphocytes. Gastrointestinal (GI) adverse events are reported to occur in greater than 20% of patients taking this drug and include abdominal pain, nausea, diarrhea, constipation, vomiting, anorexia, and dyspepsia. Initial treatment of Gl-related adverse events should not include discontinuing the drug; instead, the aim is to adjust the regimen by lowering the dose or increasing the frequency of administration. The first step is usually to increase the dosing frequency while maintaining the total daily dose. If neither modification is successful, some may consider switching to mycophenolate sodium, which is the enteric-coated formulation; however, no differences in GI adverse events were observed in early clinical trials. Switching to mycophenolate sodium is typically not the first option because clinical trials did not report improvement in GI adverse effects (Answer A is incorrect). Ultimately, if symptoms persist and are intolerable to the patient, therapy should be discontinued. Discontinuing mycophenolate mofuil is not the first-line treatment for Gl-related adverse effects, and increasing the cyclosporine dose would likely not help because GI adverse effects are not commonly associated with cyclosporine therapy (Answer B is incorrect). Decreasing the frequency of administration would likely increase the symptoms because more drug would be present in the GI tract at one time (Answer D is incorrect). Increasing the frequency of administration from two times daily to four times daily is the recommended first step in managing Gl-related adverse effects (Answer C is correct).)

A 72-year-old man is admitted to a comprehensive stroke center with acute ischemic stroke; he arrived in the emergency department 2 hours from his symptom onset. The patient's initial assessment reveals a National Institutes of Health Stroke Scale (NIHSS) score of 27 and a modified Rankin scale score of 5. Neuroimaging reveals a large malignant occlusion of his middle cerebral artery (MCA) with cerebral edema. His blood pressure (BP) is 150/82 mm Hg, heart rate (HR) 100 beats/minute in sinus rhythm, and respiratory rate (RR) 16 breaths/minute. Other laboratory results include serum glucose 148 mg/dL, INR 1.2, platelet count 202,000/mm3, prothrombin time 14 seconds, and PTT 32 seconds. The patient's medical and surgical history is noncontributory. The stroke neurologist completes the stroke workup within 45 minutes. Which one of the following is this patient's most important risk factor for symptomatic hemorrhagic transformation with intravenous alteplase? A. BP of 150/82 mm Hg. B. INR of 1.2. C. Malignant MCA occlusion. D. Glucose of 148 mg/dL.

Answer C: Malignant MCA occlusion. (The location and size of stroke, uncontrolled BP, elevated glucose, and high NIHSS score are associated with increased risk of hemorrhagic transformation after intravenous alteplase. The patient's current BP is within the acceptable range for receiving thrombolytic therapy, which is less than 180/105 mm Hg (Answer A is incorrect). His INR of 1.2 is not considered too elevated for alteplase therapy if given within 3 hours of symptom onset (Answer B is incorrect). Malignant MCA infarction is an important risk factor because a large MCA infarct with cerebral edema is an independent risk factor for hemorrhagic transformation (Answer C is correct). His glucose concentration of 148 mg/dL is less of a concern because a glucose reading on admission of greater than 200 mg/dL is associated with an increased risk of hemorrhagic transformation (Answer D is incorrect).)

Participants in a study are asked to rate their satisfaction with their medical care using a 0—5 Likert-type scale (0 not satisfied at all, 5 = completely satisfied). Which one of the following measures is best to evaluate the results of this patient satisfaction survey? A. Mean. B. Interquartile range. C. Median. D. Mode.

Answer C: Median. (The median is an estimate of the center of the distribution of responses and is not sensitive to extremes (Answer C is correct). The mean is incorrect for this question because, although the responses are ordered, it cannot be assumed that the distance between values is uniform. In addition, the mean is subject to distortion by extreme values, and satisfaction is a crude indicator of the quality of health care (Answer A is incorrect). Interquartile range reflects the distribution of the central 50% of the data (Answer B is incorrect). The mode is the most common value (Answer D is incorrect).)

During an MTM encounter, a pharmacist finds that the patient has discontinued one of his prescribed maintenance drugs. The pharmacist also discovers that the patient is illiterate and that he understands neither the indication for nor the importance of his drug regimen. Which one of the following parts of the MTM framework most likely allowed the pharmacist to discover this information? A. Medication-related action plan. B. Referral documentation. C. Medication therapy review. D. Progress note written in the subjective, objective, assessment, and plan (SOAP) format.

Answer C: Medication therapy review. (The medication-related action plan is one element of MTM and is a critical piece of documentation regarding interventions waiting for physician approval or other related issues outside the pharmacist's scope of practice (Answer A is incorrect).The medication therapy review provides an opportunity for the pharmacist to discuss problems, adherence, and other patient-specific issues such as literacy level (Answer C is correct). Referral documentation and a progress note written in SOAP format are documents used by the pharmacist; they are not part of the MTM framework in which this information is gathered from the patient (Answer B and Answer D are incorrect).)

A 32-year-old woman is admitted to the hospital for surgical repair of a broken femur from a motor vehicle crash. In the operating room, she received 2 mg of morphine intravenously. Now, 6 hours later, she rates her pain as 9/10 and complains of nausea and an inability to take drugs orally. Her medical history is significant for heroin use for 10 years before attaining remission from substance dependence. Her only home medication is methadone 100 mg by mouth daily as part of a methadone maintenance program. Which one of the following is the most appropriate choice to treat this patient's pain? A. Methadone 90 mg subcutaneously daily plus ketorolac 30 mg intramuscularly now. B. Methadone 120 mg subcutaneously daily plus morphine 5 mg intravenously every 4 hours. C. Methadone 50 mg intravenously daily plus morphine patient-controlled analgesia. D. Methadone 90 mg intravenously daily plus morphine 2.5 mg intravenously every 4 hours.

Answer C: Methadone 50 mg intravenously daily plus morphine patient-controlled analgesia. (For patients on methadone maintenance therapy who are experiencing acute pain, the baseline daily dose of methadone does not address pain. The addition of opioid agents to the baseline dose of methadone is necessary to both avoid opiate withdrawal and adequately relieve pain. Intravenous doses of methadone should be about one-half to two-thirds the home oral methadone dose (Answer C is correct). Subcutaneous doses of methadone should be one-half to two-thirds the daily oral dose of methadone. Use of opioids is recommended for severe acute pain in patients on methadone maintenance therapy. Nonsteroidal anti-inflammatory drugs can be adjuncts but not mainstays of pain management for patients with severe pain (Answer A is incorrect). It is best to use the patient's home maintenance dose of methadone to prevent withdrawal, together with another agent to manage pain. The subcutaneous dose here is higher than the patient's home dose; however, the baseline methadone dose should not be increased to control short-term pain (Answer B is incorrect). Intravenous doses of methadone should be one-half to two-thirds the daily oral dose for a patient on methadone maintenance therapy. Short-acting opioids initiated at a dose higher than appropriate for opioid-naive patients must be used to manage pain effectively (Answer D is incorrect).)

A 14-year-old adolescent (weight 80 kg [176 lb]) is brought to the pediatrician's office by his mother. His teacher has noted that the boy is unable to sit still in his seat and that he constantly fidgets, blurts out answers, and does not follow through on homework. In class, he is not easily redirected, and often, he does not turn in his homework. He received a diagnosis of major depressive disorder and currently takes fluoxetine 20 mg orally daily. Which of the following would be the best initial treatment of this patient's new condition? A. Atomoxetine 40 mg orally twice a day. B. Clonidine ER 0.2 mg orally twice a day. C. Methylphenidate SR 20 mg orally in the morning. D. Mixed amphetamine salts XR 40 mg orally in the morning.

Answer C: Methylphenidate SR 20 mg orally in the morning. (An initial dose of methylphenidate SR 20 mg (Ritalin SR) in the morning is a reasonable first choice for this patient. Using the SR dosage form avoids the need for a "school bottle" and lunchtime dosing. The SR dosage form may not last throughout the day into the late afternoon; however, this effect can be addressed with subsequent dose adjustments (Answer C is correct). Atomoxetine can be used initially for ADHD; however, because the patient is also taking fluoxetine (a strong CYP2D6 inhibitor), the initial starting dose should be lower than 40 mg/day (Answer A is incorrect). Clonidine ER 0.2 mg twice a day is a high initial dose; this agent is usually initiated at 0.1 mg/day and titrated for effect (Answer B is incorrect). Mixed amphetamine salts XR 40 mg is a high initial dose (Answer D is incorrect).)

Two weeks ago, a 45-year-old woman was treated for a Clostridium difficile infection with metronidazole 500 mg orally three times daily for 10 days. She has not taken any antibiotics or any other medications. Today, she comes to the clinic with severe abdominal pain and frequent loose stools for the past few days. The C. difficile glutamate dehydrogenase (GDH) enzyme immunoassay comes back positive for GDH and toxin, and she receives a diagnosis of recurrent C. difficile infection. Which one of the following is best to recommend for this patient? A. Metronidazole 500 mg orally three times daily for 4 weeks. B. Fidaxomicin 200 mg daily for 10 days. C. Metronidazole 500 mg orally three times daily for 10 days. D. Vancomycin 125 mg intravenously twice daily for 10 days.

Answer C: Metronidazole 500 mg orally three times daily for 10 days. (For the first recurrence of C. difficile infection, the recommendation is to repeat the previous therapy for 10 more days; therefore, metronidazole 500 mg orally three times daily for 10 days is the best option (Answer C is correct; Answer A is incorrect). Although fidaxomicin would be a good option and has been shown to decrease recurrent infections compared with vancomycin, the dose should be 200 mg twice daily (Answer B is incorrect). The vancomycin dose and route of administration are not appropriate; if used, vancomycin would be given orally 125 mg four times daily (Answer D is incorrect).)

A 72-year-old woman (weight 60 kg [132 lbs]) presents with hypovolemia and a serum sodium of 160 mEq/L. After hemodynamic stabilization, which of the following intravenous fluid regimens will most safely and effectively correct the hypernatremia? A. 0.9% sodium chloride at 100 mL/hour. B. 0.9% sodium chloride at 200 mL/hour. C. 0.45% sodium chloride at 100 mL/hour. D. 0.45% sodium chloride at 200 mL/hour.

Answer D: 0.45% sodium chloride at 200 mL/hour. (Normal saline is isotonic and will not provide free water, so it will not effectively correct hypernatremia (Answer A and Answer B are incorrect). Although 0.45% sodium chloride contains free water, a rate of 100 mL/hour is inadequate to correct the fluid deficit and the hypernatremia (Answer C is incorrect). Administering 0.45% at 200 mL/hour will deliver the necessary volume of free water over about 40 hours, which should safely correct the sodium at a rate of about 0.5 mEq/L per hour. Chronic hypernatremia with no or mild symptoms should be corrected at a rate not to exceed 0.5 mEq/L/h and a total of 8—10 mEq/d (e.g., 160—152 mEq/L in 24 hours) (Answer D is correct).)

The ICU team calls you regarding a patient who has had mitral valve repair surgery. The patient's vital signs are blood pressure 118/63 mm Hg, pulse rate 115 beats/minute, respiratory rate 16 breaths/minute, and temperature 990 F (37.20C). A pulmonary artery catheter is in place. Pertinent hemodynamic values are CVP 18 mm Hg, mean pulmonary arterial pressure (MPAP) 30 mm Hg, systemic vascular resistance (SVR) 1362 dynes x second/cm^5, pulmonary capillary wedge pressure (PCWP) 12 mm Hg, and cardiac output 3.7 L/minute. Which of the following would best treat this patient's heart dysfunction? A. Dobutamine. B. Vasopressin. C. Milrinone. D. Dopamine.

Answer C: Milrinone. (This patient's calculated pulmonary vascular resistance (PVR) is 389 dynes x second/cm5. (PVR(dyne x sec)/cm^5) = (80 x (PAP - PCWP)/CO). The CVP is elevated with a normal PCWP; the P VR is elevated, and the mean arterial pressure (MAP) is 81 mm Hg. Milrinone is the best option because the MPAP and P VR are elevated, and this patient has a reasonable blood pressure (Answer C is correct). The patient's MPAP and P VR are high, and dobutamine may help increase cardiac contractility and increase cardiac output, but compared with milrinone, dobutamine has less of an effect on pulmonary pressures (Answer A is incorrect). Vasopressin is a pure vasopressor and will increase the patient's SVR and MAP, leading to a decrease in CO (Answer B is incorrect). Dopamine is not the best option to increase the contractility of the heart compared with milrinone; it could potentially increase afterload secondary to its effects on the (I-receptor. In addition, dopamine will not decrease the PVR (Answer D is incorrect).)

You are evaluating a clinical trial with the primary combined outcome of cardiovascular death, myocardial infarction, or stroke. This outcome occurred in 9.9% of the treatment A group and in 12.1% of the treatment B group and results in a hazard ratio (A vs. B) of 0.81 and a 95% confidence interval (CI) of 0.73—0.90. Which one of the following best describes the number needed to treat (NNT) to prevent one outcome? A. 22 B. 9.9 C. 46 D. 81

Answer C: NNT = 46 (The NNT is 1 divided by the ARR. In this case, the ARR is 12.1% - 9.9% 2.2% (1/0.022 45.4). Because the NNT should be rounded up, the correct answer is 46 (Answer C is correct). The other answers provided do not represent the NNT. The calculated ARR is 2.2% (Answer A is incorrect). The EER is 9.9 (Answer B is incorrect). The hazard ratio is 0.81, but 81 is not the NNT (Answer D is incorrect).)

The nursing director of a home health and hospice care program has contacted you for a recommendation founded on evidence-based medicine (EBM) for the use of aromatherapy and massage therapy in the hospice program. Which one of the following Internet sources is most likely to be of value? A. WebMD. B. Mayo Clinic. C. National Center for Complementary and Alternative Medicine (NCCAM). D. Net Wellness.

Answer C: National Center for Complementary and Alternative Medicine (NCCAM). (The NCCAM Web site provides EBM guidance when evaluating different complementary and alternative medicine therapies (Answer C is correct). The other Web sites listed do not provide EBM recommendations that the nursing director could use in the program (Answer A, Answer B, and Answer D are incorrect). These sites may provide information on these therapies, but they require the nursing director to assimilate the data and make EBM decisions from the acquired information, which assumes that the nursing director knows proper searching strategies to find the necessary information.)

A 72-year-old woman has experienced new-onset vaginal bleeding during the past months, She is nulliparous and had her first menstrual period at age 10. She experienced menopause about 15 years ago; since then, she has been taking conjugated estrogen and progesterone once daily continuously. She admits often missing her progesterone doses, Which one of the following factors places this patient at greatest risk of endometrial cancer? A. Lack of a pregnancy, B. Use of hormone therapy, C. Nonadherence to progesterone. D. Early menarche/late menopause.

Answer C: Nonadherence to progesterone. (Nonadherence to progesterone is the risk factor most likely to contribute to the development of endometrial cancer in this patient (Answer C is correct). Unopposed estrogen therapy is associated with an increased risk of endometrial hyperplasia, which increases the risk of mutation and, ultimately, endometrial cancer. The purpose of progesterone in hormone regimens is to prevent endometrial hyperplasia. Lack of a pregnancy is a risk factor for ovarian cancer but is not associated with endometrial cancer (Answer A is incorrect). Use of hormone therapy decreased the risk of endometrial cancer in the Women's Health Study; however, long-term hormone therapy is associated with increasing the risk of both breast and ovarian cancer (Answer B is incorrect). Early menarche/late menopause is associated with increased lifetime exposure to hormones. However, for this woman, nonadherence to progesterone likely contributed more than either early menarche or late menopause (Answer D is incorrect).)

One of the objectives of the ONTARGET trial was to evaluate whether the outcomes in patients treated with telmisartan were not clinically worse compared with those in the group receiving ramipril. Which one of the following best describes this type of study design? A. Equivalence. B. Intention-to-treat. C. Noninferiority. D. Superiority.

Answer C: Noninferiority. (This type of analysis is considered a noninferiority study because it aims to determine whether one treatment is noninferior to (or no worse than) another treatment (Answer C is correct). An equivalence study aims to determine whether two treatments are the same (e.g., a bioequivalence study) (Answer A is incorrect). An intention-to-treat analysis defines a certain type of analysis rather than a type of study design (Answer B is incorrect). A superiority study is designed to detect differences between two groups (Answer D is incorrect).)

Alerts to prescribers, nurses, or pharmacists are being considered for new oral anticoagulants used within an acute care health system. Which one of the following alerts would best prevent an anticoagulation-related medication error? A. Notify prescribers that dabigatran results in significantly less intracerebral hemorrhage than warfarin when used as a stroke prevention strategy in atrial fibrillation. B. Notify pharmacists of a patient's serum creatinine level before warfarin is verified. C. Notify nurses that dabigatran capsules should not be opened, chewed, or crushed. D. Notify nurses that dabigatran may cause dyspepsia.

Answer C: Notify nurses that the dabigatran capsule should not be opened, chewed, or crushed. (The dabigatran capsule cannot be opened or manipulated in any way because doing so increases the bioavailability significantly and exposes the patient to a greater degree of anticoagulation than intended (Answer C is correct). Noting a benefit with dabigatran versus warfarin may be a reasonable way to increase the use of dabigatran, but it is not a medication-error prevention strategy (Answer A is incorrect). Warfarin use is not significantly affected by renal clearance (Answer B is incorrect). Although it is useful to acknowledge potential ADRs, this approach is not likely to minimize the chance of a medication error (Answer D is incorrect).)

A 72-year-old woman is admitted to the ICU after an episode of cardiac arrest with successful resuscitation. She was intubated during the code and is being mechanically ventilated. A nasogastric tube is in place, and she is being fed enternally. She is tolerating the tube feedings well without residuals per nasogastric aspirate. Her current drugs include amiodarone 200 mg twice daily, simvastatin 20 mg once daily at bedtime, and subcutaneous heparin 5000 units every 12 hours. Her medical history includes atrial fibrillation, hyperlipidemia, and erosive esophagitis. Which one of the following is the best agent to recommend for stress-related mucosal disease (SRMD) prophylaxis in this patient? A. Famotidine 20 mg intravenously every 12 hours. B. Esomeprazole 20 mg intravenously every 24 hours. C. Omeprazole suspension 20 mg once daily by nasogastric tube. D. Dexlansoprazole 60 mg once daily by nasogastric tube.

Answer C: Omeprazole suspension 20 mg once daily by nasogastric tube. (The patient's medical history includes erosive esophagitis, which will require PPI therapy, not an H2RA (Answer A is incorrect). In addition, the patient has a nasogastric tube in place, which can be used for drug administration, if feasible. Because the patient has been tolerating enteral feedings well, indicating a functioning gut, administration by nasogastric tube is preferred, and intravenous therapy is not required (Answer A and Answer B are incorrect). Dexlansoprazole is relatively more costly than omeprazole, which is available in a generic form, and the dosing listed for dexlansoprazole is also incorrect (the correct dose is 30 mg) (Answer D is incorrect). Omeprazole suspension is effective in preventing SRMD when administered enterally; therefore, it is the most appropriate agent for preventing SRMD in this patient (Answer C is correct).)

A 56-year-old man with a history of ischemic cardiomyopathy (left ventricular ejection fraction, 25%) was recently admitted to the hospital with acute decompensated heart failure (HF). During your morning rounds in the coronary care unit, his telemetry monitor suddenly reveals ventricular fibrillation (V F). On entering his room, you see that the patient is unconscious, with no blood pressure or pulse. Cardiopulmonary resuscitation (CPR) is immediately initiated. Which of the following is the best initial step to take in the treatment of this patient? A. Amiodarone 300 mg administered by intravenous push. B. Epinephrine 1 mg administered by intravenous push. C. One shock of 200 joules with a biphasic defibrillator. D. Atropine I mg administered by intravenous push.

Answer C: One shock of 200 joules with a biphasic defibrillator. (According to the 2010 American Heart Association (AHA) guidelines for CPR and emergency cardiovascular care (ECC), in patients experiencing a witnessed cardiac arrest because of pulseless ventricular tachycardia (VT) or VF, defibrillation should be instituted immediately if a defibrillator is available on site. Because this patient is experiencing VF in an inpatient setting, a defibrillator is readily available. Biphasic defibrillators have a higher first-shock efficacy than monophasic defibrillators and are increasingly used in clinical practice (Answer C is correct). If a biphasic defibrillator is used, the dose of the shock to be used is device-specific; however, a dose of 200 joules can be used for the initial shock if the effective dose range of the defibrillator is unknown. Delivery of only one shock at a time is recommended to minimize interruptions in chest compressions. Vasopressor therapy with either epinephrine or vasopressin can be initiated if VF persists after the delivery of one or two shocks plus CPR (Answer B is incorrect). Initiating antiarrhythmic therapy with intravenous amiodarone is not recommended unless VF persists after the delivery of two or three shocks plus CPR and after the administration of a vasopressor (Answer A is incorrect). Atropine is only used for the treatment of bradycardia, not for VF (Answer D is incorrect).)

A clinician wants a self-rated scale for patients to evaluate their need for treatment for depression as they sit in the waiting room. Which one of the following scales would be most appropriate? A. Hamilton Rating Scale for Depression (HAM-D). B. Montgomery-Asberg Depression Scale (MADRS). C. Patient Health Questionnaire 9-1tem Depression Module (PHQ-9). D. Global Assessment of Functioning (GAF).

Answer C: Patient Health Questionnaire 9-1tem Depression Module (PHQ-9). (The PHQ-9 is the best choice because it is brief (5 minutes or less), and it can be either self-administered or administered by clinical staff. Because patients can complete the questionnaire themselves, it is a good choice for evaluation in a waiting room area. In addition, it does not take clinical staff long to administer in patients who may have low health literacy or who may need assistance in completing it (Answer C is correct). Both the MADRS and HAM-D are clinician-rated rating scales that require ample time to complete; they could not be completed in a waiting area (Answer A and Answer B are incorrect). The GAF measures global functioning, not depression (Answer D is incorrect).)

As part of a workgroup assigned to implement a retrospective drug use review (DUR) program in a managed care organization, you are evaluating potential drug-related problems. Which one of the following scenarios would be best evaluated by a retrospective DUR program? A. Patients receiving warfarin and trimethoprim/sulfamethoxazole concurrently. B. Patients taking two 5-mg tablets of simvastatin a day. C. Patients with diabetes who are not receiving statin therapy. D. Patients who received another patient's prescription.

Answer C: Patients with diabetes who are not receiving statin therapy. (Patients receiving warfarin and trimethoprim/sulfamethoxazole require timely intervention because of a major drug interaction; this scenario is not appropriate for a retrospective DUR (Answer A is incorrect). The pharmacist can act by contacting the prescriber to determine whether alternative therapy is appropriate, or at least consult the patient on the signs and symptoms of bleeding and a monitoring plan. A retrospective DUR can occur months after the medication is filled. By the time the potential problem is found, the patient has completed therapy or an ADR has occurred. Although it may be of financial benefit to the monitoring plan, a retrospective DUR in a patient who continues to take two 5-mg tablets instead of one 10-mg tablet would not be clinically significant because the patient will not be harmed, and little impact on patient morbidity or mortality will result (Answer B is incorrect). The most clinically significant potential drug-related problem would occur in patients with diabetes who are not taking a statin (Answer C is correct). A retrospective review aims to detect patterns in the prescribing or dispensing of drugs. Claims data can be reviewed against national treatment guidelines to determine whether most patients are being treated appropriately. If many patients are not being treated appropriately, a clinical pharmacy program can be created. Patients who received another patient's prescription would be handled by medication error identification and reporting (Answer D is incorrect). Pharmacy claims data are insufficient to determine whether a patient received the wrong medication. A patient picking up a prescription intended for another patient would be an event that would have to be reported.)

A 72-year-old man is in the medical intensive care unit with sepsis secondary to a central line infection. He is currently receiving a dopamine infusion intravenously at 15 mcg/kg/minute for hypotension. His blood pressure is 90/62 mm Hg, heart rate 150 beats/minute, and CVP 10 mm Hg. Which one of the following would best treat this patient's tachycardia? A. Metoprolol 5 mg intravenously for one dose. B. Dofetilide 250 mg orally twice daily. C. Diltiazem infusion intravenously 5 to 15 mg/hr titrated to heart rate. D. Change from dopamine to norepinephrine infusion.

Answer D: Change from dopamine to norepinephrine infusion. (Dopamine has ß-adrenergic activity that can increase heart rate. Rather than treating this patient's tachycardia with rate-slowing agents (Answer A, Answer B, and Answer C are incorrect), the causative agent, dopamine, should be discontinued and blood pressure maintained with an alternative agent such as norepinephrine that has less ß-adrenergic activity and more a-adrenergic activity (Answer D is correct).)

A 50-year-old African American man presents with bowel perforation and receives a diagnosis of stage 11B rectal cancer. He has a primary resection, followed by 6 months of adjuvant chemotherapy because of the high-risk pathologic features of the resected specimen. Three years later, at a follow-up appointment, routine surveillance imaging shows one 10-cm liver lesion in the right lobe and two distinct 3-cm liver lesions in the left lobe that are confirmed by biopsy as adenocarcinoma of the colon. All laboratory results are within normal limits except for ALK 300 U/L. The patient is a candidate for resection. Which one of the following is the best systemic chemotherapy option for this patient? A. Neoadjuvant irinotecan, fluorouracil, and leucovorin. B. Adjuvant fluorouracil and leucovorin. C. Perioperative oxaliplatin, fluorouracil, and leucovorin. D. Perioperative fluorouracil and leucovorin.

Answer C: Perioperative oxaliplatin, fluorouracil, and leucovorin. (Perioperative chemotherapy was evaluated in a randomized study that showed a significantly better progression-free survival with perioperative chemotherapy than with resection alone. Studies evaluating neoadjuvant irinotecan, fluorouracil, and leucovorin and adjuvant fluorouracil and leucovorin have not shown a significant progression-free survival advantage over resection alone (Answer A and Answer B are incorrect). Perioperative fluorouracil and leucovorin is not the best answer because fluorouracil and leucovorin alone are not sufficient (Answer D is incorrect). Adding oxaliplatin to this therapy is necessary to provide improved response rates and an overall survival advantage (Answer C is correct).)

A 60-year-old woman who received a diagnosis of PD 3 years ago presents for a routine follow-up. She lives at home with her husband. Physical examination reveals a mild resting tremor of both upper extremities, with predominance for the right. Her gait is stable, but moderate bradykinesia is noted, and she requires assistance when rising from a chair. Her husband reports that she was been doing well until the past year, when she experienced noticeable deterioration in her ability to perform activities of daily living (e.g., bathing, dressing, using eating utensils). This decline has resulted in the cancellation of many social and travel plans. She is mentally alert and oriented and reports no sleep difficulties. Her medical history is significant for severe depression, currently managed with phenelzine 15 mg three times/day. She is not taking any drugs for her PD. Which one of the following is best to recommend for this patient? A. Apomorphine. B. Entacapone. C. Pramipexole. D. Rasagiline.

Answer C: Pramipexole. (The primary goal is to provide this patient with timely relief of her bradykinesia and rigidity, which are impairing her functional abilities. Pramipexole provides symptomatic relief of motor symptoms, and the patient has no contraindications to its use (Answer C is correct). Apomorphine is reserved for patients experiencing motor fluctuations (Answer A is incorrect). Entacapone is effective only when used as an adjunct to levodopa (Answer B is incorrect). The use of rasagiline is contraindicated with nonselective monoamine oxidase inhibitors such as phenelzine (Answer D is incorrect).)

Your research team wants to assess causal effects between anticoagulants and bleeding adverse events. Which one of the following statistical methods would be best to use? A. Multivariate analysis. B. Regression analysis. C. Propensity score-matching analysis. D. Logistic regression analysis.

Answer C: Propensity score-matching analysis. (The best method for assessing causal effects from practice-based observational data is propensity score-matching analysis (Answer C is correct). Because of the nature of practice-based research involving observational data, propensity score-matching analysis minimizes selection bias by applying statistical matching criteria to samples in comparison groups. Multivariate, regression, and logistic regression analyses assess associations, not causal inferences (Answer A, Answer B, and Answer D are incorrect).)

A 53-year-old woman is hemodynamically stable after surgery for a ruptured appendix, but she remains intubated and requires mechanical ventilation and a sedative agent to assist with ventilator synchrony. Which of the following agents, titrated to an appropriate sedation goal, would be best to recommend for this patient? A. Midazolam intravenous infusion 1-5 mg/hour. B. Lorazepam 2 mg intravenously every 6 hours as needed. C. Propofol 10-20 mcg/kg/minute. D. Pentobarbital intravenous infusion I mg/kg/hour.

Answer C: Propofol 10-20 mcg/kg/minute. (Current guidelines recommend nonbenzodiazepine-based regimens such as dexmedetomidine or propofol as first-line agents for sedation (Answer C is correct) because of the increased incidence of delirium and ventilator days with benzodiazepines (Answer A and Answer B are incorrect). Pentobarbital is not recommended to be used for routine sedation in the current guidelines. (Answer D is incorrect).)

Some trials do not find a statistically significant difference in an outcome (i.e., "negative" trials). These trials are less likely to be found in the medical/pharmacy literature. Which one of the following is the best description of this phenomenon? A. Selection bias. B. Channeling bias. C. Publication bias. D. Misclassification bias.

Answer C: Publication bias. (Publication bias is the term used to describe the situation when negative trials are not as prevalent in the literature because of perceptions (e.g., that they are less important to publish). This is an important consideration when performing a systematic review/meta-analysis because investigators may have to look at sources other than the data published in medical/pharmacy journals to ascertain all the existing data on a particular intervention (Answer C is correct). Selection bias is a statistical bias in which there is an error in choosing the individuals or groups to take part in a scientific study (Answer A is incorrect). Channeling bias is a form of allocation bias in which drugs with similar therapeutic indications are prescribed to groups of patients with prognostic differences. Claimed advantages of a new drug may channel it to patients with special preexisting morbidity, with the consequence that disease states can be incorrectly attributed to use of the drug (Answer B is incorrect). Misclassification bias can occur if errors are made when categorizing a patient exposure or outcome (Answer D is incorrect).)

A 3-day-old boy born at 24 weeks' gestation (birth weight 590 g [1.3 lb]) is currently receiving a parenteral nutrition (PN) solution at 2.7 mL/hour; this solution provides dextrose 9.5 mg/kg/minute and amino acids 3 g/kg/day. He is also receiving a 20% intravenous fat emulsion at 0.2 mL/hour. On his morning laboratory assessment, his serum glucose concentration was 250 mg/dL; when repeated 3 hours later, it was 210 mg/dL. Other laboratory values obtained today include albumin 2.8 g/dL and SCr 0.4 mg/dL. Which one of the following is best to recommend for this patient? A. Continue PN as previously written. B. Start an intravenous insulin infusion at 0.03 unit/hour. C. Reduce the dextrose infusion by 20%. D. Give regular insulin 0.5 unit intravenously.

Answer C: Reduce the dextrose infusion by 20%. (An evaluation of the PN solution written for this neonate reveals a daily intake of 118 mL/kg, 73 kcal/kg, and 1.6 g fat/kg and a glucose infusion rate of 9.5 mg/kg/minute. His serum glucose is elevated (210—250 mg/dL). Because hyperglycemia can result in a myriad of complications and morbidities (e.g., dehydration, steatosis, respiratory compromise, electrolyte imbalance, intracranial hemorrhage, infection), the serum glucose concentration should be addressed (Answer A is incorrect). Exogenous insulin must be used with caution in the neonate. A subcutaneous regular insulin dose of 0.1—0.2 unit/kg (0.11 unit]dose) would be appropriate for this infant. Serial dilutions would be necessary for accurate measurement of this dose, which provides an opportunity for potential medication misadventure. The proposed dose of 0.5 unit is 0.8 unit/kg, which is an excessive intermittent dose (Answer D is incorrect). A continuous intravenous infusion of regular insulin may be considered in situations of persistent hyperglycemia, beginning with an infusion rate of 0.01 unit/kg/hour (0.006 unit/hour); thus, 0.03 unit/hour provides an excessive dose of insulin (Answer B is incorrect). Insulin infusion is complicated by unpredictable delivery with potential adsorption to tubing, immature and changing kidney function, electrolyte imbalances, and the requirement for frequent glucose monitoring. A more conservative approach and the best approach, given that this infant's glucose is only moderately elevated and trending downward, is to reduce the glucose infusion rate by 20% (Answer C is correct). Decreasing the glucose infusion rate would still maintain the glucose infusion rate within the minimal range of 6—8 mg/kg/minute as well as maintain acceptable energy intake while reducing the risk of hypoglycemia.)

A 55-year-old woman with a history of severe chronic obstructive pulmonary disease is admitted to the hospital after several days of worsening shortness of breath. She was recently discharged from the hospital after a similar episode and was feeling well until 3 days before this admission. She developed a productive cough, requiring an increase in her home oxygen administration and more frequent use of her metered-dose inhalers. On admission to the medical intensive care unit (ICU), she was anxious and markedly distressed with rapid shallow breaths. She was hypertensive (160/90 mm Hg), tachycardic (140 beats/minute), and tachypneic (28 breaths/minute). Her arterial blood gas (ABG) profile was found to be pH 7.30, PaCO2 59 mm Hg, PaO2 50 mm Hg, HCO3- 28 mEq/L, and Sa02 83% on 6 L/minute of oxygen by facemask, and she was immediately intubated. Which one of the following primary acid-base disturbances is most likely responsible for this patient's presentation and laboratory data? A. Metabolic acidosis. B. Metabolic alkalosis. C. Respiratory acidosis. D. Respiratory alkalosis.

Answer C: Respiratory acidosis. (This patient's ABG profile is consistent with a respiratory acidosis (Answer C is correct; Answer D is incorrect). The pH is below 7.40 (indicating an acidosis), and the PaC02 (an acid) is higher than normal (about 40 mm Hg). In chronic respiratory acidosis, the kidneys will conserve HCOC (a base) in an attempt to maintain a normal PH. This compensatory metabolic alkalosis is obvious in this patient, whose serum HC03- is 28 mEq/L (about 4 mEq/L higher than normal). The elevated HC03- concentration in this patient confirms the diagnosis of respiratory acidosis because the HC03- would be expected to be less than 24 mEq/L if the acidemia were attributable to a metabolic cause (Answer A and Answer B are incorrect).)

A study follows the experience of a group of people over a defined period. The observation period is started after the event of interest has already occurred; the study participants are followed to determine exposure status and outcome occurrence. The incidence rate in individuals who were exposed to the risk factor of interest is then compared with the incidence rate in individuals not exposed to that risk factor. Which one of the following best describes this type of study? A. Nested case-control. B. Prospective cohort. C. Retrospective cohort study. D. Prospective case-control.

Answer C: Retrospective cohort study. (Cohort studies follow the experience of a group of people who share a common characteristic. Because the event of interest has already occurred, the investigators identify the cohort of individuals from the past (historical cohort) and follow them forward in time to determine their exposure status and outcome occurrence (Answer C is correct). A nested case-control study has a clearly defined cohort and includes selection of control subjects (Answer A is incorrect). To be a prospective study, observation would have to begin before the outcome of interest occurs (Answer B is incorrect). This experiment is neither a case-control nor a prospective study (Answer D is incorrect).)

As the pharmacy specialist, you are asked to see a patient on the geriatric unit. The patient is a 60-year-old man with AD, H TN, and atrial fibrillation who presented to the emergency department with increased confusion, agitation, and aggression. His home drugs include donepezil 10 mg once daily, metoprolol 50 mg twice daily, and aspirin 325 mg once daily. The patient lives at home, uses a cane, and denies any alcohol, tobacco, or illicit drug use. On admission, his vital signs and pertinent laboratory values were as follows: blood pressure (BP) 177/90 mm Hg, heart rate (HR) 90 beats/minute, respiratory rate 19 breaths/minute, Na 125 mEq/mL, BUN 27 mg/dL, and SCr 0.9 mg/dL. After sodium and fluid status are corrected, the patient's confusion resolves; however, his aggression and agitation persist. A friend who comes to visit the patient states that this behavior has been typical for the past 2 months. Which one of the following is best to recommend for this patient's behavioral issues? A. Carbamazepine 200 mg twice daily. B. Lorazepam 0.5 mg twice daily. C. Risperidone 0.5 mg twice daily. D. Venlafaxine 75 mg twice daily.

Answer C: Risperidone 0.5 mg twice daily. (Risperidone 0.5 mg orally twice daily is the most appropriate drug to initiate at this time. It carries a low risk of extrapyramidal symptoms and is warranted for aggression and agitation (Answer C is correct). Carbamazepine is not appropriate because of its effects on sodium (which was recently corrected), as well as its potential to cause gait or balance disturbances, because this patient already uses a cane to ambulate (Answer A is incorrect). Lorazepam is not the preferred agent for the treatment of agitation and aggressive behavior (Answer B is incorrect). Venlafaxine is an antidepressant and a serotonin-norepinephrine reuptake inhibitor (SNRI), which should not be used in patients with uncontrolled HTN because of its potential to elevate BP. In addition, all SNRIs can potentially cause hyponatremia, which was the reason for the patient's current admission (Answer D is incorrect).)

A 68-year-old man has chronic idiopathic thrombocytopenic purpura (IT P) that is refractory to treatment with corticosteroids. The patient recently developed recurrent thrombocytopenia after a splenectomy. He has a history of hepatitis B. His platelet count is 22,000/mm3, and he has had frequent epistaxis. His ALT is 189 IU/L, AST is 175 IU/L, ALK is 153 IU/L, and SCr is 1 mg/dL. Which one of the following is the most appropriate therapy for this patient? A. Azathioprine. B. Eltrombopag. C. Romiplostim. D. Rituximab.

Answer C: Romiplostim. (About 40% of patients with ITP and a prior splenectomy achieve a durable response after taking romiplostim. Romiplostim has no contraindications for liver dysfunction or a history of hepatitis, so it is the most appropriate agent for the patient (Answer C is correct). Azathioprine can cause hepatotoxicity with chronic use; it should be avoided in this patient (Answer A is incorrect). Eltrombopag is hepatically metabolized and has a boxed warning for hepatotoxicity. A dosage reduction to 25 mg daily is recommended in patients with moderate to severe liver dysfunction; however, romiplostim is a safer alternative in this patient with a history of hepatitis B (Answer B is incorrect). Among HBsAg-negative/anti—HBc-positive patients with lymphoma treated with rituximab, cyclophosphamide, doxorubicin, vincristine, and prednisone, 25% developed hepatitis B virus reactivation compared with none receiving the same drugs without rituximab. Close monitoring for at least 6 months after rituximab therapy is required. If an effective alternative to rituximab is available, that agent should be used (Answer D is incorrect).)

A family medicine physician prescribes inexpensive antihypertensive drugs as first-line therapy for his patients. If that regimen fails because of ineffectiveness or intolerance, the physician then tries newer, more expensive therapy. For patients who cannot afford the more expensive therapy, the physician often distributes drug samples. Which one of the following best describes this situation? A. The physician has no financial interest in this situation; therefore, no conflict of interest (COL) exists. B. Because the physician is acting in the patient's best interest, any COI is superseded by this principle. C. Samples have potential COI because they establish a gift relationship. D. Samples have potential COI because they encourage a relationship.

Answer C: Samples have potential COI because they establish a gift relationship. (The appearance of a COI takes place when any gift is accepted. The acceptance of samples establishes the gift relationship (Answer C is correct). Samples also increase access to physicians' offices and are part of the process of positioning the practitioner for future obligation (and financial gain) and potential COI (Answer A is incorrect). Regarding the statement that the patient's best interest supersedes COI concerns, it may not be in the best interest of the patient to receive samples that then lead to the use of more expensive drugs (Answer B is incorrect). In addition, about one-third of the samples are used by physicians and staff, creating an even stronger gift relationship. Although it is true that this patient-physician relationship may be strengthened by the delivery of samples, this COI is lower level compared with the physician and pharmaceutical industry relationship (Answer D is incorrect).)

A 44-year-old African American man with chronic hepatitis C virus (HCV) infection, genotype 1B strain, who is naive to HCV therapy presents to the clinic for follow-up. Six months ago, a liver biopsy showed his disease to be grade 2, stage 2. His medical history includes hypertension, atrial fibrillation, hyperlipidemia, pruritus, and hypothyroidism. Current medications include hydrochlorothiazide, hydroxyzine, warfarin, calcium and vitamin D, and simvastatin. He has no known drug allergies. Pertinent laboratory values include the following: HCV RNA 952,631 IU/mL, AST 53 IU/L, ALT 63 IU/L, INR 2.6, albumin 4.2 g/dL, cholesterol 164 mg/dL, TG 194 mg/dL, HDL-C 46 mg/dL, and LDL-C 99 mg/dL. The patient's physician prescribes an anti-HCV regimen that includes boceprevir. Which one of the following agents should be discontinued in this patient? A. Hydroxyzine. B. Hydrochlorothiazide. C. Simvastatin. D. Warfarin.

Answer C: Simvastatin. (The protease inhibitors are metabolized through the cytochrome P450 (CYP) 3A4 system. This patient has a contraindication to simvastatin if it is prescribed with boceprevir. Boceprevir is a potent inhibitor of CYP3A4, and the coadministration of these two drugs may increase the risk of myopathy and rhabdomyolysis. Therefore, simvastatin must be discontinued before initiating a boceprevir-based anti-HCV regimen (Answer C is correct). Warfarin does not have to be discontinued, but the patient's INR levels should be monitored closely (Answer D is incorrect). Boceprevir is not known to interact with either hydroxyzine or hydrochlorothiazide (Answer A and Answer B are incorrect).)

A 56-year-old man with an acute MI 3 days ago is now being transferred from cardiac intensive care to the step-down unit. He says that heart disease is common in his family and that his father died at age 40. Before this event, the patient did not take any prescription drugs. and he does not currently receive any lipid-lowering drug therapy. He has no history of liver or kidney dysfunction. Results of a fasting lipid profile today are as follows: TC 216 mg/dL, LDL-C 135 mg/dL, HDL- 39 mg/dL, and TG 185 mg/dL. Which one of the following lipid management strategies would be best to recommend for this patient? A. Wait 6 weeks and recheck lipid profile. B. Start fenofibrate 145 mg daily. C. Start atorvastatin 80 mg daily. D. Start fluvastatin 20 mg daily.

Answer C: Start atorvastatin 80 mg daily. (This patient, immediately after an MI, is at high risk and should be initiated on a high-intensity therapy within 24 hours of admission. Waiting 6 weeks to recheck the lipid profile is not appropriate given that the patient may receive pleotropic benefit from statin use given early after MI (Answer A is incorrect). Fibrates are not associated with improved mortality and would not be indicated in this patient with only mild elevated TG (Answer B is incorrect). Fluvastatin is a low-to-moderate-intensity statin and would not offer sufficient potency for LDL-C reduction to benefit this very high-risk patient (Answer D is incorrect). Patients treated with less intensive regimens have considerably more recurrent events than patients treated immeadiately and aggressively with higher-potency statins, such as the addition of atorvastatin 80 mg daily (Answer C is correct).)

Two months ago, a 36-year-old ED nurse had an exposure at work from a patient with a positive sputum for Mycobacterium tuberculosis (T B). Four days ago, the nurse's purified protein derivative (PPD) test was read as 9 mm of induration. He has no symptoms such as cough or weight loss, and chest radiography is normal. Prior PPD tests showed no induration. The nurse has a medical history of diabetes and rheumatoid arthritis, and he has no known drug allergies. His home drugs include metformin 500 mg twice daily and prednisone 20 mg once daily. Which one of the following is best to recommend for this patient? A. Repeat the PPD test in another 2 weeks. B. Order an interferon-gamma release assay (IGRA) for confirmation. C. Start isoniazid daily for 9 months. D. Start isoniazid and rifampin daily for 2 months.

Answer C: Start isoniazid daily for 9 months. (About 10% of patients infected with TB develop reactivation disease. The greatest risk of developing an infection is within 2 years of infection. A PPD in a recent contact of a patient with TB is considered positive if the reaction is 5 mm of induration or more. Isoniazid given for 9 months is the preferred drug for treating latent TB infection (Answer C is correct). Starting several drugs is not required in this case (Answer D is incorrect), although isoniazid and rifapentine given once weekly for 12 weeks (by directly observed therapy) is an alternative treatment regimen. An IGRA is inappropriate because the PPD test can boost IGRA response and should be performed no more than 3 days after the PPD is placed (Answer B is incorrect). A repeat PPD test is not needed because the current test is positive (Answer A is incorrect).)

A 26-year-old woman who is HIV positive and antiretroviral therapy (ART) naive presents to the clinic for follow-up of her HIV infection. For the past several years, her CD4+ lymphocyte counts have steadily decreased. Her most recent CD44 lymphocyte count was 284 cells/mm3 with an HIV viral load of 42,000 copies/mL. After an extended discussion with her physician, it is determined that she should begin ART. Her most recent HIV genotyping results show no drug resistance. She is sexually active and states that her partner, who is HIV negative, uses condoms sporadically. She is opposed to the salpingectomy (tubal ligation) procedure because she desires to have children in the future. Considering the current guidelines and her risk of pregnancy, which one of the following is the most appropriate ART regimen for this patient? A. Tenofovir/emtricitabine and efavirenz. B. Zidovudine/lamivudine and nevirapine. C. Tenofovir/emtricitabine and atazanavir/ritonavir. D. Zidovudine/lamivudine and efavirenz.

Answer C: Tenofovir/emtricitabine and atazanavir/ritonavir. (According to the adult and adolescent guidelines for the treatment of HIV infection, ritonavir-boosted atazanavir plus tenofovir/emtricitabine is one of the recommended initial combination regimens for the patient who is antiretroviral (AR V) naive (Answer C is correct). Although tenofovir/emtricitabine/efavirenz is also recommended, it would not be the best choice in a woman with a high risk of pregnancy because efavirenz carries a risk of teratogenic complications in the newborn (Answer A is incorrect). Nevirapine is not an initial recommended agent in the treatment of a patient who is ARV naive and is not recommended in a patient with a CD4 count greater than 250 cells/mL (Answer B is incorrect). Zidovudine/lamivudine is recommended as first-line therapy in a pregnant patient with a new diagnosis of HIV infection but is not a recommended first-line nucleoside reverse transcriptase inhibitor in a nonpregnant adult, and efavirenz has teratogenic potential (Answer D is incorrect).)

A 75-year-old man with an enlarged prostate presents with new-onset urinary symptoms, including nocturia, decreased force of stream, hesitancy, and incomplete emptying. No complications are associated with these lower urinary tract symptoms; however, the patient desires drug therapy to treat his symptoms, and his physician asks you to recommend the most appropriate drug. Which one of the following would be most useful in deciding between the use of an (I -antagonist, a 50-reductase inhibitor (5α-ARI), or the combination in this patient? A. His American Urological Association (AUA) symptom score. B. His prostate-specific antigen (PSA) result. C. The approximate size of his prostate. D. His insurance coverage of the medications.

Answer C: The approximate size of his prostate. (Determining whether the patient might benefit from an U -antagonist, a 5-ARI, or both is enhanced by knowledge of the approximate size of the prostate. Both the M TOPS and CombAT studies found that combination therapy was more effective than monotherapy at preventing complications of benign prostatic hyperplasia (BPH). However, patients in the M TOPS study with a prostate size greater than 40 g had better results with the 5-ARI therapies than did the general population. This result was likely because of the increased benefit of reducing the larger prostate size, compared with treating only urinary symptoms, such as with (I -antagonist monotherapy (Answer C is correct). Although an AUA symptom score may help determine whether watchful waiting, drug therapy, or surgery is the most appropriate treatment, it will not help determine which drug therapy is most appropriate for the patient. In addition, the patient has already decided to take a drug and is not likely to want to pursue watchful waiting, even if it were an option on the basis of his AUA score (Answer A is incorrect). Although PSA test results do not help differentiate BPH treatment, they can help monitor adherence and efficacy in a patient taking a 5-ARI (Answer B is incorrect). Finally, insurance coverage may play a role in determining which individual (I -antagonist or 5-ARI is prescribed, but it will not affect the choice of one class versus the other because generic agents are available in both drug classes (Answer D is incorrect).)

A study was conducted to assess the relationship between two variables. The correlation coefficient (r) using Pearson correlation was determined to be 0.20, and the p value was reported to be less than 0.05. Which one of the following is the best interpretation of these findings? A. ANOVA would be a more appropriate means to perform this analysis. B. There is a strong and highly important relationship between the two variables. C. There is a statistically significant relationship between the two variables, but it is unlikely to be clinically important. D. There is no statistically significant relationship between the two variables.

Answer C: There is a statistically significant relationship between the two variables, but it is unlikely to be clinically important. (Because the p value is less than 0.05, there is a statistically significant relationship between the two variables, but this is not likely to be clinically important. The relationship is not likely to be clinically significant because the independent variable explains only 20% of the variability in the dependent variable (Answer C is correct; Answer B and Answer D are incorrect). Use of ANOVA would not be appropriate in this situation (Answer A is incorrect).)

A 55-year-old woman presents to the clinic with severe papulopustular rosacea. She has extensive papules and pustules across the central region of her face. She is very concerned about her appearance because her daughter is getting married in 2 months, and she would like to "get this under control" as soon as possible. Which one of the following is the most appropriate treatment for this patient? A. Topical metronidazole B. Topical tretinoin. C. Topical azelaic acid with oral doxycycline. D. Topical azelaic acid with ibuprofen.

Answer C: Topical azelaic acid with oral doxycycline. (Systemic therapy has a more rapid onset of treatment effect in managing moderate to severe papulopustular rosacea (Answer C is correct). Tetracycline antibiotics, oral metronidazole, and low-dose isotretinoin have been used for severe rosacea. Combination therapy with azelaic acid may also be beneficial in moderate to severe cases to decrease inflammation. Topical metronidazole or topical tretinoin alone would be options if the patient had mild rosacea or did not require such a rapid treatment effect (Answer A and Answer B are incorrect). Ibuprofen may be effective in severe cases of erythematotelangiectatic rosacea, but it is not as effective at reducing the papules and pustules associated with papulopustular rosacea (Answer D is incorrect).)

A 75-year-old man presents with a diagnosis of acute macular degeneration. He has a 60 pack-year history of smoking. He requests your advice about a multivitamin to prevent his condition from progressing. Which one of the following is best to recommend for this patient? A. Vitamin B complex with vitamin C. B. Multivitamin/multimineral. C. Vitamin C, vitamin E, and zinc. D. Vitamin C, vitamin D, and beta-carotene.

Answer C: Vitamin C, vitamin E, and zinc. (A combination of vitamin C, vitamin E, beta-carotene, zinc, and copper prevents the progression of acute macular degeneration (Answer A and Answer B are incorrect). Beta-carotene has been associated with an increased incidence of lung cancer and mortality, according to the the Beta-Carotene and Retinol Efficacy Trial. Because this patient has a 60 pack-year history of smoking, it is prudent to avoid products containing beta-carotene (Answer D is incorrect). Therefore, the combination of vitamin C, vitamin E, and zinc is the most appropriate regimen for this patient (Answer C is correct).)

For which one of the following pharmacist-provided medication therapy management (MT M) services would it be most appropriate to use the Outpatient Prospective Payment System (OPPS) for billing? A. Diabetes education at a community pharmacy. B. Hypertension therapy monitoring at a community pharmacy. C. Warfarin therapy monitoring at a hospital-based outpatient clinic. D. Contraception counseling at a physician's office outpatient clinic.

Answer C: Warfarin therapy monitoring at a hospital-based outpatient clinic. (Billing for services using OPPS can occur only when the service is provided in a hospital-based outpatient clinic; thus, warfarin therapy monitoring in a hospital-based outpatient clinic can be billed using OPPS (Answer C is correct). Diabetes education in a community pharmacy, hypertension therapy monitoring at a community pharmacy, and contraception counseling in a physician's office outpatient clinic are not eligible for OPPS billing because of where the service is provided (Answer A, Answer B, and Answer D are incorrect).)

You are contacted by a hospitalist at the local hospital to assist in developing guidelines for blood glucose concentration control in the inpatient setting. Which one of the following blood glucose concentration ranges would it be best to recommend for non-critically ill patients? A. 81-108 mg/dL. B. 81—120 mg/dL with a goal preprandial glucose of less than 108 mg/dL. C. 120—140 mg/dL with a goal preprandial glucose of less than 108 mg/dL. D. 150—180 mg/dL with a goal preprandial glucose of less than 140 mg/dL.

Answer D: 150-180 mg/dL with a goal preprandial glucose of less than 140 mg/dL. (The large multicenter randomized clinical trial NICE-SUGAR failed to show benefit and showed increased mortality among patients in the study arm with tight glycemic control (glucose goal of 81—108 mg/dL) compared with those having a glucose goal of less than 180 mg/dL (Answer A is incorrect). The American Association of Clinical Endocrinologists (AACE)/ADA guidelines recommend glucose concentrations of 150—180 mg/dL with a preprandial glucose of less than 140 mg/dL as glucose treatment goals (Answer D is correct). A goal preprandial glucose concentration of less than 108 mg/dL is too low (Answer B and Answer C are incorrect). Glycemic control is important in the hospital setting, but the target concentrations, in addition to the best methods for achieving them, still require study.)

A 25-year-old woman (weight 60 kg [132 lbs]) with a medical history of type I diabetes is brought to the ED because of polyuria, polydipsia, fatigue, and vomiting for 12 hours. She is tachypneic with a blood pressure of 100/65 mm Hg. She is very drowsy and oriented x I (person). Her laboratory values are: Na 145 mEq/L, CI 115 mEq/L, scr 1.5 mg/ dL, Glucose 630 mg/dL, K 3.8 mEq/L, C02 8 mEq/L, BUN 24 mg/dL, WBC 12.4 x 108 cells/mmy, Hgb 11.2 g/dL, platelet count 254,000/mm3, and phosphate 0.8 mg/dL. ABG values are pH 6.95, PC02 35 mm Hg, 85 mm Hg, I-ICOC 5 mEq/L, and Sa02 93%. Urinalysis is +1 blood, +3 ketones, esterases positive, and nitrites positive. Chest radiography is normal. Which one of the following is the best fluid therapy for this patient? A. A I-L bolus of 0.9% sodium chloride; then 5% dextrose/0.450/o sodium chloride with 40 mEq/L of potassium at 500 mL/hour. B. A I-L bolus of 0.9% sodium chloride; then 0.9% sodium chloride with 40 mEq/L of potassium at 500 mL/hour. C. A 2-L bolus of 0.9% sodium chloride; then 0.9% sodium chloride with I ampule of sodium bicarbonate and 20 mEq/L of potassium at 250 mL/hour. D. A 2-L bolus of 0.9% sodium chloride; then 0.45% sodium chloride with 40 mEq/L of potassium at 500 mL/hour.

Answer D: A 2-L bolus of 0.9% sodium chloride; then 0.45% sodium chloride with 40 mEq/L of potassium at 500 mL/hour. (Giving a l- to 2-L bolus of normal saline and then changing to a 0.45% solution of sodium chloride, given that her corrected sodium is high, is the most appropriate fluid therapy in this patient (Answer D is correct). Continuing with normal saline is not the best practice because this patient's corrected sodium is 153 mEq/L (Answer B is incorrect). Although some clinicians would administer sodium bicarbonate because of the very low PH, it should not be put in normal saline because this administration would contribute to an already existing hypernatremia (Answer C is incorrect). The fluid should not be changed to 5% dextrose/0.45% sodium chloride until a glucose of about 250 mg/dL has been reached, although the potassium content in the solution would be appropriate for this patient's serum concentration (Answer A is incorrect).)

Which one of the following patients is at greatest risk of developing a drug-related osteoporotic fracture? A. A 28-year-old white woman receiving depot medroxyprogesterone acetate and pantoprazole for the past 6 months. B. A 60-year-old African American man receiving rosiglitazone and pantoprazole for the past 6 months. C. A 46-year-old Hispanic woman receiving olanzapine and citalopram for the past 3 years. D. A 63-year-old African American woman receiving letrozole and pioglitazone for the past 3 years.

Answer D: A 63-year-old African American woman receiving letrozole and pioglitazone for the past 3 years. (Regardless of ethnicity, a postmenopausal woman receiving an aromatase inhibitor (A1) (letrozole) and a thiazolidinedione (pioglitazone) would be at increased risk of drug-related osteoporotic fractures. Both AIs and thiazolidinediones reduce aromatase activity, thereby lowering functional estrogen concentrations in postmenopausal women with type 2 diabetes mellitus. A meta-analysis of randomized controlled trials for the use of thiazolidinediones in type 2 diabetes mellitus suggests women are at greater risk than men for thiazolidinedione-related fractures. When a thiazolidinedione is combined with an A1, there will likely be an additive effect (Answer D is correct). Depot medroxyprogesterone acetate has not been associated with osteoporotic fractures in young white women. Pantoprazole is unlikely to be associated with fractures in a 28-year-old woman because epidemiologic studies included a much older population (older than 70 years), therefore making the results not generalizable to premenopausal women (Answer A is incorrect). Studies show a sex difference for risk of fracture associated with thiazolidinediones, with one meta-analysis finding no association between fracture and thiazolidinedione use in men. Other epidemiologic studies observed a relationship between fracture and the use of pioglitazone, but not rosiglitazone, in men. Although data suggest that men are at greater risk of fracture related to proton pump inhibitors, these data are probably not generalizable in a 60-year-old man because these epidemiologic studies involved a population 70 years and older (Answer B is incorrect). Olanzapine is a prolactin-sparing antipsychotic agent. Prolactin-sparing agents do not have the same impact on bone mineral density as the prolactin-raising conventional antipsychotic agents. Although SSRIs have been associated with an increase in fractures, most studies involve patients older than 65 years (Answer C is incorrect).)

You are asked to prepare a presentation on drug-induced acute kidney injury (AKI). Which one of the following would be most important to discuss? A. Radiocontrast media and acute interstitial nephritis (AIN). B. Angiotensin-converting enzyme (ACE) inhibitors and acute glomerulonephritis. C. Penicillin G and acute tubular necrosis (AT N). D. Acyclovir and tubular obstruction.

Answer D: Acyclovir and tubular obstruction. (Acyclovir administration may lead to the deposition and accumulation of insoluble crystals, resulting in tubular obstruction (Answer D is correct). Radiocontrast media are not associated with AIN but are known to cause ATN. Administration of contrast media can cause an osmotic diuresis and then direct tubular toxicity (Answer A is incorrect). Although ACE inhibitors are considered nephroprotective, they may contribute to functional prerenal AKI in some situations; ACE inhibitors are not associated with glomerulonephritis (Answer B is incorrect). The most common AKI associated with penicillins is AIN, which is the result of a hypersensitivity reaction, not ATN as listed here (Answer C is incorrect).)

As the consultant pharmacist, you are reviewing the chart of a 78-year-old man recently admitted to a long-term care facility. His records show he received a pneumococcal vaccination 4 years ago with the pneumococcal conjugate 13-valent vaccine (PCV13). Which one of the following is best to recommend for this patient? A. No additional vaccination is needed. B. Revaccinate with an additional 0.5-mL dose of PCV13. C. Administer 0.25 mL of PPSV23. D. Administer 0.5 mL of PPSV23.

Answer D: Administer 0.5 mL of PPSV23. (Vaccination with PCV13 is recommended only for certain high-risk adults. In this patient, the previous administration of PCV13 administration was erroneous, and it would not currently offer the correct protection. The patient should receive the proper 23-valent, PPSV23, at the standard dosing of 0.5 mL (Answer D is correct and Answer C is incorrect). Administering PSV13 again would duplicate the same strains of vaccine that were already given (Answer B is incorrect). Vaccination with the correct pneumococcal vaccine is warranted (Answer A is incorrect).)

A 45-year-old kidney transplant recipient with a medical history of diabetes is seen in the outpatient kidney transplant clinic. Today's laboratory values include Na 142 mEq/L, K 3.2 mEq/L, BUN 45 mg/dL, SCr 1.4 mg/dL, and tacrolimus 13.5 ng/mL. The last SCr on record (2 weeks ago) is 1.1 mg/dL. Immunosuppression includes tacrolimus 1 mg twice daily, mycophenolate sodium 720 mg twice daily, and prednisone 10 mg daily. The patient's blood pressure is 165/94 mm Hg, and heart rate is 76 beats/minute. Which one of the following is the best antihypertensive to initiate for this patient? A. Diltiazem 120 mg daily. B. Lisinopril 10 mg daily. C. Furosemide 40 mg daily. D. Amlodipine 10 mg daily.

Answer D: Amlodipine 10 mg daily. (Diltiazem is an inhibitor of cytochrome P450 (CYP) 3A4 and can lead to increased levels of tacrolimus (Answer A is incorrect). Lisinopril could falsely elevate SCr in this patient, with an acute rise in creatinine that could be attributable to the elevated tacrolimus level, and would therefore not be the best choice (Answer B is incorrect). Furosemide could further reduce K and is not a first-line agent for hypertension (Answer C is incorrect). The best answer is to initiate amlodipine, a calcium channel blocker that can induce vasodilation and potentially reverse the vasoconstriction induced by tacrolimus (Answer D is correct).)

An 80-year-old Hispanic woman (weight 70 kg) has a medical history of HTN, T2DM, systemic lupus erythematosus, and transient ischemic attack (TIA). She has now received a diagnosis of nonvalvular AF. Her SCr is 2.1 mg/dL, and her other electrolytes are within normal limits. Using the CHA2DS2-VASc score, which of the following statements best estimates her risk of stroke and provides the most appropriate therapy recommendation? A. CHA2DS2-VASc score 5; initiate apixaban 5 mg twice daily. B. CHA2DS2-VASc score 5; initiate aspirin 325 mg daily. C. CHA2DS2-VASc score 7; initiate dabigatran 150 mg twice daily. D. CHA2DS2-VASc score 7; initiate rivaroxaban 15 mg daily.

Answer D: CHA DS -VASc score 7; initiate rivaroxaban 15 mg daily. (The CHA2DS2-VASc score stratifies stroke risk by scoring 2 points for a prior stroke or TIA, 2 points for age older than 75 years, I point for female sex, 1 point for HTN, I point for DM, and 1 point for HF. This patient has a CHA2DS2 VASc score of 7: I point for female sex, 1 point for HTN, 1 point for diabetes, 2 points for age older than 75 years, and 2 points for TIA (Answer A and Answer B are incorrect). Her CHA DS -VASc score is 7; rivaroxaban 15 mg daily is recommended in patients with a CrCl of 15—30 mL/minute. This patient's calculated CrCl is 24.9 mL/minute/1.73m2 (Answer D is correct). In patients with a CHA DS -VASc score of 2 or greater, oral anticoagulant therapy with an adjusted-dose V KA (vitamin K antagonist) (INR 2—3), a direct thrombin inhibitor (dabigatran), or an oral factor Xa inhibitor (e.g., rivaroxaban, apixaban) is recommended, unless contraindicated (Answer B is incorrect). A dosage reduction of oral anticoagulants is necessary in patients with moderate renal impairment. The twice-daily doses of apixaban 5 mg and dabigatran 150 mg are not appropriate for this patient (Answer A and Answer C are incorrect).)

A 72-year-old woman has stage IV chronic kidney disease and hypertension, for which she is currently receiving lisinopril. She presents to the ED with worsening muscle weakness. Her serum potassium concentration is 7.2 mEq/L. An electrocardiogram (ECG) reveals peaked T-waves and widened QRS complex. Which one of the following is the best initial therapy for this patient? A. Sodium Polystyrene sulfonate. B. Sevelamer. C. Paricalcitol. D. Calcium Gluconate.

Answer D: Calcium gluconate. (This patient has severe hyperkalemia and ECG manifestations. Calcium gluconate, given intravenously, can increase the threshold potential and reverse the cardiovascular manifestations (Answer D is correct). Sodium polystyrene sulfonate is an oral agent that treats hyperkalemia; however, it has a delayed onset of action and does not reverse ECG changes associated with hyperkalemia, so it is not appropriate for initial management (Answer A is incorrect). Sevelamer is a phosphate binder and paricalcitol is a vitamin D analog; therefore they are not beneficial for the treatment of hyperkalemia and are inappropriate choices (Answer B and Answer C are incorrect).)

A 26-year-old man is brought to the trauma bay by emergency medical services after being thrown from his motorcycle at a high rate of speed. His helmet is intact without obvious cracks or deformity. He has a large open wound (about 12 cm), extensive soft tissue injury, and exposed bone secondary to an acute tibia/fibula fracture. Which one of the following regimens is best to recommend for antimicrobial prophylaxis in this patient? A. Nafcillin. B. Vancomycin and high-dose penicillin. C. Clindamycin and ciprofloxacin. D. Cefazolin and gentamicin.

Answer D: Cefazolin and gentamicin. (This patient has a type III open fracture because of the wound size, extensive soft tissue damage, and exposed bone. Although the wound is not infected, antimicrobial prophylaxis is recommended for type III open fractures according to guidelines. Nafcillin provides only gram-positive coverage, which might be adequate for a type I or type II fracture, but is insufficient coverage in this case (Answer A is incorrect). The choice of antibiotics is based on the extent of the injury and the environment. For example, high-dose penicillin is added to gram-positive coverage in cases of fecal or clostridial contamination, which is not evident in this case (Answer B is incorrect). This patient requires both gram-positive and gram-negative antimicrobial coverage. Fluoroquinolones may delay fracture healing and should generally be avoided (Answer C is incorrect). Cefazolin and gentamicin provide the needed coverage (Answer D is correct).)

A 35-year-old man presents to the ED with a 3-day concern about abdominal pain, nausea and vomiting, and fever. His vital signs include BP 120/62 mm Hg, HR 88 beats/minute, RR 25 breaths/minute, and temperature 102.6^oF (39.2^oC). He has no significant medical history and currently takes no medications. Abdominal CT reveals a perforated appendicitis. Which one of the following is best to initiate in this patient? A. Piperacillin/tazobactam 3.375 g intravenously every 6 hours. B. Ampicillin/sulbactam 1.5 g intravenously every 6 hours. C. Meropenem 1 g intravenously every 6 hours. D. Cefoxitin 2 g intravenously every 6 hours.

Answer D: Cefoxitin 2 g intravenously every 6 hours. (This patient has a community-acquired, mild- to moderate-severity intra-abdominal infection that may include perforated appendicitis. Piperacillin/tazobactam and meropenem are indicated for health care—associated, severe, or complicated infections (Answer A and Answer C are incorrect). Ampicillin/sulbactam is no longer recommended as empiric therapy because of high rates of E. coli resistance (Answer B is incorrect). Cefoxitin is an acceptable treatment option, providing broad gram-negative and anaerobic coverage as well as some gram-positive coverage (Answer D is correct).)

A 2-year-old girl is evaluated by her primary care physician for a new complaint of 1 week of diarrhea. The patient has no abdominal pain, fever, or blood in the stool. She has had no recent changes in her diet or sick contacts. Her medical history is significant for a new diagnosis of GERD I month ago by esophageal pH monitoring, and she has been receiving omeprazole 10 mg every morning. Which one of the following is best to recommend for this patient? A. Discontinue omeprazole and start cimetidine. B. Change omeprazole to 5 mg orally two times/day. C. Continue current therapy and add loperamide. D. Change to lansoprazole.

Answer D: Change to lansoprazole. (Diarrhea is a relatively common adverse effect with PPI therapy (5%—10% of patients). Other causes of acute diarrhea (e.g., gastroenteritis) must be excluded. Recommendations for patients who develop diarrhea after receiving a PPI are to change to a different PPI (Answer D is correct) or to decrease the PPI dosage. Modifying the regimen to twice daily is not a decrease in total daily dose and will likely not decrease the diarrhea (Answer B is incorrect). Diarrhea usually does not subside with continued therapy, even with the use of antidiarrheal agents. In addition, loperamide is indicated only for acute diarrhea (e.g., with gastroenteritis) and for chronic diarrhea associated with inflammatory bowel disease; it is not recommended to combat an adverse effect of an agent known to cause diarrhea (Answer C is incorrect). It is not necessary to discontinue PPI therapy when a patient experiences diarrhea (Answer A is incorrect).)

A 60-year-old patient is admitted to the medical intensive care unit for pneumonia. He is placed on unfractionated heparin 5000 units subcutaneously every 8 hours for deep venous thrombosis (DVT) prophylaxis. During his treatment, the patient develops acute renal failure requiring dialysis. Which one of the following anticoagulation strategies would be best to recommend for this patient? A. Change to enoxaparin. B. Decrease the heparin dose to 5000 units every 12 hours. C. Discontinue heparin; DVT prophylaxis no longer warranted. D. Continue current treatment.

Answer D: Continue current treatment. (No dose adjustment for unfractionated heparin is necessary in renal impairment (Answer B is incorrect). Unfractionated heparin is an appropriate agent for DVT prophylaxis for a patient with several CHEST risk factors according to the CHEST guidelines (Answer D is correct). This therapy should not be discontinued in a patient with acute renal failure (Answer C is incorrect). Enoxaparin should be avoided in patients with acute renal failure and is not approved for use in patients requiring dialysis (Answer A is incorrect).)

A 72-year-old man with Parkinson disease and atrial fibrillation fell 2 days ago, hitting his head on concrete and experiencing a loss of consciousness. He was immediately transported to the emergency department, where it was determined that he had a large, acute, subdural hematoma. He was treated by neurosurgery with burr holes and evacuation of the clot within several hours of the injury. Currently, he is oriented to person and responds slowly to painful stimuli. His home drugs before admission included warfarin and ropinirole. Because of concerns about seizures, phenytoin was initiated immediately after surgery. Which one of the following is the best approach to antiepileptic drug therapy use in this patient? A. Change phenytoin to levetiracetam. B. Continue phenytoin for 2 years after his surgery. C. Discontinue phenytoin immediately. D. Continue phenytoin for 7 days after surgery and then stop.

Answer D: Continue phenytoin for 7 days after surgery and then stop. (Seizures can occur after a head injury, and several studies have considered seizure prophylaxis. The only study of seizure prophylaxis with antiepileptic agents in head injury that showed any benefits was with phenytoin. In that study, phenytoin was only effective in preventing seizures during the first week after the head injury (Answer D is correct). Continuing phenytoin beyond that point did not show any benefit (Answer B is incorrect). Studies with newer antiepileptic agents in this setting are lacking (Answer A is incorrect). Because his head injury and surgery occurred only 2 days ago, phenytoin should be continued (Answer C is incorrect).)

A 47-year-old man has a new diagnosis of diabetic nephropathy. He is unemployed and has no health insurance or prescription coverage. His estimated glomerular filtration rate is stable at 34 mL/minute/1.73m2 (stage 3 CKD). His laboratory results are calcium 8.5 mg/dL, albumin 3.8 g/dL, phosphorus 4.1 mg/dL, and intact parathyroid hormone (iPTH) 52 pg/mL. Which one of the following best describes the patient's condition and his optimal management? A. Secondary hyperparathyroidism requiring aggressive vitamin D therapy. B. Probable renal osteodystrophy, which should be evaluated by bone biopsy. C. Hyperphosphatemia and at risk of renal osteodystrophy; requires a calcium-containing phosphate binder with meals. D. Current laboratory parameters at goal; should be rechecked in 6—12 months.

Answer D: Current laboratory parameters at goal; should be rechecked in 6—12 months. (This patient has stage 3 CKD. Although patients may develop secondary hyperparathyroidism at this stage, he has no signs or symptoms, and his laboratory values are all within the normal range for stage 3 CKD. His iPTH is 52 pg/mL, within the range of 35—70 pg/mL, so vitamin D therapy is not indicated (Answer A is incorrect). Because his calcium, phosphorus, and iPTH are within normal range for stage III CKD, these markers do not suggest bone disease, so a bone biopsy (an invasive procedure) is not warranted (Answer B is incorrect). Adding a calcium-based phosphate binder is incorrect because his phosphorus level is within the normal limits of 2.7—4.6 mg/dL (Answer C is incorrect). In stage 3 CKD, laboratory parameters should be measured every 6—12 months (Answer D is correct).)

An 81-year-old woman presents with a cough producing thick, yellow mucus; shortness of breath; and sore throat for about 4 days. Her drug allergy history includes doxycycline and amoxicillin, which have caused nausea and vomiting; and clarithromycin, penicillin, and sulfa drugs, which have caused hives. Her pertinent physical examination findings include an erythematous throat without exudates and a negative rapid streptococcal test. Which one of the following is best to recommend for this patient? A. Treat her with oral penicillin. B. Treat her with intramuscular penicillin. C. Treat her with oral clindamycin. D. Do not treat her with antibiotics.

Answer D: Do not treat her with antibiotics. (This patient has had symptoms for 4 days, and her rapid streptococcal antigen screening was negative; therefore, her infection is likely viral and does not require antibiotic therapy (Answer D is correct). If antibiotics had been warranted, her allergy to oral penicillin would have precluded the use of penicillin; a severe allergic reaction would be too risky to use any form of penicillin (Answer A and Answer B are incorrect). Clindamycin has good coverage for the possible infecting organisms (S. pneumoniae or Staphylococcus pyogenes) and would be an appropriate agent only if an antibiotic were warranted (Answer C is incorrect).)

A 24-year-old woman with a single, painless chancre on the genitalia is 3 months pregnant. She admits having multiple sex partners, with her last unprotected sex encounter 60 days before presentation. She has no history of sexually transmitted diseases. Allergies include penicillin (hives) and sulfonamides (rash). A rapid plasma reagin (RPR) test is positive. The confirmatory test (fluorescent treponemal antibody-absorption test) is also positive. Which one of the following would be the best treatment plan for this patient? A. Initiate therapy with doxycycline 100 mg orally two times/day for 14 days. B. Initiate therapy with ceftriaxone I g/day intravenously for 10 days. C. Treat with azithromycin I g orally as a single dose plus ceftriaxone 125 mg intramuscularly in a single dose. D. Desensitize and treat with benzathine penicillin G 2.4 million units intramuscularly in a single dose.

Answer D: Desensitize and treat with benzathine penicillin G 2.4 million units intramuscularly in a single dose. (According to the positive RPR and positive confirmatory test, the patient can be given a diagnosis of syphilis. Penicillin is the drug of choice for the medical management of all stages of syphilis. In pregnant patients, a proven alternative to penicillin for the treatment of syphilis does not currently exist; therefore, desensitization and treatment with benzathine penicillin is recommended (Answer D is correct). Doxycycline is contraindicated in this patient because she is pregnant (Answer A is incorrect). Ceftriaxone may be an effective alternative for the treatment of early syphilis in nonpregnant women; however, the potential for cross-reactivity with penicillin and lack of data to support an optimal dose or therapy duration are disadvantages (Answer B is incorrect). Azithromycin plus ceftriaxone is a treatment option for a patient coinfected with Chlamydia trachomatis and Neisseria gonorrhoeae; it is not an appropriate regimen for this patient (Answer C is incorrect).)

A 25-year-old woman who previously received a diagnosis of mild persistent asthma reports to her primary care physician's office for her 6-week follow-up. She states that she still has symptoms several times per week that require the use of her albuterol inhaler, as well as one or two nighttime awakenings per month. Her current drug regimen includes fluticasone 110 mcg 1 puff by mouth twice daily. Which one of the following would be the most appropriate change to her asthma medication regimen? A. Increase fluticasone to 220 mcg 2 puffs by mouth twice daily. B. Discontinue fluticasone and initiate mometasone 110 mcg 1 puff by mouth twice daily. C. Increase fluticasone to 220 mcg I puffby mouth twice daily plus montelukast 10 mg by mouth daily. D. Discontinue fluticasone and initiate fluticasone'salmeterol 100/50 1 puff by mouth twice daily.

Answer D: Discontinue fluticasone and initiate fluticasone/salmeterol 100/50 1 puff by mouth twice daily. (This patient has mild asthma (step 2) that is not well controlled. Therefore, she needs step-up therapy to step 3. The preferred treatment for step 3 therapy is a low-dose ICS plus a LABA or a medium-dose ICS alone (Answer D is correct). Fluticasone 220 mcg 2 puffs twice daily is a high-dose ICS, which is appropriate for step 5 and 6 therapy (Answer A is incorrect). Mometasone 110 mcg 1 puff twice daily is a low-dose ICS regimin, appropriate for step 2 therapy; it does not represent a step-up in therapy (Answer B is incorrect). An increase to fluticasone 220 mcg 1 puff twice daily plus montelukast is a medium-dose ICS plus a leukotriene and is consistent with step 4 therapy (Answer C is incorrect).)

A 78-year-old man has a medical history of hypertension, type 2 diabetes mellitus, transient ischemic attacks, and CKD. His current drugs include metformin 1000 mg two times/day, aspirin 81 mg/day, controlled-release diltiazem 180 mg/day, and hydrochlorothiazide 25 mg/day. He is scheduled to undergo an elective carotid endarterectomy. The patient's baseline BUN is 18 mg/dL and SCr is 1.5 mg/dL. Which one of the following is the best strategy for management of this patient? A. Discontinue aspirin and begin intravenous fluid hydration and oral N-acetylcysteine (NAC). B. Discontinue diltiazem and metformin and begin oral NAC. C. Discontinue hydrochlorothiazide and aspirin and begin ascorbic acid. D. Discontinue hydrochlorothiazide and metformin and begin intravenous fluid hydration.

Answer D: Discontinue hydrochlorothiazide and metformin and begin intravenous fluid hydration. (This patient is at risk of contrast-induced nephropathy (CIN) because of his advanced age, kidney disease, and diuretic use. Adequate hydration is the best strategy for preventing CIN, so his diuretic therapy should be discontinued and intravenous hydration administered (Answer D is correct). Using NAC to prevent CIN remains controversial, but it is acceptable to administer it in addition to intravenous fluids (Answer A and Answer B are incorrect). Ascorbic acid would not be an unreasonable agent to help reduce his risk of CIN; however, it has not been as widely studied as oral NAC for this purpose (Answer C is incorrect). Metformin will not increase this patient's risk of CIN, but it should be discontinued because of the risk of lactic acidosis, should the patient develop AKI from the contrast.)

An equipment coach for a local high school football team develops significant furunculosis in the groin and genital area and presents to the ED in severe pain and discomfort. Incision and drainage is performed on six cutaneous abscesses, and cultures are sent. The genital involvement is a grade 2 panniculus. The patient's medical history includes hypertension, type 2 diabetes mellitus, and chronic kidney disease. His vital signs include BP 150/90 mm Hg, HR 89 beats/minute, and temperature 1000F (37.80C). His WBC is 11 x 103 cells/mm3, and his body mass index is 31 kg/m2. The patient is allergic to sulfa drugs (rash) and morphine (dizziness). Culture and susceptibilities are pending, but community-acquired MRSA is suspected. The physician dresses the wound and prescribes mupirocin 2% topical ointment to apply three times/day for 7 days after wound cleaning and dressing change. Which one of the following is the most appropriate treatment for this patient? A. Continue mupirocin topical wound treatment as directed. B. Change therapy to topical chlorhexidine. C. Continue mupirocin topically and add oral doxycycline. D. Discontinue mupirocin and start oral doxycycline.

Answer D: Discontinue mupirocin and start oral doxycycline. (Doxycycline is the most appropriate therapy because of the difficulty of adequate incision and drainage, the patient's medical history of type 2 diabetes mellitus, and the multiple infection sites involving the groin and genital area (Answer D is correct). Appropriate wound treatment requires an assessment of the degree of tissue damage and subsequent host inflammatory response. Topical antimicrobial therapy may be appropriate for minor skin infections or secondary infected skin lesions but would not be appropriate for this patient (Answer A is incorrect). Chlorhexidine is used only with body decolonization regimens, not for wound treatment (Answer B is incorrect). No clinical evidence supports the efficacy of the combination of topical and systemic antimicrobial wound treatment compared with the use of a single systemic agent with standard wound care for several abscesses secondary to furunculosis (Answer C is incorrect).)

After undergoing routine mammography, a 52-year-old woman (weight 67 kg [148 lb]) receives a diagnosis of stage II adenocarcinoma of the left breast. She has had a breast biopsy, lumpectomy with sentinel lymph node biopsy, and subsequent axillary node dissection. Her tumor was estrogen-receptor positive and human epithelial receptor-2 positive, and she is scheduled to receive adjuvant chemotherapy with myelosuppressive agents, followed by 5 years of tamoxifen. During chemotherapy, a laboratory examination reveals Hgb 10.6 g/dL. Which one of the following would best manage this patient's treatment-related anemia? A. Epoetin alfa 40,000 units subcutaneously weekly during her chemotherapy. B. Darbepoetin 150 mcg subcutaneously weekly during her chemotherapy. C. Initiate therapy with an Erythropoietin stimulating agent (ESA) only after the HgB falls below 10 g/dL. D. ESA therapy is contraindicated in this patient.

Answer D: ESA therapy is contraindicated in this patient. (This patient has potentially curable stage II breast cancer and should not receive any ESAs, according to the American Society of Hematology/American Society of Clinical Oncology guidelines, because of the potential of stimulating erythropoietin receptors on the tumor cells (Answer D is correct). The concern is that the use of ESAs in potentially curable tumors may decrease the effectiveness of adjuvant chemotherapy; therefore, ESAs should be avoided. Initiating ESA therapy would be appropriate for a patient with metastatic breast cancer whose Hgb less than 10 g/dL and is receiving chemotherapy; however, it is not appropriate for a patient with early-stage disease (Answer A, Answer B, and Answer C are incorrect).)

The director of the university ambulatory care center asks the clinical pharmacist to conduct a survey to identify pharmacy services that are provided at similar clinics nationally. She needs the results in 3 weeks to present to the hospital administrator at budget hearings. She gives the pharmacist no extra money to conduct the study, but the pharmacist can use university resources (e.g., computers, telephones, envelopes) to conduct the study. The director provides the pharmacist with a list of ambulatory clinics that includes the clinic name, the clinic director's name, and the director's e-mail address. Which type of data collection would be the best in this situation? A. Mail survey. B. Computer-aided telephone interview survey. C. Face-to-face interview survey. D. Electronic survey.

Answer D: Electronic survey. (Electronic survey is the best approach given the limited budget and need for results quickly (Answer D is correct). Because a mail survey would take several months, it is not appropriate (Answer A is incorrect). A face-to-face interview is not feasible because the potential respondents are spread throughout the country, and there is no money or time available for travel (Answer C is incorrect). A computer-aided telephone interview survey would be the next-best option to an electronic survey, assuming the pharmacist has time to program the survey into the computer system, call the potential respondents, recall participants, and set up interview times. However, the pharmacist's contact list has only the name and e-mail address, not the clinic director's telephone numbers, which would need to be determined (Answer B is incorrect).)

A 78-year-old woman residing in a long-term care facility was stable until 2 weeks ago, when she experienced an increase in confusion that necessitated a psychiatric evaluation. The patient has a history of HTN, T2DM, mild cognitive impairment associated with AD, and recent-onset urge urinary incontinence. Her current drugs include glipizide 10 mg/day, metformin 500 mg three times/day, lisinopril 10 mg/day, aspirin 81 mg/day, donepezil 10 mg/day, and tolterodine 2 mg twice daily. A urinalysis was negative for leukocyte esterase and nitrite. Which one of the following is best to recommend for this patient? A. Start empiric therapy with sulfamethoxazole/trimethoprim. B. Increase the dose of donepezil to 20 mg/day. C. Add memantine 5 mg/day. D. Evaluate tolterodine use.

Answer D: Evaluate tolterodine use. (Confusion in the elderly can be a symptom of many different medical conditions. Because the urinalysis is negative for leukocyte esterase and nitrite, urinary tract infection is not likely the cause, and empiric therapy is not warranted (Answer A is incorrect). Increasing the donepezil dose to 20 mg exceeds the recommended dosage for mild to moderate AD (Answer B is incorrect). Disease- and drug-related causes for changes in mental status should be ruled out before initiating any additional therapy (Answer C is incorrect). Confusion may be related to the use of anticholinergic drugs; therefore, the patient's use of tolterodine should be evaluated (Answer D is correct).)

A 64-year-old white man presents to the clinic for a new patient visit. He received a diagnosis of COPD 8 years ago and has a 30 pack-year history of smoking; he quit cigarettes 6 years ago but continues to have a productive cough on most days. His medical history includes coronary artery disease, hypertension, hypercholesterolemia and significant hearing loss requiring a hearing aid. Post-bronchodilator spirometry shows an FEV(1)/FVC of 0.64 and an FEV(1) of 1.3 L (48% of predicted). He reports experiencing SOB when he walks to his mailbox (about 100 ft), but he does not have to stop because of his SOB. His home drugs are nebulized albuterol/ipratropium (usually two or three times daily), fluticasone/salmeterol DPI 250/50 mcg twice daily, metoprolol 25 mg twice daily, aspirin 81 mg daily, atorvastatin 40 mg daily, and nitroglycerin as needed. In the past year, he had one COPD flare, for which his physician prescribed an oral prednisone burst and a fluoroquinolone. The patient also was admitted to the hospital for an acute exacerbation of COPD within the past year, which led him to change health care providers. The patient has no known drug allergies. His COPD Assessment Test score is 15 and Modified Medical Research Council (mMRC) Dyspnea Scale score is 2. To which one of the following Global Initiative for Chronic Obstructive pulmonary Disease (GOLD) patient groups would it be best to assign this patient? A. Group A. B. Group B. C. Group C. D. Group D.

Answer D: Group D. (Using the information fouml in figure 2.3 Of GOLD guidelines, this patient meets the criteria for GOLD patient group D (based on high risk and more symptoms). This classification is based on thus patient's risks (GOLD class Ill with an FEV(1) 48% of predicted, 2 exacerbations in the past year, and symptoms (mMRC Dyspnea Scale grade 2 and COPD Assessment Test (CAT) score greater than 101 Therefore. the patient is at high risk, with more symptoms (Answer D is correct; Answer A, Answer B. and Answer C are incorrect).)

A 66-year-old man presents with hypertension, type 1 diabetes mellitus, and end-stage kidney disease. He receives hemodialysis three times/week and takes enalapril 5 mg daily, aspirin 81 mg daily, insulin glargine 12 units daily, calcium carbonate 500 mg three times/day, and sevelamer 400 mg three times/day. He does not have a documented immunization history, and he cannot recall which vaccines, if any, he has received in the past 10 years. Which one of the following vaccination plans is best for this patient? A. Herpes zoster (HZ), diphtheria and reduced tetanus toxoids and acellular pertussis (Tdap). B. HZ, pneumococcal polysaccharide 23-valent vaccine (PPSV23). C. HZ, Tdap, PPSV23. D. HZ, Tdap, PPSV23, hepatitis B (HBV) vaccine.

Answer D: HZ, Tdap, PPSV23, HBV vaccine. (This patient requires all four vaccinations at this time (Answer D is correct). According to the Advisory Committee on Immunization Practices (ACIP) guidelines, an HZ vaccination is FDA approved for individuals 50 years or older; however, the ACIP recommends HZ vaccine for individuals 60 years or older, regardless of previous HZ status. A one-time Tdap booster is indicated for all individuals without documentation of a previous dose. A dose of PPSV23 vaccine should be administered to all individuals older than 65 and at least 5 years after the previous dose (the patient has no record of a previous pneumococcal vaccination, and when in doubt, vaccinate). In addition, the HBV vaccine is indicated because this patient is on hemodialysis, which puts him at high risk of the disease (Answer A, Answer B, and Answer C are incorrect).)

A representative from the National Patient Safety Foundation is conducting a personal interview survey of pharmacists to understand more about medication safety. Which one of the following survey questions would be most likely to illicit response bias? A. How do pharmacists feel when they realize a prescription has been filled incorrectly? B. What steps are taken in pharmacies when a medication error occurs? C. Are you familiar with the National Patient Safety Foundation? D. Have you ever filled a prescription incorrectly that harmed a patient?

Answer D: Have you ever filled a prescription incorrectly that harmed a patient? (The question of "Have you ever filled a prescription incorrectly that harmed a patient?" may threaten the pharmacist, causing many pharmacists to answer "No." A better way to word this question is to depersonalize the issue and therefore reduce the response bias. An example of this would be, "Of 100 prescriptions, how many do you believe are filled incorrectly by pharmacists?" The questions "How do pharmacists feel when they realize a prescription has been filled incorrectly?" and "What steps are taken in pharmacies when a medication error occurs?" also depersonalize the issue and therefore reduce response bias (Answer A and Answer B are incorrect). In addition, the question "Are you familiar with the National Patient Safety Foundation?" most likely does not threaten the respondent; however, acquiescence may be a minor issue here because some respondents may feel compelled to answer yes, but it is not of primary concern (Answer C is incorrect).)

In preparing a research poster for a national pharmacy meeting, you and a resident analyze data from a pilot project of systolic blood pressure and oral contraceptive use in 16 women. The systolic blood pressure (SBP) readings for each person before and after starting oral contraceptive (OC) use are shown below. It is assumed that systolic blood pressure is normally distributed at baseline. ------------------------------------------------- Patient No. || SBP w/o OC || SBP w/ OC 1 || 120 || 133 2 || 117 || 120 3 || 112 || 111 4 || 124 || 133 5 || 120 || 127 6 || 143 || 150 7 || 131 || 137 8 || 110 || 114 9 || 109 || 107 10 || 124 || 126 11 || 110 || 123 12 || 99 || 102 13 || 100 || 99 14 || 121 || 130 15 || 133 || 140 16 || 109 || 116 ------------------------------------------------- The SD of the mean difference in systolic blood pressure is 4.6. Which one of the following is the best interpretation of the SEM difference in systolic blood pressure? A. The average distance of each value in this data set from its expected value is 5.29. B. The average distance of each value in this data set from its expected value is 1.32. C. In general, the sample mean differs from a set of sample means by 0.29. D. In general, the sample mean differs from a set of sample means by 1.15.

Answer D: In general, the sample mean differs from a set of sample means by 1.15. (The SEM is calculated by dividing the SD (4.6) by the square root of the number of study participants; for n=16, the √16 4 (Answer D is correct). Dividing the square of the SD by the square root of the number of study participants yields a value of 5.29 (Answer A is incorrect). Dividing the square of the SD by the number of study participants yields a value of 1.32 (Answer B is incorrect). Dividing the SD by the number of study participants yields a value of 0.29 (Answer C is incorrect).)

In a study of the number of adverse drug events observed in 10,000 people over a year, the authors report 50 events (that is, 5 events per 1000 annually). Which one of the following epidemiologic measures does this estimate represent? A. Relative risk. B. Hazard ratio. C. Prevalence. D. Incidence rate.

Answer D: Incidence rate. (The incidence rate is a measure of the number of study subjects with the outcome of interest divided by the total time contributed by the individuals followed (Answer D is correct). The relative risk is the number of individuals in an exposed group who experience an outcome during a period divided by the number of individuals in an unexposed group who experience an outcome (Answer A is incorrect). The hazard ratio is the instantaneous probability of experiencing the outcome assuming the person is at risk of the event up until that point (Answer B is incorrect). Prevalence is a measure of outcome status that shows how many outcomes of interest have occurred (Answer C is incorrect).)

A 7-year-old girl (height 3'1", weight 16 kg [35 lb]) has a medical history of stage 5 vesicoureteral reflux and repeated episodes of pyelonephritis. She is admitted to the hospital for placement of a central venous catheter for intravenous antibiotic administration, although she has no current infection. Her vital signs on admission are as follows: temperature 37.1^oC (98.7^oF), heart rate 95 beats/minute, blood pressure 115/75 mm Hg (90th percentile), and respiratory rate 20 breaths/minute. Laboratory values are as follows: Na 137 mEq/L, Cl 114 mEq/L, K 4.8 mEq/L, HC03 22 mEq/L, BUN 27 mg/dL, scr 2.5 mg/dL, serum glucose 95 mg/dL, AIC 10 g/dL, Hct 29.6%, Hgb 10g/dL, ferritin 70 ng/mL, phosphorus 9.2 mg/dL, calcium 6 mg/dL, and parathyroid hormone 30.1 pg/mL. The patient's home drugs include sulfamethoxazole 400 mg/trimethoprim 80 mg once daily, erythropoietin alpha 800 units subcutaneously three times/week, and captopril 6.25 mg twice daily. Which one of the following is best to manage this patient's anemia? A. Maintain the erythropoietin dosage; start oral iron therapy (elemental iron 3 mg/kg/day). B. Increase the erythropoietin dosage to 1200 units subcutaneously three times/week. C. Increase the erythropoietin dosage to 1600 units subcutaneously three times/week and start oral iron therapy (elemental iron 3 mg/kg/day). D. Increase the erythropoietin dose to 1200 units subcutaneously three times/week and start oral iron therapy (elemental iron 3 mg/kg/day).

Answer D: Increase the erythropoietin dose to 1200 units subcutaneously three times/week and start oral iron therapy (elemental iron 3 mg/kg/day) daily. (In children with chronic kidney disease treated with erythropoietin, the target Hgb concentration is 12 g/dL. Because this patient's Hgb concentration is 10 g/dL, increasing the erythropoietin dose by about 25% to 1200 units is warranted (Answer A is incorrect). Because the ferritin concentration is relatively low, additional iron will be needed for optimal response to erythropoietin (Answer D is correct; Answer B is incorrect). However, increasing the erythropoietin dose to 1600 mg (doubling the dose) is excessive given the patient's Hgb concentration (Answer C is incorrect).)

When enforcing the Prescription Drug Marketing Act, the FDA states that it will focus its regulatory efforts on areas in which there is a strong likelihood of counterfeiting. Which one of the following scenarios is most likely to be a primary concern for the FDA? A. Veterinary drug for distemper in short supply. B. Newly marketed angiotensin-converting enzyme inhibitor. C. Patient individually reimporting drugs from Canada. D. Influenza vaccine in short supply.

Answer D: Influenza vaccine in short supply. (The FDA will focus its initial efforts of enforcing drugs that are at a high risk for counterfeiting. A shortage of influenza vaccine is an example of a prime target for counterfeiting (Answer D is correct). Veterinary products are not covered under the Prescription Drug Marketing Act (Answer A is incorrect). The FDA has stated that it will exercise selective enforcement authority and will not pursue actions against individuals importing drugs for their own use at this time (Answer C is incorrect). New prescription drugs are at a higher risk for counterfeiting, but enforcement will depend on the product's marketplace influence and the ease of detecting a counterfeit product by the patient and the medical community. A newly marketed angiotensin-converting enzyme inhibitor will have less marketplace influence, and the effects of a counterfeit product are easily observed (Answer B is incorrect).)

A 17-year-old African American male adolescent presents with comedonal and inflammatory acne. He has about 25 comedones and 10 inflammatory papules on his forehead, nose, and cheeks. He has used benzoyl peroxide 3% cleanser twice daily for 6 weeks with minimal results. In addition to his acne, he is concerned about the dark "scars" present as the lesions heal. He notices that, although the acne is worse during baseball season, it is present throughout the year. He is counseled on proper skin care, including using a gentle cleanser twice daily, avoiding scrubbing his skin, and showering after athletic activity. In addition to using an SPF 30 broad-spectrum sunscreen, which one of the following is the most appropriate pharmacologic regimen for this patient? A. Continue benzoyl peroxide 3% cleanser for another 6 weeks. B. Initiate azelaic acid 15% gel twice daily. C. Initiate dapsone 5% gel daily. D. Initiate adapalene 0.3% gel at bedtime.

Answer D: Initiate adapalene 0.3% gel at bedtime. (Early and aggressive treatment of acne is recommended for dark-skinned individuals because of increased risk of keloid formation and postinflammatory hyperpigmentation (PIH). Topical retinoids improve PIH and prevent comedone formation, so adapalene 0.3% gel is a good initial therapy for this patient (Answer D is correct). Studies have shown that adapalene 0.1% gel decreases the density of hyperpigmented macules in 65% of patients. Benzoyl peroxide wash may be used as adjunctive therapy, but it will not have as great an impact on comedone formation or PIH in this patient (Answer A is incorrect). Azelaic acid may be used in patients who do not tolerate traditional topical retinoids (Answer B is incorrect). Dapsone gel may decrease the number of inflammatory lesions and can be used in conjunction with a topical retinoid; however, it will not directly help PIH (Answer C is incorrect).)

A 44-year-old woman receives a diagnosis of a DVT in the ED. Her medical history is significant for a ruptured Achilles tendon, which was surgically repaired on an outpatient basis 10 days ago. The ED nurse practitioner wants to treat the patient with subcutaneous enoxaparin and oral rivaroxaban. Which one of the following is best to recommend regarding the use of rivaroxaban in this patient? A. Initiate at the same time as enoxaparin. B. Initiate 24 hours before enoxaparin is discontinued. C. Initiate 24 hours after enoxaparin is discontinued. D. Initiate immediately after DVT diagnosis, with no enoxaparin needed.

Answer D: Initiate immediately after DVT diagnosis, with no enoxaparin needed. (Unlike warfarin, which requires a minimum of 5—7 days of dosing to achieve a therapeutic INR, orally administered rivaroxaban immediately produces an anticoagulant effect. Bridging with an initial injectable anticoagulant is unnecessary, as confirmed in clinical trials using this agent for the treatment of VT E. In this patient, rivaroxaban can be given immediately (Answer D is correct), without the need for initial enoxaparin use (Answer A, Answer B, and Answer C are incorrect).)

A 57-year-old woman with asthma presents to the emergency department with a fever and shortness of breath. She is given a diagnosis of community-acquired pneumonia and is initially admitted to the medical intensive care unit for potential intubation. When her oxygenation improves, she is moved to the standard medical floor. The patient's home drugs include an albuterol rescue inhaler and fluticasone 88 mcg 2 puffs two times/day. The emergency department pharmacist reconciles her drug list. After a 5-day hospital stay, she is given standard discharge counseling and sent home with oral doxycycline to complete a 10-day course of antibiotics. She is instructed to follow up with her primary care physician in 1—2 weeks. During which one of the following care transitions is an ADE most likely to occur in this patient? A. Home to emergency department. B. Emergency department to inpatient hospital stay. C. Medical intensive care unit to medical ward. D. Inpatient hospital stay to home.

Answer D: Inpatient hospital stay to home. (Antibiotics (e.g., doxycycline) are commonly implicated in medication-related errors and ADEs. This patient also did not receive pharmacist or other health care provider follow-up postdischarge, even though this has been shown to decrease ADEs in the postdischarge period. Although ADEs can occur during other transition periods, no data exist to support the premise that medication errors occur during these transitions, especially compared with the hospital to home transition (Answer A, Answer B, and Answer C are incorrect). Data have shown that when patients are discharged from hospital to home, two-thirds of the adverse events experienced are medication related (Answer D is correct).)

A 76-year-old woman has bilateral knee OA pain that has not been sufficiently controlled with physical therapy, simple analgesics, systemic NSAIDs, or a short trial of opioid combination analgesics. Today, she presents to the clinic for follow-up. She is unable to perform many activities of daily living because she requires a walker, which considerably impairs her mobility. Which one of the following is the best long-term therapy option to address this patient's chronic pain? A. Glucosamine 1500 mg and chondroitin 1200 mg/day. B. Ketorolac 10 mg every 6 hours. C. Morphine sulfate extended release 15 mg twice daily. D. Intra-articular corticosteroid injection up to every 3 months as needed.

Answer D: Intra-articular corticosteroid injection up to every 3 months as needed. (Intra-articular corticosteroids are the best choice to help alleviate this patient's chronic pain secondary to her OA of the knee. Although the medication may take several days to reach maximal efficacy, it will offer her the most long-term pain relief with fewer systemic adverse reactions (Answer D is correct). According to the American College of Rheumatology guidelines for the treatment of OA, glucosamine and chondroitin are not recommended for patients with OA of the knee (Answer A is incorrect). Ketorolac will provide the most significant pain relief in the least amount of time, but use beyond 5 days increases the risk of gastrointestinal bleeding: thus, ketorolac is not a long-term option for this patient (Answer B is incorrect). Finally, the use of opioids for OA pain (and in the elderly) should discouraged of the increased of somnolence and unsteady gait (Answer C is incorrect).)

A 74-year-old woman, currently residing in a long-term care facility, lost her husband to cancer 6 months ago. Since then, she has lost 6.8 kg (15 lb), and her daughter is extremely concerned about her mother's mood and nutritional status. During a care plan meeting, the team determines that the patient's depressive symptoms are not subsiding and decides to initiate pharmacotherapy. Which of the following is best to recommend for this patient? A. Sertraline. B. Paroxetine. C. Duloxetine. D. Mirtazapine.

Answer D: Mirtazapine. (Although effective at treating depression, selective-serotonin reuptake inhibitors (SSRIs) and SNRIs have not been associated with significant weight gain in older patients in need of weight gain. Of the antidepressants listed, sertraline and paroxetine are SSRIs (Answer A and Answer B are incorrect), and duloxetine is a an SNRI (Answer C is incorrect). Mirtazapine, a tetracyclic antidepressant, has been associated with improvements in appetite and weight gain in older individuals and is the best initial agent for this patient (Answer D is correct).)

A 69-year-old man with Parkinson disease (PD) and neurogenic overactive bladder presents to the primary care clinic with complaints of frequent urination and urge incontinence. His current drugs include amantadine 100 mg three times/day, carbidopa/levodopa 25/100 mg four times/day, rasagiline I mg/day, and citalopram 10 mg/day. Medical notes show that amitriptyline, desipramine, darifenacin, and oxybutynin have been initiated to treat the patient's overactive bladder in the past, but these agents were poorly tolerated because of treatment-emergent confusion. His PSA (prostate-specific antigen) level was within normal age limits. Which one of the following would best manage this patient's overactive bladder? A. A trial of prazosin. B. A trial oftamsulosin. C. A trial of tolterodine. D. Intra-detrusor botulinum toxin injection.

Answer D: Intra-detrusor botulinum toxin injection. (Intra-detrusor injections of onabotulinumtoxin A are effective and well tolerated for overactive bladder (Answer D is correct). Prazosin and tamsulosin may not be effective for symptoms of overactive bladder, particularly in men without benign prostatic hyperplasia (Answer A and Answer B are incorrect). Given this patient's history of intolerance to anticholinergic agents, tolterodine is not an appropriate recommendation (Answer C is incorrect).)

A 55-year-old man has a medical history of acute prostatitis. He has had two episodes within the past year; both were treated with trimethoprim/sulfamethoxazole. Now, his symptoms have returned. His physician is considering therapy options for chronic prostatitis. Which one of the following is best to recommend for this patient? A. Cephalexin. B. Azithromycin. C. Amoxicillin. D. Levofloxacin.

Answer D: Levofloxacin. (The fluoroquinolones are the preferred agents to treat prostatitis because of the high degree of distribution in the prostatic tissue (Answer D is correct). The other antibiotics are not associated with similar treatment outcomes (A. Answer B, and Answer C are incorrect).)

A 57-year-old man (height 5'9", weight 100 kg [220.5 lb]) is brought to the ED with progressive altered mental status. His family reports that he has had hallucinations and periods of lethargy alternating with agitation. He responds to verbal cues, knows his name, does not know his location, and denies headache. His medical history includes hypertension, hyperlipidemia, and a kidney transplant (2 years ago). His drugs include tacrolimus 2 mg orally two times/day, omeprazole 40 mg orally daily, lisinopril 5 mg orally daily, and ibuprofen 200 mg orally two times/day as needed for headache. Head CT shows atrophy. Vital signs in the ED are temperature 98.7^oF (370C), HR 92 beats/minute, RR 20 breaths/minute, and BP 122/85 mm Hg. Laboratory values are as follows: Na 139 mEq/L, K 2.9 mEq/L, Cl 104 mEq/L, BUN 21 mg/dL, SCr 1.8 mg/dL (baseline 2 months earlier 1.2 mg/dL), serum glucose 77 mg/dL, AST 36 IU/L, ALT 40 IU/L, WBC 4.4 x 103 cells/mm3, Hct 35%, and platelet count 224,000 cells/mm3. Two sets of blood cultures are pending. A CSF examination shows pale yellow fluid, WBC 1.9 x 10^3 cells/mm^3, lymphocytes 81%, monocytes 12%, eosinophils 7%, protein 345 mg/dL, glucose 22 mg/dL, and RBC 208 cells. The CSF Gram stain shows few WBCs and no bacteria. Results of polymerase chain reaction are negative for enterovirus and herpes simplex virus (HSV)-l/HSV-2, and the CSF cryptococcal antigen is positive. Which one of the following is best for this patient? A. Amphotericin deoxycholate 70 mg intravenously every 24 hours plus flucytosine 2.5 g orally every 6 hours. B. Amphotericin deoxycholate 600 mg intravenously every 24 hours. C. Liposomal amphotericin B 300 mg intravenously every 24 hours. D. Liposomal amphotericin B 300 mg intravenously every 24 hours plus flucytosine 2.5 g orally every 12 hours.

Answer D: Liposomal amphotericin B 300 mg intravenously every 24 hours plus flucytosine 2.5 g orally every 12 hours. (This patient has had a kidney transplant and has a diagnosis of cryptococcal meningitis. His creatinine is increased from baseline. The preferred induction therapy is a lipid formulation of amphotericin with flucytosine (Answer D is correct). In patients who are unable to tolerate the hematologic toxicities of flucytosine, lipid formulations of amphotericin B may be given as monotherapy for an extended period as an alternative induction regimen (Answer C is incorrect). In addition, induction regimens without flucytosine are associated with higher rates of early mycologic failure. The use of amphotericin deoxycholate has a higher incidence of nephrotoxicity, and the lipid formulations are preferred in solid organ transplant recipients (Answer A and Answer B are incorrect).)

A 36-year-old woman at 32 weeks' gestation is brought to the emergency department after her husband witnessed a seizure lasting 1—2 minutes. The patient has abdominal pain and headache. Her blood pressure on admission is 180/100 mm Hg with a heart rate of 90 beats/minute. Results of a urinalysis are as follows: 3+ protein, negative nitrite, and no WBCs. A chemistry panel and complete blood cell count are within normal limits. Fetal monitoring shows no significant fetal distress. Which one of the following would be the best therapy to initiate for this patient? A. Methyldopa. B. Nifedipine. C. Atenolol. D. Magnesium sulfate.

Answer D: Magnesium sulfate. (This patient is suffering from eclampsia (hypertension, proteinuria, and seizure). Magnesium sulfate is the treatment of choice for eclampsia (Answer D is correct). Methyldopa can be used to treat preeclampsia (Answer A is incorrect). Nifedipine and atenolol can be used for chronic hypertension during pregnancy, but atenolol has been associated with intrauterine growth retardation (Answer B and Answer C are incorrect).)

A 57-year-old postmenopausal woman received a diagnosis of HIV in 1994 and now has an extensive history of antiretroviral therapy. She has been clinically stable during the past 2 years on her current regimen (tenofovir, lamivudine, and lopinavir/ritonavir); her current CD4+ cell count is 280 cells/mm3; and her viral load is undetectable. Her laboratory tests show TC 320 mg/dL, HDL-C 35 mg/dL, and TG 450 mg/dL. The patient's family history is significant for a father who died of a myocardial infarction (MI) at age 50 and two brothers with a previous MI at age 48 and 52, respectively. Which one of the following is best to recommend for this patient's dyslipidemia? A. Switch the antiretroviral regimen to tenofovir, lamivudine, and atazanavir/ritonavir. B. Switch lopinavir/ritonavir to darunavir/ritonavir; add lipid-lowering therapy with pravastatin 40 mg daily. C. Maintain the current antiretroviral regimen; recommend diet, exercise, and lifestyle modification. D. Maintain the current antiretroviral regimen; add lipid-lowering therapy with rosuvastatin 10 mg daily.

Answer D: Maintain the antiretroviral regimen; add lipid-lowering therapy with rosuvastatin 10 mg daily. (This patient has dyslipidemia that requires intervention. The elevations in TC and TG observed in this patient are likely the result of several factors, including the presence of lopinavir and ritonavir. In this case, the patient has extensive antiretroviral experience and is currently clinically stable on her present regimen. Given her history, it would be difficult to change agents and ensure virologic efficacy (Answer A and Answer B are incorrect). In addition, the drug-drug interaction between pravastatin and darunavir should be avoided (Answer B is incorrect). Because of degree of dyslipidemia present in this patient, diet, exercise, and lifestyle modification alone would not affect her lipid profile to the desired extent (Answer C is incorrect). As a result, the best option for this patient is to maintain her current antiretroviral regimen while starting lipid-lowering therapy. A recent randomized trial showed that rosuvastatin 10 mg/day was more effective than pravastatin 40 mg/day in lowering LDL-C and TG concentrations in patients with HIV infection who were treated with ritonavir-boosted protease inhibitors (Answer D is correct).)

A 32-year-old woman (weight 50 kg [110 lb]) was recently given a diagnosis of secondary hypothyroid disease caused by idiopathic pan-hypopituitarism. She began treatment with levothyroxine 75 mcg orally once daily 2 weeks ago. The results of today's thyroid function tests are total thyroxine (T4) 4 mcg/dL, total triiodothyronine (T3) (75 ng/dL), and TSH less than 0.1 mlU/mL. Given these results, which one of the following is best to recommend regarding this patient's levothyroxine therapy? A. Increase dose because T3 and T4 levels are low. B. Decrease dose because TSH levels are low. C. Make no change in therapy because free T 4 levels are not available. D. Make no change in therapy because she has not yet reached steady state at this dose.

Answer D: Make no change in therapy because she has not yet reached steady state at this dose. (The half-life ofT4 in a euthyroid patient is about I week. This patient has been on therapy for 2 weeks, or about 2 half-lives. Steady state is reached in about 5 half-lives (5 weeks). Levels of T 4 and T3 should be repeated after 3-4 additional weeks of therapy, with goal levels in the upper limits of the normal ranges. Adjusting the dose based on the T3 and T4 values at this time is premature (Answer A is incorrect). A free T 4 level is not required and would again be drawn too soon after the initial dose of levothyroxine (Answer C is incorrect). Because this patient has secondary hypothyroidism, her TSH level will always be low; this level does not represent over-replacement, as would be the case in primary hypothyroidism (Answer B is incorrect).)

A 24-year-old man with a long-standing history of drug use (cocaine, alcohol) is admitted to the ED with a suspected cocaine overdose. His BP is 210/115 mm Hg. His laboratory values are within normal limits except for SCr 2.4 mg/dL and K 5.2 mEq/L. The patient's cardiac enzymes are normal. In addition to a benzodiazepine, which one of the following agents would be the best recommendation for this patient? A. Sodium nitroprusside. B. Esmolol. C. Labetalol. D. Nicardipine.

Answer D: Nicardipine. (Nitroprusside is not the optimal agent because this patient's elevated SCr puts him at greater risk of cyanide toxicity (Answer A is incorrect). No acute HF is present; therefore, nicardipine would be the best agent to treat the hypertension in this patient because it is associated with fewer adverse effects than sodium nitroprusside (Answer D is correct). Pure ß-blockers (e.g., esmolol) are contraindicated in the setting of cocaine overdose because they can result in unopposed alpha activity and exacerbate coronary vasospasm (Answer B is incorrect). Although labetalol is sometimes used, the evidence is conflicting regarding its use for this indication. Current recommendations state that administration of combined u- and ß-blocking agents may be reasonable for hypertension management provided the patient has first received a vasodilator. Therefore, labetalol is not the best initial choice (Answer C is incorrect).)

An 89-year-old woman experiences frequent urination with a burning sensation. Her urine culture is positive for Escherichia coli sensitive to cephalexin, trimethoprim/sulfamethoxazole, ciprofloxacin, and nitrofurantoin. Which one of the following, if prescribed for this patient, most accurately represents use of a high-risk medication in the elderly? A. Cephalexin. B. Trimethoprim/sulfamethoxazole. C. Ciprofloxacin. D. Nitrofurantoin.

Answer D: Nitrofurantoin. (Cephalexin, trimethoprim/sulfamethoxazole, and ciprofloxacin are not on the Beers list and are not listed on the HEDIS measure as PIMs (potentially inappropriate medications) for the elderly (Answer A, Answer B, and Answer C are incorrect). Nitrofurantoin is a high-risk medication according to the Beers list and was included in the HEDIS measure "Use of High-Risk Medications in the Elderly" (or DAE [drugs to avoid in the elderly]). Patients with impaired kidney function, which is more common in the elderly, are at higher risk of adverse effects because nitrofurantoin is contraindicated when the creatinine clearance is less than 60 mL/minute (Answer D is correct). However, higher incidences of pulmonary and hepatic toxicity have been reported in elderly patients than in younger patients, both from acute and chronic use of the drug.)

A 3-year-old boy with no significant medical history comes to your clinic with symptoms of irritability, temperature of 37.9^oC (100.2^oF), and ear tugging. After pneumatoscopic examination, you notice a red, bulging left tympanic membrane. This presentation is his first episode of acute otitis media (AOM). Which therapy should be recommended for his AOM? A. Ceftriaxone 100 mg/kg/day. B. Ciprofloxacin 10 mg/kg/day. C. Amoxicillin 60 mg/kg/day. D. No antibiotic at this time; follow up in 48—72 hours if symptoms unimproved or worsened.

Answer D: No antibiotic at this time; follow up in 48—72 hours if symptoms unimproved or worsened. (No antibiotic is necessary at this time (Answer D is correct). According to the 2013 American Academy of Pediatrics AOM guidelines, if the patient is older than 2 years, does not have a temperature above 39^oC (102.2^oF), and has mild (not severe) ear pain, watchful waiting is an acceptable option, and antibiotics are not required. This patient's ear pain is considered nonsevere because his crying is not inconsolable, and he has some irritability. Ceftriaxone is recommended only after amoxicillin (Answer A is incorrect). Ciprofloxacin would not be recommended for AOM (Answer B is incorrect). Amoxicillin would be recommended after watchful waiting at a dose of 90 mg/kg/day. An amoxicillin dose of 60 mg/kg/day is lower than recommended (Answer C is incorrect).)

A 47-year-old man with a history of alcohol abuse presents to the ICU with severe abdominal pain. Laboratory evaluation shows he has elevated amylase and lipase concentrations. The patient is found to have severe acute pancreatitis with no necrosis. Which of the following is best to recommend regarding antimicrobial therapy for this patient? A. Amoxicillin/clavulanate. B. Ampicillin/sulbactam. C. Piperacillin/tazobactam. D. No antimicrobial therapy.

Answer D: No antimicrobial therapy. (According to current guidelines, the routine use of prophylactic antibiotics in patients with severe acute pancreatitis is not recommended. It is also not recommended to use antibiotics to prevent the development of infected necrosis in patients with sterile necrosis (Answer A, Answer B, and Answer C are incorrect). Because this patient has no apparent necrosis, no evidence exists to support the use of antibiotics (Answer D is correct). A diagnosis of infected necrosis should be considered in patients with pancreatic or extrapancreatic necrosis whose necrosis deteriorates or fails to improve after 7—10 days of hospitalization. In these patients, initial computed tomography—guided fine-needle aspiration for Gram stain and culture can guide the use of appropriate antibiotics.)

A 34-year-old woman presents to the clinic with her male partner, with whom she is sexually active. She is concerned about a cream-colored, malodorous vaginal discharge. A potassium hydroxide whiff test is positive. Her partner is asymptomatic. The physician writes a prescription for the patient and asks you to recommend an empiric treatment strategy for her partner. Which one of the following is best to recommend for the patient's partner? A. Metronidazole 500 mg orally two times/day for 7 days. B. Clindamycin mg orally two times/day for 7 days. C. Tinidazole 2 g orally as a single dose. D. No treatment warranted.

Answer D: No treatment warranted. (Based on the patient's symptoms (cream-colored, malodorous vaginal discharge and positive potassium hydroxide whiff test), her most likely diagnosis is bacterial vaginosis; therefore, routine treatment of her asymptomatic male partner is unwarranted (Answer D is correct). Metronidazole 500 mg or clindamycin 300 mg orally two times/day for 7 days are therapeutic options for the patient but are not necessary for treatment of her sexual partner (Answer A and Answer B are incorrect). Had she received a diagnosis of trichomoniasis, tinidazole 2 g orally as a single dose would be appropriate therapy for her sexual partner. In this case, however, bacterial vaginosis is the most likely cause of her symptoms; therefore, tinidazole is inappropriate (Answer C is incorrect).)

A 73-year-old White man (height 73 inches, weight 98 kg [216 lb]) with a history of hyperuricemia, recurrent gout, and HTN presents with intense pain, swelling, and redness of his right metatarsophalangeal joint. His current drugs include febuxostat 40 mg/day and lisinopril 40 mg/day. His most was 2.1 mg/dL He is admitted to the hospital for severe community-acquired pneumonia, where he begins treatment with antibiotics, including clarithromycin and ceftriaxone. He undergoes joint aspiration; the fluid is positive for monosodium urate crystals, and culture yields no growth. Which one of the following is best to recommend for this patient? A. Indomethacin. B. Colchicine. C. Intra-articular corticosteroid. D. Oral corticosteroid.

Answer D: Oral corticosteroid. (Nonsteroidal anti-inflammatory drugs (NSAIDs) such as indomethacin are not the best choice for this patient because his CrCl of 35 mL/minute indicates kidney function impairment (Answer A is incorrect). Colchicine is not the best option because, in combination with clarithromycin, it can cause rhabdomyolysis and death even with short-term therapy, in patients with kidney function impairment. Several case reports of fatalities and rhabdomyolysis caused by this severe reaction are reported in the literature, as well as in a retrospective review of 116 patients, concluding that combination therapy should be avoided in patients with kidney impairment (Answer B is incorrect.) The American College of Rheumatology guidelines support the of corticosteroids in patients unable to tolerate either NSAIDs or and in whom septic arthritis has ruled out. If one or two small joints are involved, the guidelines recommend oral corticosteroids. If one or two large joints are involved, intra-articular corticosteroids are recommended. Because this patient is pain in his metatarsophalangeal joint, oral corticosteroid therapy is the most appropriate treatment (Answer D is correct; Answer C is incorrect).)

A 32-year-old man presents to the clinic with a I-h€Air history of headache. He describes the pain as "an ice pick through my eye." The headache is unilateral. and he denies any symptoms of nausea or aura but notes nasal congestion. He reports that he has had four headaches of this type during the past 2 years, occurring in the spring and fall, and notes that his father has similar headaches as well. Which one of the following is the most effective abortive therapy for this patient's headache? A. Ergotamine tartrate. B. Lithium. C. Naratriptan. D. Oxygen.

Answer D: Oxygen. (This patient is experiencing a cluster headache based on several factors. One factor is his sex because cluster headaches are more common in men. In addition, cluster headaches often have a genetic component, and his father experiences the same type of headaches. Cluster headaches are shorter in duration and present with more excruciating pain than migraine headaches. The clinical factors that are consistent in this case are the patient's symptoms of "ice pick" pain sensation through his eye, unilateral location, and no nausea or aura experienced. The patient also describes his headaches as having a "cluster" pattern for the past 2 years in the spring and fall. Oxygen is the standard treatment for acute cluster headaches (Answer D is correct). Ergotamine tartrate and naratriptan have limited usefulness because of the time needed for absorption (Answer A and Answer B are incorrect). Lithium is used for prophylaxis of cluster headaches, but not abortive therapy (Answer C is incorrect).)

A 13-year-old boy (weight 50 kg) with sickle cell disease (SCD) is brought to the emergency department because of increasing lethargy and tiredness for the past 2 weeks. He states he can hardly get out of bed in the morning because he is so tired. He takes ibuprofen 400 mg orally every 8 hours as needed for pain and acetaminophen/hydrocodone 325 mg/ 7.5 mg orally every 6 hours as needed for pain. His oral temperature is 37.5^oC (99.5^oF). His oxygen saturation is 90% on room air by nasal cannula. His laboratory results show Hgb 5.4 g/dL, WBC 5 x 10^3 cells/mm^3, and Hgb S concentration 20%. He has been initiated on intravenous fluids. Which one of the following is best to recommend for this patient? A. Ceftriaxone 2.5 g given intravenously every 12 hours. B. Oxygen by nasal cannula to keep saturations greater than 95%. C. Hydroxyurea 500 mg orally once daily. D. Packed RBC transfusion.

Answer D: Packed RBC transfusion. (Transfusion with packed RBCs will have the most immediate effect on resolving this patient's symptoms because of his very low Hgb (5.4 g/dL), which is probably the main cause of his lethargy (Answer D is correct). Although oxygen administration will help, it is not a long-term solution, and he will need to have his Hgb increased to sustain better oxygen saturations (Answer B is incorrect). Hydroxyurea will help increase his concentration of Hgb F and subsequently decrease his Hgb S concentration, but it may take up to 6 months or more to achieve full effect (Answer C is incorrect). Ceftriaxone would be an appropriate choice if he had signs and symptoms of infection; however, with a normal WBC and no fever, he does not appear to have an infection (Answer A is incorrect).)

A woman with T2DM is receiving metformin 1000 mg twice daily. Her AIC concentration today is 7.8%. The patient has a known history of hypertension (H TN), pancreatitis, dyslipidemia, and depression. She has low blood pressure and reports dizziness on a near-daily basis. Which one of the following would best facilitate glycemic control for this patient? A. Liraglutide 0.6 mg once daily. B. Canagliflozin 100 mg once daily. C. Linagliptin 5 mg once daily. D. Pioglitazone 15 mg once daily.

Answer D: Pioglitazone 15 mg once daily. (This patient has a mild to moderately elevated AIC but is experiencing bouts of hypotension. Case reports suggest that liraglutide and linagliptin increase the risk of pancreatitis. These agents should not be used in patients with a history of pancreatitis if other options are available (Answer A and Answer C are incorrect). Canagliflozin increases the potential for hypotension (Answer B is incorrect). Pioglitazone is the best choice for this patient because there are no contraindications for its use, and it could provide sufficient control of hyperglycemia in this patient to reach the goal AIC (Answer D is correct).)

A 68-year-old man (height 5'5", weight 90 kg [198.5 lbs]) has emergency coronary artery bypass graft surgery. Before surgery, he had a catheterization for non-ST-segment elevation myocardial infarction and received aspirin, clopidogrel load, and eptifibatide. Catheterization revealed an ejection fraction of 35% and a W/o occlusion of his left main coronary artery, requiring emergency open-heart surgery. On admission to the ICU after surgery, vital signs are blood pressure 78/42 mm Hg, pulse rate 98 beats/minute, respiratory rate 21 breaths/minute, and temperature 970 F (36.10 C). His chest tube output is bloody, not serosanguineous, and has averaged 300 mL/hour for the past 3 hours. Pertinent laboratory values are BUN -52 mg/dL, SCr 2.1 mg/dL, INR 1.3, PTT 33 seconds, fibrinogen 254 mg/dL, Hgb 7.9 g/dL, Hct 23.2%, and platelet count 90, 000/mm3. His only drugs postoperatively are cefazolin I g intravenously every 12 hours and pantoprazole 40 mg intravenously every 24 hours. Which of the following would be best for treating this patient's coagulopathy? A. Fresh frozen plasma. B. Cryoprecipitate. C. Packed red blood cells. D. Platelets.

Answer D: Platelets (This patient received aspirin, a clopidogrel load, and eptifibatide in the catheterization laboratory before surgery: all three are antiplatelet agents with differing mechanisms of action. In addition, his platelets would be the most likely blood product to help decrease his postoperative bleeding (Answer D is correct). Fresh frozen plasma will not be beneficial because his INR is 1.3 (Answer A is incorrect). Cryoprecipitate would also not be as helpful because his fibrinogen is within normal limits (Answer B is incorrect). Packed red blood cells will treat the anemia but not the bleeding (Answer C is incorrect).)

A 24-year-old woman (height 160 cm, weight 86.4 kg) receives a diagnosis of systemic lupus erythematosus based On a typical malar rash, positive antinuclear and anti-double-stranded DNA antibodies, positive antiphospholipid antibodies, hematocrit 30%, and hemoglobin 10 g/dL. Additional laboratory studies reveal platelet count 240,000/mm^3, WBC 3.4 x 10^3 cells/mm^3, SCr 1.3 mg/dL, total bilirubin 2.0 mg/dL, AST 30 IU/L, and hemolysis on blood smear. The patient experiences fatigue but is still able to perform daily living activities. She has splenomegaly without hepatomegaly. No other significant findings are noted after a review of her symptoms, vital signs, physical examination, and routine lipid and glucose studies. Her family history is unremarkable for cardiovascular disease or cancer. Which one of the following is best to recommend for this patient's anemia? A. Pulse intravenous cyclophosphamide 0.5 g/m2 once; repeat in 4-6 weeks if needed. B. Mycophenolate mofetil 1000 mg twice daily. C. Hydroxychloroquine 200 mg/day. D. Prednisone 0.5 mg/kg/day.

Answer D: Prednisone 0.5 mg/kg/day. (Prednisone 0.5 mg/kg/day is the best answer because corticosteroids are the first-line treatment for immune-mediated hemolytic anemia (Answer D is correct). Cyclophosphamide and mycophenolate mofetil are not necessary and would put the patient at risk of unnecessary immunosuppression and other adverse effects (Answer A and Answer B are incorrect). Hydroxychloroquine does not have consistent effects on acute disease manifestations (Answer C is incorrect).)

A patient from the anticoagulation clinic has found a Web site that provides a comparison between dabigatran and warfarin for atrial fibrillation. The patient insists on switching to this new therapy because this Web site states that dabigatran is more effective than warfarin and does not require any laboratory tests. When you inquire about the source of the information, the patient tells you the Web site name is "Dean's Stroke Musings." The patient has read several other Internet testimonials on treatment with dabigatran and the ways in which it has changed the writer's life. Which one of the following is the best education point to give this patient on using the Internet for treatment information? A. Discourage the patient from using blogs and patient testimonials for advice because they lack objective information to guide treatment. B. Discourage the use of any Internet information by this patient. C. Redirect the patient to WebMD to compare treatments for atrial fibrillation. D. Provide the patient with standard questions to determine Web site quality and provide examples of high-quality Web sites.

Answer D: Provide the patient with standard questions to determine Web site quality and provide examples of high-quality Web sites. (Redirecting the health care consumer to better Web sites and providing tools to use when searching for health-related information provide a platform for the pharmacist to interact with the consumer and serve as a guide to better health information. This approach also allows a dialogue with the patient about looking at testimonials and blogs without appearing too critical of the patient's choice of Web sites (Answer D is correct). Discouraging and cautioning the patient-consumer are more critical in nature, may be interpreted as judgmental, and may potentially damage the patient/provider relationship. Furthermore, this does not provide an opportunity for the pharmacist to educate the patient and provide alternatives (Answer A and Answer B are incorrect). WebMD may have a good discussion of atrial fibrillation; however, this information does not directly answer the consumer's question related to the comparison of the drugs (Answer C is incorrect). Moreover, use of WebMD does not open a dialogue with the health care provider.)

A 63-year-old man with HTN is referred to you for cardiovascular risk reduction, He takes lisinopril 20 mg daily and furosemide 20 mg daily. His BP is 135/78 mm Hg. Pertinent laboratory values include K 4.3 mEq/L and SCr 1.0 mg/dL. Fasting lipid profile is TC mg/dL, LDL-C 126 mg/dL, HDL-C 49 mg/dL, and TG 145 mg/dL. Using the Cohort Equation, his 10-year risk of atherosclerotic cardiovascular disease (ASCVD) is 14.3%. Which one of the following is to recommend for this patient? A. Ezetimibe 10 mg daily. B. Simvastatin 10 mg daily. C. Pravastatin 20 mg daily. D. Rosuvastatin 10 mg daily

Answer D: Rosuvastatin 10 mg daily. (According to the recent guideline recommendations, this meets the criteria for a statin-benefitting group (i.e., 40-75 years of age with LDL-C 70-189 mg/dL and a 10-year ASCVD risk of 7.5% or higher) and should receive moderate-to high-intensity statin therapy. Rosuvastatin 10 mg daily is the only agent listed that meets the criteria of a moderate- to high-intensity statin (Answer D is correct). Simvastatin 10 mg and pravastatin 20 mg daily are examples of low-intensity statin therapy (Answer B and Answer C are incorrect). Ezetimibe monotherapy has not been shown to decrease ASCVD-related mortality and is not recommended in the current guidelines (Answer A is incorrect).)

A 68-year-old woman presents to the clinic with complaints of dizziness, headaches, and difficulty concentrating. Her medical history is significant for H TN, hyperlipidemia, atrial fibrillation, diabetes mellitus, and depression. Her home drugs include amlodipine 10 mg daily, metoprolol 25 mg daily, atorvastatin 10 mg daily, dabigatran 150 mg twice daily, metformin 1000 mg twice daily, and sertraline 75 mg at bedtime. Serum chemistry values are normal except for sodium 124 mg/dL. Other laboratory values include TSH 1.8 mU/L, AIC 7.20/0, serum osmolality 265 mOsm/kg, urine sodium 42 mmo1/L, and urine osmolality 520 mOsm/kg. She has no recent history of nausea or vomiting or change in dietary status. On physical examination, no edema is noted. Which one of the following is most likely responsible for the patient's current laboratory profile? A. Amlodipine. B. Dabigatran. C. Metformin. D. Sertraline.

Answer D: Sertraline. (This patient's presentation and laboratory profile suggest syndrome of inappropriate antidiuretic hormone secretion (SIADH). Laboratory findings in diagnosis of SIADH include euvolemic hyponatremia less than 134 mEq/L, plasma osmolality less than 275 mOsm/kg, urine osmolality greater than 100 mOsm/kg, and urine sodium concentration greater than 40 mEq/L with normal dietary salt intake. Selective serotonin reuptake inhibitors (SSRIs) such as sertraline are known to cause hyponatremia and SIADH (Answer D is correct). Patients older than 65 are at increased risk of SIADH- and SSRI-induced hyponatremia. The patient's other drugs are not associated with SIADH (Answer A, Answer B, and Answer C are incorrect).)

The parents of a 6-week-old girl report that she has had bloody stools for about 2 days. The infant appears otherwise healthy, is afebrile, and has no abdominal distention. She is currently receiving a standard term infant formula at 2 oz every 3 hours. The primary care provider suspects cow's milk-protein allergy (CMA). Using the info below, which infant formula would be best to recommend for this infant? -------------------------------------------------------------------- Enfamil Gentlease [Carbohydrate source; Corn Syrup solids, lactose 20% || Protein Source; Nonfat cow's milk, whey protein concentrate, taurine, I-carnitine || Fat Source; Palm olein, soy, coconut, and high oleic sunflower oil, DHA, ARA || Other; 20% of lactose in standard formulas] Gerber Good Start Soy [Carbohydrate Source; Corn maltodextrin, sucrose || Protein Source; hydrolyzed soy protein isolate, I-methionine || Fat Source; Palm olein, soy, coconut, and high oleic safflower or sunflower oil || Other; Milk- and lactose-free, kosher] Nutramigen AA LIPIL [Carbohydrate Source; Corn syrup solids, modified tapioca starch || Protein Source; Free amino acids || Fat Source; MCT 0% || Other; Lactose-free] Similac Expert Care Alimentum [Carbohydrate Source; Corn maltodextrin, sucrose || Protein Source; Casein hydrolysates, I-cystine, I-tyrosine, I-tryptophan|| Fat Source; High oleic safflower, MCT, soy oils, MCT 50%, LCT 50%|| Other; Lactose-free] -------------------------------------------------------------------- A. Enfamil Gentlease. B. Gerber Good Start Soy. C. Nutramigen AA LIPIL. D. Similac Expert Care Alimentum.

Answer D: Similac Expert Care Alimentum. (Infants with CMA can present with a variety of symptoms, which can be categorized as (l) immunoglobulin E (IgE)-mediated or (2) non—lgE-mediated. Colitis, as evidenced by blood in the stool, is a non—lgE-mediated symptom. Infants with CMA and non—lgE-mediated symptoms should receive a non—cow's milk-based, extensively hydrolyzed formula, such as Similac Expert Care Alimentum (Answer D is correct). Soy protein—based formulas such as Gerber Good Start Soy can be used in patients with IgE-mediated symptoms such as urticaria, wheezing, and anaphylaxis; however, this infant does not have these symptoms (Answer B is incorrect). Reduced-lactose formulas such as Enfamil Gentlease may be useful in infants with symptoms of lactose intolerance, but not in those with CMA (Answer A is incorrect). Although a free amino acid formula such as Nutramigen AA LIPIL would be tolerated in an infant with CMA, the cost and palatability of these products make them most appropriate for the very small number of infants (less than 10%) whose CMA-associated symptoms do not respond to an extensively hydrolyzed product (Answer C is incorrect).)

A 27-year-old woman who received a diagnosis of PCOS 6 years ago presents to the fertility clinic. Although she stopped taking a combined oral contraceptive 3 months ago, she has been unable to conceive. She currently takes metformin, and her PCOS symptoms appear to be well controlled. On physical examination, her blood pressure is 118/76 mm Hg, heart rate is 88 beats/minute, and body mass index is 30.8 kg/m2. Her fasting lipid profile is within normal limits. Which one of the following is best to recommend to help improve fertility in this patient? A. Start letrozole 2.5 mg/day orally. B. Undergo laparoscopic electrocautery of the ovaries. C. Start medroxyprogesterone 10 mg/day orally. D. Start clomiphene 50 mg orally two times/day.

Answer D: Start clomiphene 50 mg orally two times/day. (Letrozole 2.5 mg daily has limited data supporting its efficacy in PCOS and is a second-line therapy (Answer A is incorrect). Laparoscopic electrocautery of the ovaries should be reserved for patients whose PCOS-associated infertility is resistant to or intolerant of ovulation-inducing drugs (Answer B is incorrect). No evidence supports the use of medroxyprogesterone for PCOS-associated infertility (Answer C is incorrect). Clomiphene is considered first-line therapy for anovulatory infertility and may increase fertility when used with metformin (Answer D is correct).)

An 18-year-old man received a heart transplant 5 years ago. His graft function has been stable, and he presents today to the internal medicine clinic for a well-patient visit. He is accompanied by his mother and would like to receive all vaccines possible before starting college in the fall. Which one of the following vaccine regimens is best to recommend for this patient? A. Intranasal influenza and pneumococcal. B. Pneumococcal and varicella. C. Varicella and diphtheria and reduced tetanus toxoids and acellular pertussis (Tdap). D. Tdap and meningococcal.

Answer D: Tdap and meningococcal. (Live vaccines are contraindicated after transplantation secondary to the risk of infection with the live virus in the vaccine. In addition, vaccines must be timed appropriately. During the first several months after transplantation, or after the treatment of rejection with lymphocyte-depleting therapies, the host has an inability to mount a proper immune response to any vaccine, live or inactivated; therefore, providers are encouraged to wait at least 3 months after transplantation or lymphocyte-depleting therapies. Answer D is correct because both vaccines are safe to administer to a transplant recipient and are indicated according to the most recent CDC guidelines. Live vaccines such as varicella and intranasal influenza are contraindicated in a solid organ transplant recipient (Answer A, Answer B, and Answer C are incorrect).)

You are analyzing data from a case-control study of the effect of drug A on cardiovascular events. As you review the data, you realize that some individuals were classified as having received the drug when they did not, and that the distribution of these individuals is similar between cases and controls. Which one of the following is the most likely consequence of this misclassification? A. The estimate will be biased either away from or toward the null. B. The misclassification will not result in bias of the estimate. C. The estimate will be biased away from the null hypothesis of no effect. D. The estimate will be biased toward the null hypothesis of no effect.

Answer D: The estimate will be biased toward the null hypothesis of no effect. (The most likely consequence of nondifferential misclassification (which is the case here) is that estimate will be biased toward the null hypothesis of no effect (Answer D is correct). A differential misclassification occurs when the estimate is biased either away from or toward the null (Answer A is incorrect). Nondifferential misclassification generally produces bias toward the null (Answer B is incorrect). A bias away from the null hypothesis of no effect is incorrect because this phenomenon is a possible effect of differential misclassification (Answer C is incorrect).)

A 45-year-old man (height 6'2", weight 100 kg [220.5 lb] Temp 37 oc [98.60F]) with chronic HCV cirrhosis (Child-Pugh class C) presents to the emergency department with abdominal distension. He reports no known drug allergies. Abdominal ultrasonography shows large ascites, and diagnostic abdominal paracentesis shows an ascitic protein of I g/dL, albumin 1.4 g/dL, and WBC 17 x 103 cells/mm3 with 75% neutrophils. The patient's SCr is 1.4 mg/dL, and serum albumin is 2.2 g/dL. The rest of the examination is normal. Which one of the following therapies is best to initiate in this patient? A. Ciprofloxacin 500 mg by mouth twice daily for 5 days, followed by oral trimethoprim/sulfamethoxazole. B. Intravenous cefotaxime I g every 8 hours for 5 days. C. Intravenous cefotaxime 2 g every 8 hours for 7 days, followed by oral ciprofloxacin 500 mg/week. D. Trimethoprim/sulfamethoxazole I double-strength tablet by mouth once daily.

Answer D: Trimethoprim/sulfamethoxazole 1 double-strength tablet by mouth once daily. (According to the AASLD guideline, this patient meets the criteria for spontaneous bacterial peritonitis (SBP) prophylaxis (ascitic protein less than 1.5 g/dL and SCr of 1.2 mg/dL or greater) and should receive therapy for prevention, which includes trimethoprim/sulfamethoxazole (Answer D is correct). This patient does not present with SBP on the basis of polymorphonuclear neutrophils and symptoms/signs (increase in WBC or temperature); intravenous cefotaxime is used for the treatment of SBP (not prophylaxis) and therefore should not be initiated (Answer B and Answer C are incorrect). In addition, ciprofloxacin twice-daily dosing is used for the treatment of SBP (not prophylaxis) and should not be recommended for SBP prevention (Answer A is incorrect).)

A 31-year-old woman (height 5'4", weight 65 kg [143.3 lb]) with HIV infection is admitted to the medical ward secondary to a compromised respiratory status. She reports a 2-week history of cough and chest pain that has steadily worsened. Her platelet count is 123,000 cells/mm3; other laboratory test results are as follows: LDH 400 IU/L, SCr 1.1 mg/dL, BUN 24 mg/dL, and Pa02 76 mm Hg. Her most recent CD4+ count was 61 cells/mm3, and she takes no antiretrovirals or prophylaxis medications. Chest radiography shows bilateral infiltrates with a ground-glass appearance. She is placed on supplemental oxygen with a 3-L nasal cannula; subsequently, her Sa02is 98%. Proper diagnostics are initiated. Which one of the following is the best empiric therapy for this patient's pulmonary infection? A. Trimethoprim/sulfamethoxazole 400 mg intravenously every 8 hours and prednisone 40 mg twice daily to taper. B. Levofloxacin 750 mg intravenously daily, clindamycin 600 mg intravenously every 6 hours, and primaquine 30 mg orally daily. C. Atovaquone 750 mg orally twice daily, dapsone 100 mg/day, ceftriaxone I g intravenously daily, and azithromycin 500 mg intravenously daily. D. Trimethoprim/sulfamethoxazole 300 mg intravenously every 6 hours and levofloxacin 750 mg intravenously daily.

Answer D: Trimethoprim/sulfamethoxazole 300 mg intravenously every 6 hours and levofloxacin 750 mg intravenously daily. (The patient's presentation is consistent with P. jirovecii pneumonia (formerly PCP), although community-acquired pneumonia (CAP) has not been ruled out. The first-line therapy for PCP is trimethoprim/sulfamethoxazole, and empiric CAP treatment with levofloxacin is appropriate (Answer D is correct). A prednisone taper is not indicated with this woman's respiratory status and apparent mild disease with a PaO2 greater than 70 mm Hg. In addition, empiric therapy should include treatment of CAP until the diagnosis of PCP is confirmed (Answer A is incorrect). Clindamycin and primaquine are second-line therapy, and the patient has no contraindication to trimethoprim/sulfamethoxazole therapy (Answer B is incorrect). Atovaquone is also considered an alternative therapy, and dapsone does not provide additional benefit to the regimen (Answer C is incorrect).)

Your clinical manager has instructed you to create a survey to assess the information literacy of all pharmacists and technicians at your institution. Before administering the survey to the entire department, you ask two pharmacists and two technicians whether the survey measures what is intended. Which one of the following best describes what you are ensuring with this added step? A. Reliability. B. Sensitivity. C. Specificity. D. Validity.

Answer D: Validity. (Validity is a survey's or questionnaire's ability to measure what is intended; obtaining the opinion of likely respondents is an acceptable method to ensure validity and is termed face validity (Answer D is correct). Reliability means that the survey taken by the subject under the same conditions would yield the same result (Answer A is incorrect). Sensitivity pertains to the test's ability to identify positive results; this is the proportion of people known to have a disease who will test positive for it (Answer B is incorrect). Specificity is the ability of a test to show a negative result; this is the proportion of patients known not to have the disease who will test negative for it (Answer C is incorrect).)

A 10-year-old boy has been given a diagnosis of absence seizures. He has been treated with ethosuximide, but he is still experiencing breakthrough seizures. Which one of the following medications would be best to try next in this patient? A. Oxcarbazepine. B. Phenobarbital. C. Topiramate. D. Valproic acid.

Answer D: Valproic acid. (Ethosuximide and valproic acid are the only two agents that have been FDA indicated for use in absence seizures. Valproic acid should be used next in light of recent breakthrough seizures with ethosuximide (Answer D is correct). Oxcarbazepine, phenobarbital, and topiramate do not have an established role in the therapy of absence seizures (Answer A, Answer B, and Answer C are incorrect).)

A 75-year-old man has been referred to your service for evaluation of his current therapy. His medical history includes prostate cancer with bone metastasis and cerebrovascular accident. He has no history of fragility fracture. Because of swallowing difficulties after his stroke, the patient receives enteral nutrition and all of his drugs through a nasogastric tube. His current drugs include atorvastatin 20 mg at bedtime, aspirin 81 mg/day, and lisinopril 20 mg/day. Results of his DXA are as follows: Region || T-Score || Z-Score -------------------------------------- L1-L4 || -3.0 || -1.9 -------------------------------------- Femoral Neck || -2.7 || -1.6 -------------------------------------- Total hip || -2.8 || -1.7 Which one of the following is best to recommend for this patient? A. Testosterone. B. Risedronate delayed release. C. Teriparatide. D. Zoledronic acid.

Answer D: Zoledronic acid. (Zoledronic acid is a first-line recommendation because it decreases vertebral, nonvertebral, and hip fractures. In addition, it is available in a once-yearly intravenous formulation, which might be helpful for patients with a nasogastric tube (Answer D is correct). Testosterone therapy has no fracture prevention data and is contraindicated in patients with a history of prostate cancer (Answer A is incorrect). Risedronate delayed release cannot be crushed or split; thus, it cannot be administered through a nasogastric tube (Answer B is incorrect). Teriparatide, which protects against vertebral fractures only, is indicated for severe osteoporosis or bisphosphonate failures. In addition, teriparatide should not be used in patients with a history of bone metastasis (Answer C is incorrect).)

A 45-year-old woman with a history of type 2 diabetes mellitus with related neuropathy, hypertension (HTN), and moderate hepatic dysfunction secondary to chronic alcohol abuse comes to the clinic with complaints of chronic pain and fatigue during the past 4 months. She is given a diagnosis of fibromyalgia. The attending clinic physician asks for your recommendation for therapy to manage both her diabetic neuropathy and her fibromyalgia-related pain. Which one of the following is best to recommend for this patient? A. Amitriptyline 75 mg at bedtime. B. Gabapentin 600 mg twice daily. C. Fluoxetine 80 mg daily. D. Pregabalin 75 mg twice daily.

Answer D: pregabalin 75 mg twice daily. (Pregabalin is U.S. Food and Drug Administration (FDA) approved for neuropathic pain. fibromyalgia, and postherpetic neuralgia. It is 90%—98% excreted unchanged in the urine and does not require dose adjustment in hepatic impairment. Pregabalin 75 mg twice daily is the recommended starting regimen (Answer D is correct). Amitriptyline used for chronic pain is considered off-label for these indications. Amitriptyline is metabolized in the liver and requires dose adjustments in the setting of hepatic dysfunction; therefore, it is not the best choice for this patient (Answer A is incorrect). Fluoxetine has been used in the treatment of fibromyalgia but is not preferred to pregabalin. Moreover, because of the patient's underlying hepatic dysfunction, the starting dose of 80 mg is not recommended (Answer C is incorrect). Gabapentin is indicated for diabetic-related neuropathy and has used in patients with fibromyalgia; however, the starting dose should be 300 mg daily and titrated to the desired effect (Answer B is incorrect).)

A 39-year-old man with asthma presents to the clinic for a follow-up amy»intrnent. His medical history includes very poorly controlled asthma, erectile dysfunction, degenerative disk disease, and hypertension. His current drugs include mometasone/formoterol 200/5 mcg 2 puffs twice daily, albuterol HFA I or 2 puffs every 4-6 hours as needed for SOB/wheezing, sildenafil 50 mg as needed, naproxen 500 mg twice daily, and hydrochlorothiazide 12.5 mg once daily. His vital signs include oxygen saturation 97% on room air, blood pressure (BP) 130/74 mm Hg, heart rate (HR) 90 beats/minute, and peak expiratory flow 300 L/minute (best of three with adequate effort, personal best 550 L/minute). His social history is positive for tobacco use (1/2 pack/day), and he states that he does not drink alcohol or take illicit drugs. Induced sputum sample is positive for WBCs: neutrophils predominant. Other test results are fraction of nitric oxide (FeNO) 8 ppb and immunoglobulin E (IgE) 250 U/L. Skin prick testing is negative for grasses, trees, and animal allergens. At his previous evaluation 4 months ago, pulmonary function test (PFT) results for FEV(1) and forced vital capacity (FVC) were as follows: Test || Pre-Bronchodilator || Post-Bronchodilator ------------------------------------------------------ FEV (1) || 72% || 80% ------------------------------------------------------ FEV/FVC || 59 || 82 ------------------------------------------------------ FVC || 73L || 79L What is the best to add to this patient's regimen?

Tiotropium DPI 18 mcg once daily. (This patient is most likely to benefit from a trial of tiotropium added to his high-dose ICS plus LABA therapy)


Kaugnay na mga set ng pag-aaral

Injury/Illness Prevention & Wellness Protection

View Set

managing people and organizations test 4

View Set

Lewis Ch. 17 - Preoperative Care

View Set